Ευκλειδης Β 66

84
Θέματα Μαθηματικού Διαγωνισμού "ΘΗΣ" 2007

description

 

Transcript of Ευκλειδης Β 66

Page 1: Ευκλειδης Β 66

• Θέματα Μαθηματικού Διαγωνισμού

"ΘΑΛΗΣ" 2007

Page 2: Ευκλειδης Β 66

·-Άλγεβρα

Α' Ενιαίου Λυκείου Θ. Τσιούμας, I. Σιάχος

Μεθοδολογία Άλγεβρας Α Ενιαίου Λυκείου

Μ. Ευσταθίου, Ε. Πρωτοπαπάς

Μεθοδολογία Άλγεβρας Β' Ενιαίου Λυκείου

Γενικής Παιδείας

Γεωμετρία Β' Ενιαίου Λυκείου

Γενικής Παιδείας Ε. Πρωτοπαπάς

μαθηματικά Γ' Evιolou Λuκεlοu

Μαθηματικά r· Ενιαίου Λυκείου

Γ ενικής Παιδείας Μ. Τ σιλπιρίδης

Ε. Πρωτοπαπάς

Αρχέc; οικονομικής θεωρίας Γ Ενιαίου Λυκείου

Μάθημα επιλογής

για όλες τις κατευθύνσεις Π. Μηλίτσης

ΒΙΒΛΙΟΠΩΛΕΙΑ

rιιr Ελλιινιιtιjr

ΙΙ•Βιιμtιrιιtιjr Εr•ι peί •r

.. � �·,� Θ

""'"' �-. ... �:--: ·�·-"'·.�: .. �:�'!'-:";:• .. ;:. �-'" .... c

. .. ,ο Ζιι•λ�;;# "�xit."�

·ιιιιιa·ι:··. {; ··;· . . '

. ';, .. , . :: .. ::��!\�:;;;:.::-; ", (!j� ..

ΕλλΗΝΙΚΗ ΜΑΘΗΜΑηκΗ ΕΤΑΙΡΕΙΑ

,,,,,, ltll ,,,. ,.,..,., Χ. Δ. λi"Ιιnρ:=-.·-:-.-; •.: 5 ι(_ Ch•kr•bartl

Πφιοδι""r'ι Ι!!τtισ1'ημονιιι.ι'ι Έιςδοαη

Ελληνιιc.ή Μαθηματική Ετc:ιιρι:Ια

.)/ �-t });]-�;;] .Jt.�? G: a -�� t;f !lJ ,·!f-.)��:ι:·J �1J -:>ι..'J :�-��=-.:ι} .,;:�} ��!.{) {} p.r.f/tr.J .')f {f .��'?-:.W J}::J�if.�.r/,�:t

V�v��ιdY�μf@v Ε4 Υ�

_:::-__ ::::::=:_---==-� --��

--;;:__ -�-=-

·.

Page 3: Ευκλειδης Β 66

ΕΛΛΗΝΙΚΗ ΜΑΘΗΜΑΤΙΚΗ ETAIPEIA Τεύχος 66 · Οκτώβριος · Νοέμβριος . Δεκέμβριος 2007· Έτος λη' • Ευρώ: 3,50

e-mail: [email protected] www.hms.gr

ΜΑΘΗΜΑΤΙΚΟ ΠΕΡΙΟΔΙΚΟ ΓΙΑ ΤΟ ΛΥΚΕΙΟ

ΠΕΡΙΕΧΟΜΕΝΑ ./ Ιστορικές Μαθηματικές Αναφορές

Έτος Euller 2

./ Μαθηματικοί Διαγωνισμοί "Ο ΘΑΛΗΣ" 2007 6

./ Homo Mathematicus 12

Μαθηματικά Α' Τάξης ./ Άλγεβρα Εξισώσεις 2ου βαθμού. Σύνολα.

Γενικά περί συναρτήσεων. Η συνάρτηση f(χ)=αχ+β 16 ./ Γεωμετρία Παραλληλόγραμμα 25

Μαθηματικά Β' Τάξης ./ Άλγεβρα Πολυώνυμα - Πολυωνυμικές εξισώσεις 34

Γράμμα της Σύνταξης Αγαπητοί συνάδελφοι

./ Γεωμετρία μετρικές αχέσεις στον κύκλο - Εμβαδά 38

Σας ευχόμαστε ολόψυχα καλή χρο­

νιά και ευτυχισμένο το 2008, με υγεία

και δημιουργικότητα για σας και τους μα­

θητές σας. Πρόθεσn μας είναι να παρακολου­

Θούμε με ενδιαφέρον τις μαθηματικές εργα­

σίες που μας στέλνετε. Για τη μαθηματικn

ομοιομορφία των άρθρων Θα μας επιτρέψετε

σε συνεννόηση μαζί σας να γίνονται κάποιες

επεμβάσεις στον τρόπο παρουσίασης. Ας μην

ξεχνάμε ότι την κύρια ευθύνη για την επιστη­

μονικn ορθότητα κάθε επιστημονικού άρθρου

φέρει ο ίδιος ο aρθρογράφος.

./ Κατεύθυνση ευθεία 47 Μαθηματικά Γ' Τάξης

./ Μαθηματικά Κατεύθυνσης Προβλήματα εύρεσης μέγι- Ειδικά, Θέλουμε να πιστεύουμε, ότι η διά­

δοση του περιοδικού για τους μαθητές μας,

είναι σημαντικός στόχος για όλους μας .

στης και ελάχιστής τιμής μιας συνάρτησης 52 ./ Μαθηματικά Γενικής Παιδείας 58 ./ Κατεύθυνση Θεωρήματα συνεχών συναρτήσεων 62 • • • • • • • • • • • • • • • • • • • • • • • • • • • • • • • • • •

Ενώνουμε λοιπόν τις δυνάμεις μας

για την επίτευξη του καλύτερου δυνατού .

./ Τα Μαθηματικά μας Διασκεδάζουν 69

./ Ο Ευκλείδης προτείνει... 71

./ Νιοστή ρίζα 74 ./ Το βήμα του Ευκλείδη 75

Με συναδελφικούς χαιρετισμούς ο Πρόεδρος της Συντακτικnς Επιτροπnς

Γ.Σ. Τοaaόπουλος

Ο Αντιπρόεδρος της Συντακτικnς Επιτροπnς Β. Ευαταθίου

••••••••••••••••••••••••••••••••••••••••••••••••••••••••••••••••••••••••••••••

ΕΚΔΟΣΗ ΤΗΣ ΜΑΘΗΜΑτΙΚΗΣ ΕτΑΙρΕΙΑΣ

ΠΑΝΕΠΙΠΗΜΙΟΥ 34 • 1 06 79 ΑΘΗΝΑ Τηλ.: 21 Ο 3617784 - 3616532 Fax:2103641025 Εκδότης: Αλεξανδρής Νικόλαος

Διευθυντής: Τuρλής Ιωάννης Κωδικι)ς ΕΛ.ΤΑ.: 2055 ISSN: 1105- 7998

Επιμέλεια • Εκδοσης:

Εκτελεaτική Γραμματεία

Πρόεδρος: Ταααόπουλος Γιώργος

Αντιπρόεδρος: Ευσταθίου Βαγγέλης

Γραμματέας: Χριστόπουλος Παναγιώτης

Μέλη: Αργυράκης Δ.

Δρούτaας Π.

Λουρίδας Σ.

Ταπεινός Ν.

Σuνταντική επιτροπή

Αθανασόπουλο$ Γεώργιος Λουρίδας Σωτήρης Αναστασίου ΓιαννΙJς Μαλαφέκας Θανασης ΑνδρουλακάκηςΝικος ΜανοΛάκου Σταματική Αντωνόπουλος Νίκος Μαυρογιαννάκης Λεωνίδας Αργυρ�κης Δημ.ήτ9ιος ΜενδQινός Γιάννης Βακαλοπουλος Κωστας Μεταξά_ς Νικόλαος Γράψας Κων/νος Μυ,λωVας ΔΙJμήτρης Δρουτσας Παναγιώτης Μωκος Χρηστος Ευσταθίου Βαγγέλης Πανουσάκης Νίκος Ζαχαρόπουλος Κων/νος Ρέyκλης _Δημήτρης Ζώτος Βαγγέλfις Σαιτη Ευα Καλίκας Σταμάτης Στα"ίκος Κώστας ΚανέλλΟς Χρήστος Στά"ίκος Παναγιώτης Καραγκού�ης Δ!Jμήτρης Στρατής Γιάννης Καρακατσανης Βασίλης Ταπεινός Νικόλαος Καρκάνης Βασίλης Τασσόπουλος Γιώργος Κατσούλη_ς Γιώργος τζιώτζιος Θανάσης Κερασαφδης Γιαννης Τριάντος Γεώργιος Καρδαμιτσης Σπύρος Τσαγκάρης Ανδρέας Κnπουρός Χρήστος Τσατούρα_ς Ευάγγελος ΚΛάδη Κατερίνα Τσικα.λσuδάκη� Γιώργος Κόντζια_ς Νίκος Τσιού;.ιας Θανασης

Ταaaόπουλος Γιώργος Κοτσιφqκης Γιώργος_ Τυρλ_ης �ωάννης Κυριακοπουλος Αντωνης Φανέλη Άννυ Σύνθεaη εξωφύλλου:

Ευaταθίου Βαγγέλης Κυ�ιακόπουλος Θανάσης Χαραλάμποπούλου Λίνα Κυ ερν!Ιτου Χqψιτ. Χαραλάμπους Θάνος Λα αρίδης Χρηστος Χqιστόπουλος Παναγιώτης

από αυνάρτηaη Eϋler και Modulαr Form

...................................•........ f����:���� ....... r��������� ..... .

• Τα διαφημιζόμενα βιβλία δε σημαίνει ότι προτείνοvtαι από την Ε.Μ.Ε. • Οι συνεργάτες, τα άρθρα, οι προτεινόμενες ασκήσεις, οι λύσεις ασκήσεων κτλ. πρέπει να στέλνοvtαι έγκαιρα, στα γραφεία της

Ε.Μ.Ε. με την ένδειξη "Για τον Ευκλείδη β'". Τα χειρόγραφα δεν επιστρέφονται. Τιμή Τεύχους ευρώ 3,50

Ετήσια συνδρομή (12,00 + 2,00 Ταχυδρομικά= ευρώ 14,00)

Ετήσια συνδρομή για Σχολεία ευρώ 12,00

Το αντίτιμο για τα τεύχη που παραγγέλνοvtαι στέλνεται με απλή επιταγή σε διαταγή Ε.Μ.Ε. Ταχ. Γραφείο Αθήνα 54 Τ. Θ. 30044 ή πληρώνεται στα γραφεία της Ε.Μ.Ε.

Εκτύπωση: /ΝτΕΡΠΡΕΣ Α.Ε. τηλ.: 210 8160330 Υnειίθuνος τunογpαφείοu: Β. Σωτηkης

Page 4: Ευκλειδης Β 66

2007: ΕΤΟΣ EULER «Διάβασε Euler ... , διάβασε Euler. Αυτός είναι δάσκαλος για όλους μας».

Laplace

Αθανασίου Λιπορδέ�η

Ο Κ. ΚΑΡΑΘΕΟΔΩΡΗ ΥΠΗΡΞΕ ΕΚΔΟΤΗΣ ΣΗΜΑΝΤΙΚΟΥ ΒΙΒΛΙΟΥ ΜΑΘΗΜΑΤΙΚΩΝ ΤΟΥ EULER

Στη διαχρονική πορεία της επιστήμης ξεχωρίζου\' οι υ:ϊ:ε�""�βα­τικοί αριθμοί π, e και ο φανταστικός ί με του; ο;τοίου; ο Euler έφτιαξε το ποίημα των μαθηματικών όπω; α;τοιωί.είται η σχέση eiπ = - 1 . Ο e είναι αριθμός του Euler, που βρέθηκε σε επιστολή του στον Ι . Bemoulli .

Ο Leonhard Euler ή ελληνιστί Ευλήρος γεννιέται στη Βασιλεία της Ελβετίας στις 15 Απριλί­ου 1707. Ο πατέρας του ήταν μαθητής του Ιακώβου Bemoulli και ήταν αυτός που του έδειξε τα πρώτα βήματα στα Μαθηματικά.

Γνωρίστηκε γύρω στα 1720 με τον Ιωάννη Bemoulli , που την εποχή εκείνη, εθεωρείτο ο με­γαλύτερος μαθηματικός αφού ο Leibniz είχε πεθάνει και ο Newton, ήταν πλέον αρκετά μεγάλος. Ο Bemoulli aντιλαμβανόμενος το ταλέντο του Euler τον ενθαρρύνει προς τα μαθηματικά και προσφέρεται να του παραδίδει δωρεάν μαθήματα.

Το 1724 παίρνει το βαθμό του «Magister» και εκφωνεί λόγο με θέμα την σύγκριση της φι­λοσοφίας του Newton με αυτήν του Descartes προκαλώντας ευοίωνες εντυπώσεις. Δύο χρόνια αργότερα δημοσιεύει ένα υπόμνημα με τίτλο Constructio Linearum in Medio quocunque resis­tente (Κατασκευή ισόχρονων γραμμών εντός μέσου, πανταχόθεν ανθισταμένου) προκαλώντας τον ενθουσιασμό του δασκάλου του Ι. Bemoulli που σε μία επιστολή του τον αποκαλεί «πρίγκι­πα των Μαθηματικών» και «ασύγκριτον άνδρα». Το 1727 η Ακαδημία επιστημών των Παρισίων του παραχωρεί το «accessit» για ένα υπόμνημα επί του εξαερισμού των πλοίων.

Με ατυχή παρότρυνση συμμαθητών μεταβαίνει στην Αγία Πετρούπολη σε μία μη γόνιμη εποχή για τις επιστήμες στην Ρωσία. Παρ' όλα αυτά καταφέρνει να αναδειχτεί σε αυθεντία στις

ΕΥΚΛΕΙΔΗΣ Β' 66 τ.2/2

Page 5: Ευκλειδης Β 66

------------- Ιστορικές Μαθη ματικές Αναφορές -------------

ναυτικές επιστήμες. Το 1730 παίρνει την θέση του καθηγητού της Φυσικής και το 1733 διαδέχεται στην Ακαδη­

μία ως μέλος τον Δανιήλ Bemoulli και έτσι του δίδεται χρόνος και ευκαιρία να ασχοληθεί με την έρευνα οπότε συγγράφει την μηχανική της κίνησης των βημάτων που εκδίδει η ίδια η Ακα­δημία .

Παρ' ότι ακολουθεί μία περίοδος πολιτικής αστάθειας στην Ρωσία το 174 1 ο Φρειδερίκος ο 2°ς ο μέγας τον στέλνει να αναδιοργανώσει στο Βερολίνο την Ακαδημία που φιλοτέχνησε ο Leibniz.

Το σχέδιο όμως ναυάγησε εξ' αιτίας του πολέμου της Σιλεσίας και έτσι ο Eu1er ευρισκόμε­νος χωρίς επίσημα καθήκοντα καταλήγει να κάνει ιδιαίτερα μαθήματα σε παιδιά βασιλικών οι­κογενειών. Σημαντικοί μαθητές του Eu1er υπήρξαν, ο γιος του, από τον πρώτο του γάμο Jean A1bert Eu1er. Ο Cristian Go1dbach, ο Nico1as Fuss, ο Pau1 Henri. Ο G. W. Κrafft και ο Α. J. Lex­ell που άφησαν με το έργο τους το όνομά τους στην ιστορία της επιστήμης.

Την διετία 1744-1746 γράφει 35 πολύτιμες διατριβές με ποικιλία επιστημονικών θεμά­των και οι οποίες γνωρίζουν μεγάλη εκδοτική επιτυχία γραμμένος και σε άλλες γλώσσες!!!

Εν τω μεταξύ το 1743 ο Φρειδερίκος ο 2°ς ο μέγας ιδρύει την Ακαδημία των επιστημών και των γραμμάτων του Βερολίνου και αναθέτει το μαθηματικό τμήμα στον Eu1er. Έτσι από το 1745 έως το 1769 δημοσιεύονται 25 τόμοι και 330 υπομνήματα!!!

Λέγεται ότι ο Eu1er χάνοντας τις ελπίδες του να διαδεχθεί τον Maupertuis στην προεδρία της ακαδημίας του Βερολίνου, το ζητά από τον βασιλιά της Πρωσίας και ενδίδει στις δελεαστικές προκλήσεις της Αικατερίνης της 2ης για να αναλάβει διευθυντής της Ακαδημίας Πετρούπολης.

Οι ατυχίες όμως, συνεχίζουν να σφυροκοπούν τον μέγα μαθηματικό . Το 1760 είδε το σπίτι του στο Βερολίνο να πλήττεται από τον Ρωσικό στρατό, τώρα κατά την μετάβασή του στην Πε­τρούπολη τα υπάρχοντά του ακόμη και χειρόγραφα, χάνονται σε ναυάγιο .

Μόλις φθάνει στη νέα του διαμονή, ασθενεί βαρύτατα και ενώ από το 1735 είχε χάσει την όραση του δεξιού ματιού του από υπερβολική εργασία κατά τη μελέτη γεωγραφικών χαρτών τώρα αισθάνεται ότι θα τυφλωθεί εντελώς.

Έτσι ο μεγάλος επιστήμων αναζητά τρόπους να γράφει και να διαβάζει χάνοντας το φως του . Σαν να μην έφθαναν όλα αυτά, 6 χρόνια μετά την εγκατάστασή του στην Πετρούπολη το σπίτι του γίνεται παρανάλωμα του πυρός, ενώ ο ίδιος σώζεται, την τελευταία στιγμή και χάρις στον ηρωισμό ενός υπηρέτη του .

Ο Eu1er είχε μία πολυμελή οικογένεια από την αγάπη της οποίας αντλούσε την έμπνευση ενώ από την συμπαράσταση των aφοσιωμένων φίλων του την δύναμη για να συνεχίζει την έ­ρευνά του και με αντίξοες συνθήκες.

Άξιον θαυμασμού είναι το γεγονός ότι το 50% των εργασιών του το έγραψε προς το τέ­λος της ζωής του που ήρθε απροσδόκητα 5 μέρες πριν γεννηθεί ένα άλλο μεγάλο άστρο των μαθηματικών, ο Κ. Καραθεοδωρή ο οποίος υπήρξε μετέπειτα προς μεγάλη μας χαρά και ικανοποίηση εκδότης ενός έργου του Euler για τον λογισμό των μεταβολών που θα κοσμεί το μουσείο Καραθεοδωρή στην Κομοτηνή.

Η επιστήμη υποκλίνεται στον τιτάνα που άνοιξε τόσους νέους δρόμους σε όλους τους κλά­δους των καθαρών και εφηρμοσμένων μαθηματικών. Τριακόσια χρόνια πέρασαν από την γέν­νησή του και πολύ σωστά η επιστήμη απεκάλεσε το έτος 2007 ΕΤΟΣ EULER.

ΕΥΚΛΕΙΔΗΣ Β ' 66 τ.2/3

Page 6: Ευκλειδης Β 66

------------- Ιστορικές Μαθηματικές Αναφορές -------------

< . ·

· ..•. ' :.··

Το έργο του σίγουρα δεν μπορούμε να τολμήσουμε, να το συμπεριλάβουμε στο βιογραφικό αυτό αφιέρωμα. Αμφιβάλλουμε αν αρκεί ένα περιοδικό και μόνο για σύντομες αναφορές. Θα αναφέρουμε όμως μερικούς τύπους και θεωρήματα που οφείλονται στον Euler και ξεχωρίζουν ως φάρος στον ωκεανό της μαθηματικής επιστήμης.

lim (ι+ α )ν = eα V---H-00 χ

eix - e -ix ημχ= � eix +e

συνχ = --

2 -ix foo ημχ dx =Π ο χ 2

Επιγραμματικά αναφέρουμε την συμβολή του στην Άλγεβρα για τη θεωρία εξισώσεων. Στη Θεωρία Αριθμών είχε θαυμαστές αποδείξεις και θεωρήματα που αποδίδονται σ' αυτόν το μέγα επιστήμονα που είχε την έμπνευση να αραιώσει τα σκοτάδια που περιβάλλουν τους πρώτους α­ριθμούς, ρίχνοντας φως από την γνώση του και την ευφυί'α του.

Στην aπειροστική ανάλυση με απαράμιλλη δεξιοτεχνία και ενίοτε ανησυχητική άνεση όπως τον παρατηρούσε ο Δανιήλ Bemoulli χειρίζεται τις σειρές χωρίς να διαχωρίζει τις αποκλί-

2 00 1 ' κλ' Σ' ' 'λ ' λ ' π Σ ' νουσες απο τις συγ ινουσες. αυτον οφει εται και η ανακα υψη του τυπου - = -2 ενω 6 ι η ' ' 11' ( 1 1 1 11 ) θ ' ' ' ' ' στην σημαντικη σχεση .ιιm -+- + ... +--.ι η χ = γ, η στα ερα γ φερει το ονομα του και ειναι

Χ-Η«> 1 2 Χ γ = Ο, 57722 1 56649 ... χωρίς να είναι ακόμη γνωστό, αν είναι ρητός ή άρρητος.

Στην ανάλυση, ο άοκνος εργάτης προσφέρει αναρίθμητες βελτιώσεις στις παραγώγους και τα ολοκληρώματα. Ενδεικτικά αναφέρουμε την εισαγωγή και επιβολή απ' αυτόν του συμβόλου of I 8χ για τις μερικές παραγώγους, η αντιστροφή της τάξεως των παραγωγίσεων. Εις αυτόν α­ποδίδονται τα «Ολοκληρώματα Euler πρώτου και δεύτερου είδους», καθώς και ο τρόπος προσ­διορισμού ιδιοτήτων ολοκληρωτικών καμπύλων, ο οποίος μένει στάσιμος για ένα περίπου αιώνα μετά τον Euler.

Ασχολήθηκε με τα ισοπεριμετρικά προβλήματα και το 1 744 γράφει ειδικό υπόμνημα το ο­ποίο θεωρείται ο πρόδρομος του λογισμού των μεταβολών στον οποίο ένας άλλος πολύ μεγάλος μαθηματικός, ο δικός μας ο Κ. Καραθεοδωρή θα αναδειχτεί σε αυθεντία του είδους. Απ' αυτό το

ΕΥΚΛΕΙΔΗΣ Β' 66 τ.2/4

Page 7: Ευκλειδης Β 66

------------- Ιστορικές Μαθη ματικές Αναφορές -------------

υπόμνημα του Euler εμπνεύστηκε και ο Langrange τις βάσεις του νέου αυτού αλγορίθμου. Στην στοιχειώδη Γεωμετρία και τριγωνομετρία πραγματικά εκπλήσει το γεγονός ότι ο

ασχολούμενος με την μελέτη του ωκεανού της επιστήμης (Ανάλυση) θα εύρισκε χρόνο να ασχο­ληθεί με την κλασική γεωμετρία και τριγωνομετρία.

Σ' αυτόν οφείλεται η σχέση BC · AD + CA · BD + ΑΒ · CD = Ο μεταξύ 4 σημείων μίας ευθείας της οποίας είναι γνωστή η εφαρμογή στη θεωρία των διπλών λόγων τετράδας σημείων. Επίσης η σχέση α2 + β2 + γ2 + δ2 = ε2 + ζ2 + 4d2 όπου α, β, γ, δ οι πλευρές ενός τετραπλεύρου ε, ζ οι δια­γώνιοι και d το ευθύγραμμο τμήμα που ενώνει τα μέσα των διαγωνίων. (Η σχέση αυτή βρέθηκε σε επιστολή στον Goldbach το 1745) . Εδώ είναι σημαντικό να γράψουμε ότι η τελική μορφή της εικασίας του Goldbach που ακόμα αναζητά τη λύση της, από την επιστήμη οφείλεται στον Euler.

Πρώτος παρατήρησε ότι το ορθόκεντρο, βαρύκεντρο και περίκεντρο τριγώνου βρίσκονται στην ίδια ευθεία που ονομάστηκε ευθεία του Euler . Σ' αυτόν οφείλονται πολύτιμες γεωμετρικές κατασκευές.

Στην στερεομετρία υπολογίζει τον όγκο τετραέδρου από τις ακμές καθώς και τον τύπο α + β = γ + 2 (αριθμός εδρών, β αριθμός κορυφών, γ αριθμός ακμών).

Στην τριγωνομετρία παρουσιάζει τόσο σημαντικό έργο ώστε να αποκαλείται ο μέγας μεταρ­ρυθμιστής του κλάδου.

Πρωτοστατεί και κάνει σοβαρή και σπουδαία δουλειά και στην αναλυτική γεωμετρία κα­θώς και την απειροστική γεωμετρία χωρίς να κάνει χρήση των παραγώγων για την μελέτη κα­μπύλων από τις εξισώσεις τους.

Στην τοπολογία όλοι γνωρίζουν την συμμετοχή του Euler από το γνωστό πρόβλημα με τις γέφυρες του Κένσιμπεργκ (Konigsberg) πόλης της Γερμανίας που διασχίζει ο ποταμός Pregel δημιουργώντας 5 τόπους.

Το τοπολογικό πρόβλημα στην περίπτωση αυτή είναι ότι πρέπει να επισκεφτούμε όλους τους τομείς της πόλης που δημιουργούνται από την διέλευση του ποταμού περνώντας από κάθε γέφυ­ρα μία μόνο φορά. Όμως μπορούμε να μπούμε και να βγούμε από κάθε τομέα περισσότερες από μία φορές. Οι σκέψεις του επί του ζητήματος αυτού είναι τόσο πρωτότυπες και σημαντικές ώστε να γεννηθεί ο κλάδος της τοπολογίας. Στην επιστολή του στον Goldbach (Γκόλντμπαχ) το 1757 διδάσκει τον τρόπο μετακίνησης του αλόγου πάνω στον πίνακα του ζατρικίου (σκάκι) ώστε να καταλάβει διαδοχικώς και μία μόνο φορά όλες τις κυψέλες του πίνακα. Την μέθοδό του ανέπτυ­ξε σε μία εργασία των χρονικών της Ακαδημίας του Βερολίνου, εμπνέοντας τις μελέτες του με­ταγενέστερου σπουδαίου Γάλλου γεωμέτρη του Vanderrnonde.

ΕΥΚΛΕΙΔΗΣ Β ' 66 τ.2/5

Page 8: Ευκλειδης Β 66

flιιι�J�ιι,�·��·' A.,,l'.,��"''l'"'' -- Μιιd..,pιιτιt.ιιέ.�· OJ41pnιιt.λl.ε�

Επιτροπή Διαγωνισμών της Ε.Μ.Ε.

68ος ΠΑΝΕΛΛΗΝΙΟΣ ΜΑΘΗΤΙΚΟΣ ΔΙΑΓΩΝΙΣΜΟΣ ΣΤΑ ΜΑΘΗΜΑΤΙΚΆ

''Ο ΘΑΛΗΣ" ΣΑΒΒΑΤΟ, 24 ΝΟΕΜΒΡΙΟΥ 2007

Α' τάξη Λυκείου Πρόβλημα ι Δύο παιδιά συζητούν για αλγεβρικά προβλήματα. Ο Γιάννης λέει στη Μαρία: Έχω σκεφτεί δύο ακέραιους αριθμούς χ και y που είναι τέτοιοι

ώστε, αν μειώσω τον χ κατά 50 και αυξήσω τον y κατά 40, τότε το γινόμενό τους δεν μεταβάλλεται. Η Μαρία ρωτάει το Γιάννη : Αν αυξήσεις τον αριθμό χ κατά 100 και μειώσεις τον αριθμό y

κατά 20, τότε πάλι το γινόμενό τους δεν μεταβάλλεται; Ο Γιάννης απαντάει: Πράγματι, αυτό ισχύει. Η Μαρία καταλήγει: Τότε γνωρίζω τους αριθμούς που σκέφθηκες. Έχει δίκιο η Μαρία; Εσείς μπορείτε να βρείτε τους αριθμούς που σκέφθηκε ο Γιάννης; Λύση Σύμφωνα με τη συζήτηση που είχε ο Γιάννης με τη Μαρία, αν x,y είναι οι αριθμοί, τότε θα {xy=(x-50)(y+40)} { 40x-50y=2000} {χ=ΙΟΟ} ισχύουν: xy=(x+IOO)(y- 20) <:::> -20x+l00y=2000 <:::> y=40 ·

Πρόβλημα 2 Α ν α, β, γ ε R με (α -β )(β -γ)( γ - α) "* Ο τότε να υπολογίσετε την τιμή της παράστασης:

Λύση

Α = (α-1 )(α + 1) +

(β -1 )(β + 1) +

(γ -1 )(γ+ 1) . (α-β)(α-γ) (β-α)(β-γ) (γ-α)(γ-β)

Το ελάχιστο κοινό πολλαπλάσιο των παρανομαστών είναι (α-β)(β-r )(r-α)* Ο, Α (β-y)(d -Ι) (y-αx;J -1) (α-fJXY-1) οπότε έχουμε + +---'----'--'-"------'---{α-β)(β-y)(y-α) -{α-β)(β-y)(y-α) -{α-β)(β-y)(y-α) = (β-y)α2 +(y-α)β2 +(α-β)y2 +(β-r+ r-α+α-β) (β- y)α2 + βy(β-y)-α(β2-r2)

(α -β)(β -r )(y -α)

Πρόβλημα 3

(α-β)(β-y)(y-α) = (β- y)(α2 + βy-α(β + y)) = (α-β)( α-y) =Ι. (α -β)(β -r )(y -α) (α -β)(y -α)

Θεωρούμε ισοσκελές τρίγωνο ΑΒΓ με ΑΒ = ΑΓ και Α= 45°. Φέρουμε ευθεία ε κάθετη προς την ΑΓ στο Α η οποία τέμνει την προέκταση της ΓΒ στο Ε. Πάνω στην ευθεία ε παίρνουμε σημείο Δ

ΕΥΚΛΕΙΔΗΣ Β ' 66 τ.2/6

Page 9: Ευκλειδης Β 66

--------- Μαθηματικοί Διαγωνισμοί- Μαθηματικές Ολυμπιάδες --------­

τέτοιο ώστε ΑΔ = ΑΓ με το σημείο Α να βρίσκεται μεταξύ των Ε και Δ. Να υπολογίσετε συναρτήσει της πλευράς ΑΓ = β :

(α ) το εμβαδόν του τετραπλεύρου ΑΒΓΔ, (β ) το μήκος του ευθύγραμμου τμήματος ΑΕ.

Β Γ

ΛίJση (α ) Το τρίγωνο ΑΓΔ είναι ορθογώνιο και ισοσκελές, οπότε Α Γ Δ = 45° . Άρα είναι Α ΓΔ = 45° = Β ΑΓ , οπότε ΑΒ ΙΙΓΔ , αφού τεμνόμενες από την ΑΓ σχηματίζουν δύο εντός εναλλάξ γωνίες ίσες. Άρα το τετράπλευρο ΑΒΓ Δ είναι τραπέζιο με βάσεις ΑΒ =β, ΓΔ = �β2 + β2 = βJ2 και

' ΑΖ ΓΔ βJ2 Ά ' β δ ' υψος = 2 = -2

- . ρα εχει εμ α ον

( ΑΒΓ Δ ) = β + βJ2 · βJ2 = β2 ( 2 + J2)

. 2 2 4

(β) Επειδή είναι ΑΒ 11 Γ Δ τα τρίγωνα ΕΑΒ και ΕΔΓ είναι όμοια, οπότε, αν ΕΑ = χ , θα έχουμε :

�= ΕΔ <:::::>� = χ + t <=:>xJ2 = χ + β<=> x(J2 - 1 ) = β<=> χ = ! = β(J2 + 1 ) . ΑΒ ΔΓ β βν2 ν2-1

Πρόβλημα 4 Να βρεθούν οι θετικοί ακέραιοι αριθμοί x,y που ικανοποιούν τη σχέση :

χ6 + 2x3y2 + 3χ3 + y4 + 3y2 - 40 =Ο

Λύση Η δεδομένη σχέση γράφεται διαδοχικά: χ6 + 2χ3 / + y 4 + 3χ3 + 3/ = 40

(χ3 + /)2 + 3(χ3 + /) +2 = 42

( χ3 + / + Ι)· ( χ3 + / + 2) = 42 .

Οι αριθμοί όμως χ3 + y2 + 1 και χ3 + y2 + 2 , είναι θετικοί ακέραιοι με χ3 + / + Ι < χ3 + / + 2 και γινόμενο 42 = 1 · 4 1 = 2 · 21 = 3 · 1 4 = 6 · 7.

Επομένως θα πρέπει: χ3 + / + 1 = 1 και χ3 + y2 + 2 = 42 (1) 3 2 1 2 3 2 2 2 1 χ + y + = και χ + y + =

χ3 + y2 + 1 = 3 και χ3 + y2 + 2 = 14 (2)

(3) χ3 + / + 1 = 6 και χ3 + / + 2 = 7 ( 4)

Προφανώς οι σχέσεις (1 ) , (2), (3) είναι αδύνατες και από τη σχέση (4) , έχουμε : χ3 + y2 = 5 που αληθεύει για χ = 1 και y = 2 .

Διαφορετικά, θα μπορούσαμε να θεωρήσουμε το τριώνυμο ω2 + 3ω - 40 = Ο, όπου ω = χ3 + /, η οποία, αφού χ, y > Ο έχει τη μοναδική λύση χ3 + y2 = 5 , που αληθεύει μόνο για χ = 1 και y = 2 .

Β' τάξη Λυκείου Π ρόβλη μα 1 Να βρεθούν οι πραγματικοί αριθμοί x,y που ικανοποιούν τη σχέση :

χ6 + χ4 - 2χ3 - 2xzyz - 2yz + 2y4 + 2 = Ο .

ΕΥΚΛΕΙΔΗΣ Β ' 66 τ.2/7

Page 10: Ευκλειδης Β 66

--------- Μαθηματικοί Διαγωνισμοί- Μαθηματικές Ολυμπιάδες ---------

-� ,· 1 ; •. υση Ισοδύναμα από την δεδομένη ισότητα, έχουμε:

� + χ4 - 2χ2 / + / + � = ο <::::> ( χ3 - Ι) 2 + ( χ2 - y2 ) 2 + (/ - Ι) 2 = ο

<::::> ( χ3 - Ι = Ο και χ2 -/ = Ο και y2 - Ι = Ο ) <::::> (χ = Ι και y = Ι) ή (χ = Ι και y = - 1 )

π 1 Να βρεθούν όλες οι δυνατές τιμές των θετικών μονοψήφιων ακεραίων αριθμών κ, λ, μ για τους

οποίους η δευτεροβάθμια εξίσωση κχ2 + λχ + μ = Ο έχει δύο ακέραιες ίσες λύσεις.

Για να έχει η εξίσωση διπλή λύση , πρέπει η διακρίνουσά της να είναι μηδέν. Δ = Ο <::::> λ 2 - 4κμ = Ο <::::> λ 2 = 4κμ .

Στη περίπτωση αυτή η διπλή λύση είναι: χι = χ2 = � 2κ Ο αριθμός 4κμ είναι άρτιος. Άρα και ο λ 2 είναι άρτιος, οπότε ο λ είναι άρτιος. Οι δυνατές τιμές που μπορεί να πάρει ο λ (δεδομένου ότι είναι μονοψήφιος θετικός ακέραιος) είναι:

λ = 2 ή λ = 4 ή λ = 6 ή λ = 8 . Αν λ = 2 τότε : 4 = 4κμ <::::> κμ = Ι , οπότε οι δυνατές τιμές για τα κ και μ είναι κ = Ι και μ = 1 . Αν λ = 4 τότε : Ι6 = 4κμ <::::> κμ = 4 , οπότε οι δυνατές τιμές για τα κ και μ είναι

(κ = 1 και μ = 4 ) ή (κ = 4 και μ = Ι) ή (κ = 2 και μ = 2 ) . Αν λ = 6 τότε : 36 = 4κμ <::::> κμ = 9 , οπότε οι δυνατές τιμές για τα κ και μ είναι

(κ = Ι και μ = 9 ) ή (κ = 9 και μ = 1 ) ή (κ = 3 και μ = 3 ) . Α ν λ = 8 τότε : 64 = 4κμ <::::> κμ = Ι6 , οπότε οι δυνατές τιμές για τα κ και μ είναι

( κ = 2 και μ = 8 ) ή ( κ = 8 και μ = 2 ) ή (κ = 4 και μ = 4 ) . Άρα οι δυνατές τιμές για τη διατεταγμένη τριάδα (κ, λ, μ) είναι:

( Ι, 2, Ι) ' (1, 4, 4)' (2, 4, 2) ' (1 , 6, 9) ' (3 , 6, 3) ' (2, 8, 8) ' (4, 8, 4) ο

Οι άλλες περιπτώσεις απορρίπτονται, διότι δεν δίνουν ακέραια λύση . Οι εξισώσεις που προκύπτουν, με την αντίστοιχη διπλή λύση είναι:

χ2 + 2χ + Ι = Ο με διπλή λύση Χι = χ2 = - 1 , χ2 + 4χ + 4 = Ο με διπλή λύση Χι = χ2 = -2 , χ2 + 6χ + 9 = Ο με διπλή λύση Χι = χ2 = -3 .

ΙΠ Δίνεται ορθογώνιο και ισοσκελές τρίγωνο ΑΒΓ και ημιευθεία Αχ//ΒΓ (η Αχ βρίσκεται στο

ίδιο ημιεπίπεδο με το σημείο Γ ως προς την ευθεία ΑΒ). Στην ημιευθεία Αχ θεωρούμε τα σημεία Δ και Ε έτσι, ώστε το τετράπλευρο ΒΓ ΔΕ να είναι ρόμβος (το σημείο Ε βρίσκεται ανάμεσα στο Α και

στο Δ). Στο σημείο Δ θεωρούμε την κάθετη ευθεία στη ΔΓ που τέμνει την προέκταση

Δ της πλευράς ΒΑ στο Ζ. Να αποδειχθεί ότι το τρίγωνο ΔΕΖ

είναι ισόπλευρο. (β} Να αποδειχθεί ότι το Ε είναι

έγκεντρο του τριγώνου ΑΓΖ.

(α) Εφόσον το ΒΓ ΔΕ είναι ρόμβος, θα ισχύουν οι ισότητες: ΒΓ = Γ Δ = ΔΕ = ΒΕ ( Ι)

Θεωρούμε ΑΛ και ΕΚ κάθετες στη ΒΓ. Τότε ΑΛ = ΕΚ (διότι ΑΛΚΕ είναι

ορθογώνιο παραλληλόγραμμο).

ΕΥΚΛΕΙΔΗΣ Β' 66 τ.2/8

Page 11: Ευκλειδης Β 66

--------- Μαθηματικοί Διαγωνισμοί - Μαθηματικές Ολυμπιάδες --------­

Η ΑΛ είναι διάμεσος στο ορθογώνιο τρίγωνο ΑΒΓ, οπότε ΑΛ = ΒΓ . 2

ΒΓΟ>ΒΕ Άρα ΑΛ = ΕΚ =-= - . Δηλαδή στο ορθογώνιο τρίγωνο ΒΕΚ, έχουμε : 2 2

ΒΕ Α

ΕΚ = - οπότε Βι =30°. 2

Από το ρόμβο ΒΓ ΔΕ έχουμε Β ι =Δι = 30° και επειδή Γ Δz = 90° έχουμε τελικά ότι:

IΔ2 = 60° I (2) Το τετράπλευρο ΑΓΔΖ είναι εγγράψιμο (διότι Α=Δ=90°) και η ΑΔ είναι διχοτόμος της γωνίας

Γ ΑΖ . Άρα το Δ είναι μέσο του τό ου ΓΖ, οπότε

(3) Από τις σχέσεις (2) και (3) συμπεραίνουμε ότι το τρίγωνο ΔΕΖ είναι ισόπλευρο. (!)) Προφανώς η ΑΕ είναι διχοτόμος της γωνίας Γ Az . Αρκεί να αποδείξουμε ότι η ΖΕ είναι

διχοτόμος της γωνίας ΓΖΑ . Εφόσον το τρίγωνο ΔΕΖ είναι ισόπλευρο, θα ισχύει ΕΖ = ΕΒ και επειδή Β2 = 15°, θα ισχύει:

z2 = 1so (4) Από το εγγράψιμο τετράπλευρο ΑΓΔΖ έχουμε ΑΖΓ =Δι= 30°, οπότε θα είναι Ζ1 = 15°.

!ΠΙ ' ! ' ' ,j � a /}�ιιJJ,ημα ..,., Α ν χ, y, z ε R * , να λυθεί το σύστημα: 3x2y + 2yz2 = 70xz

7y2z + 4zx 2 = 256xy

5z 2x + 6xy2 = 52yz .

Λi>ση Για xyz *Ο το σύστημα είναι ισοδύναμο με το 3XJl + 2ΥΖ = 70, 7 yz + 4zx = 256, Szx + 6XJl =52 ,

z χ χ Υ Υ z

το οποίο, αν θέσουμε

γίνεται

xy yz zx -=u,-=ν,-=w z χ Υ { 3u + 2ν = 70

7ν+4w=256 5w+6u=52

(1) } (2) . (3)

Με πρόσθεση κατά μέλη των τριών εξισώσεων λαμβάνουμε 9 (u+ν+w )=37 8<:=> u+ν+w=42 .

Λόγω της (4) η εξίσωση (2) γίνεται 7ν+4 ( 42 -u - ν )= 256 <:=> -4u +3ν= 8 8.

(4)

(5) Από τις (1 ) και ( 5) λαμβάνουμε u = 2, ν= 32 , οπότε από την ( 4) προκύπτει ότι w = 8 . Άρα έχουμε το

σύστημα xy = 2, yz = 32, zx = 8 (6) z χ Υ από το οποίο με πολλαπλασιασμό κατά μέλη των τριών εξισώσεων έχουμε

Από τις (6) και (7) λαμβάνουμε xyz = 2 · 8 · 32 . (7)

{38�2

: ξ ·88· ·:;} <:=> { ;: : �: } <:=> { ; : :: } ' 2z2 =2 · 8 ·32 z2 =256 z=±16

ΕΥΚΛΕΙΔΗΣ Β' 66 τ.2/9

Page 12: Ευκλειδης Β 66

--------- Μαθηματικοί Διαγωνισμοί- Μαθηματικές Ολυμπιάδες -------­

οπότε, λαμβάνοντας υπόψη την εξίσωση (7), με έλεγχο προκύπτουν συνολικά 4 τριάδες που είναι λύσεις του συστήματος: ( x, y ,z) =( 4, 8, 1 6 )ή ( -4, -8, 1 6 )ή ( 4, -8, - 1 6 )ή ( -4, 8, - 1 6).

Γ' τάξη Λυκείου Πρόβλημα 1 Έστω ισοσκελές τρίγωνο ΑΒΓ με ΑΒ = ΑΓ και Α = 30" . Στα σημεία Α και Γ θεωρούμε τις

εφαπτόμενες του περιγεγραμμένου κύκλου του τριγώνου ΑΒΓ που τέμνονται στο Δ. (α) Να αποδείξετε ότι τα τρίγωνα ΑΒΓ και ΑΓΔ είναι όμοια. (β) Να υπολογίσετε το εμβαδόν του τετραπλεύρου ΑΒΓΔ συναρτήσει της πλευράς ΒΓ = α του

τριγώνου ΑΒΓ.

,'\(ιση (α) Τα τρίγωνα ΑΒΓ και ΔΑΓ είναι ισοσκελή '(ΔΑ = ΔΓ, ως εφαπτόμενες από το Δ στον

περιγεγραμμένο κύκλο) και έχουν Γ =Γ Μ , ως εντός εναλλάξ. Άρα είναι όμοια. Παρατηρούμε ότι Β ΟΓ = 2Α = 60" , οπότε το τρίγωνο ΟΒΓ είναι ισόπλευρο και ισχύει ότι R=ΒΓ=α .

Έστω η ΑΟ τέμνει τη ΒΓ στο σημείο Μ. Επειδή είναι ΟΑ = ΟΒ και ΑΒ = ΑΓ η ΟΑ είναι η μεσοκάθετη της ΒΓ. Άρα είναι ΑΔ 11 ΒΓ και το τετράπλευρο ΑΒΓΔ είναι τραπέζιο .

Επιπλέον από το τρίγωνο ΑΜΓ έχουμε

Α Δ

ΑΓ' �[α+ α�)'+: �ΑΓ�α�2+J3

Επειδή τα ισοσκελή τρίγωνα ΑΒΓ και ΔΑΓ είναι όμοια (Γ = Γ Μ ) , θα έχουμε ΑΔ = ΑΓ <=:> ΑΔ = ΑΓ 2

<=:>AΔ=a (2+J3 ) . ΑΓ ΒΓ ΒΓ Άρα είναι

(ΑΒΓΔ) α+α( 2+-/3 ) α( 2+-/3 ) α2 (9 + 5-/3 )

J!lιH'iHλ]1!JIU 2 • ι; � �

2 2 4

(α) Να προσδιοριστούν οι παράμετροι λ, μ Ε � έτσι ώστε ο αριθμός 2 να είναι ρίζα των

εξισώσεων: λχ3 - (μ + 4) χ -2 = 0 και μχ2 -4χ - λ -2 = 0 . (β) Για τις τιμές των λ, μ που βρήκατε στο ερώτημα (α), να λύσετε την εξίσωση λχ3-(μ+4)χ-2

= 17

μχ2-4χ-λ-2 8

Λύση {8λ -2(μ+4) -2 = 0} {λ =2} Για να είναι το 2 κοινή ρίζα των δύο εξισώσεων πρέπει και αρκεί: <=> .

4μ-4·2- λ-2 = 0 μ = 3

Γ λ 2 3 δ δ ' ξ ' ' 2χ3 - 7 χ -2 1 7 ( !1) ια = και μ= η ε ομενη ε ισωση γινεται: 2 = - . 3χ - 4χ - 4 8

Όμως έχουμε τις παραγοντοποιήσεις 2χ3 -7 χ -2 = (χ -2 ) ( 2χ2 + 4χ + 1 ) 3χ2 - 4χ - 4 = (χ -2) ( 3χ +2) .

' ξ' ' δ ' ξ ' 2χ2 + 4χ + 1 1 7 οποτε η ε ισωση ει ναι ισο υναμη με την ε ισωση = - , 3χ +2 8

ΕΥΚΛΕΙΔΗΣ Β ' 66 τ.2/1 0

Page 13: Ευκλειδης Β 66

-------- Μαθηματικοί Διαγωνισμοί- Μαθηματικές Ολυμπιάδες --------

2 { 2 } ' 1 3 { 2 } 1 3 <:::>16χ -19χ -26=0 xE.IR- 2 -- <=>χ= 2ηχ= -- xE.IR - 2 -- <=>χ= --. ' ' 3 16' ' 3 16

Πρόβλημα 3 Αν για τη συνάρτηση : .IR � .IR ισχύει: ( ( ) - ( ) ) = ( ( ) ) - , για κάθε x,y Ε .IR, τότε να αποδείξετε ότι η συνάρτηση είναι περιττή.

ΛίJση Για χ = y = Ο από τη δοσμένη συναρτησιακή σχέση (1) έχουμε :

f (f(O) -f(O)) = f (f(O))-Ο � /(0) = f (f(O) ) (2) Από τη δοσμένη συναρτησιακή σχέση (1) θέτοντας όπου y το f(x) έχουμε:

f( f(x) -f (f(x))) = f (f(x) ) -f(x) (3)

Αν τώρα στη (3) θέσουμε χ= Ο έχουμε: .f( f(O) -f (f(O))) = f (f(O)) -f(O) και σε συνδυασμό με την (2) καταλήγουμε f(f(O) ) = f(O) =Ο. Θέτοντας στην (1) όπου χ= Ο , έχουμε : f(/(0)-f(y) ) = f(/(0)) -y και δεδομένου ότι f(J(O) )=f(O)=O,

καταλήγουμε στη σχέση f(-f(y ) ) = -y . (4) Θέτοντας στην (1) όπου y το χ έχουμε :

f(f(x) -f(x)) = f (f(x) ) -χ� f(O) = f (f(x) ) -χ<=> f (f(x) ) =χ (5) Αντικαθιστώντας στην (4) όπου y το /(χ) , έχουμε : f( -f (f(x) ))=-f(x)

και σε συνδυασμό με την (5) , καταλήγουμε στη σχέση f( -χ)=-f(x) , για κάθε χ Ε .IR, δηλαδή η f είναι περιττή .

Πρόβλημα 4 Για κάθε τρεις μη μηδενικούς πραγματικούς αριθμούς a, b και c, που είναι διαφορετικοί

, , , , , α+b · b+c - c+α -( ) ' ( ) ' ( ) '

μεταξυ τους ανα δυο, να αποδειξετε οτι: -- + -- + -- � 2 .

ΛίJση Α θ , α+b , λ β ' ν εσουμε χ=--, τοτε αμ ανουμε α-b

α-b b-c c-α

α χ+1 b χ -1 (1 )

(, 1 , b Ο) Ο , θ ' b+c c+α , λ β ' ειναι χ =F- , αφου =F- • μοιως, αν εσουμε y = --, z = -- , τοτε αμ ανουμε b-c c-α b- y+1 c y -1 (2)

c z+ 1 και -=-. (3) α z-1 Από τις (1 ), (2) και (3) με πολλαπλασιασμό κατά μέλη λαμβάνουμε

(x+1 ) (y+1 ) (z+1) αbc ( ) ( ) ( )

=-=1=>xy+yz+zx=-1. χ -1 y -1 z -1 bca

Όμως έχουμε Ο � (χ + y + z γ = χ2 + / + z2 + 2 ( xy + yz + zx) = χ2 + / + z2 -2

=> χ2 + y2 + z2 -2 � Ο => χ2 + / + z2 � 2.

ΕΥΚΛΕΙΔΗΣ Β ' 66 τ.2/1 1

Page 14: Ευκλειδης Β 66

Η Homo Mathematicus είναι μια στήλη στο περιοδικό μας, με σκοπό την ανταλλαγή απόψεων και την ανάπτυξη προβληματισμού πάνω στα εξής θέματα: 1 ) τι είναι τα Μαθηματικά, 2) Πρέπει ή όχι να διδάσκονται, 3 ) Ποιοι είναι οι κλάδοι των Μαθηματικών και ποιο το αντικείμενο του καθενός, 4) Ποιες είναι οι εφαρμογές τους, 5 ) Ποιες επι­στήμες ή κλάδοι επιστημών απαιτούν καλή γνώση των Μαθηματικών για να μπορέσει κάποιος να τους σπουδάσει.

Για τους συνεργάτες της στήλης: παράκληση! τα κείμενα της στήλης αυτής, ως προς το περιεχόμενό τους και ως προς το επίπεδό τους, θα πρέπει να είναι συμβιβαστά με τα ενδιαφέροντα και το επίπεδο κατανόησης από μέρους των παιδιών.

επ ιμέλεια : Καρκάνης Βασίλης, Κερασαρίδης Γ ιάννης

Ι . "τι είναι τα Μαθηματικά;" • « . . . Η μαθηματική γνώση δεν είναι απλά μια

"παρέλαση συντακτικών μεταβολών", μια ομάδα "δομικών μετασχηματισμών" ή ακόμα και "απλού τύπου γραμμικές συνδέσεις" . Όσο περισσότερο βυθιζόμαστε εθνογραφικά στους μαθηματικούς κό­σμους, τόσο περισσότερο εντυπωσιαζόμαστε από τη γενικότητα της κοινωνιολογικής απαίτησης. Οι μαθηματικοί τύποι ή τα μαθηματικά αντικείμενα όλο και περισσότερο τείνουν να εξετάζονται σαν λογικότητες, συλλογικοί μετασχηματισμοί και πα­γκοσμίου ενδιαφέροντος απόψεις. Τα θεμέλια των μαθηματικών δεν εντοπίζονται στη λογική ή σε συστήματα από αξιώματα αλλά στην κοινωνική ζωή . Οι μαθηματικοί τύποι και τα μαθηματικά α­ντικείμενα ενσαρκώνουν τους μαθηματικούς κόσμους. Περιλαμβάνουν την κοινωνική ιστο­ρία της κατασκευής τους. Παράγονται μέσα και από τους μαθηματικούς κόσμους. Τελικά, είναι οι κόσμοι αυτοί και όχι κάποιοι μεμονω­μένοι μαθηματικοί αυτοί που δημιουργούν τα μαθηματικά . . . » {Πηγή: Εργασία φοιτητών για το μάθημα "Επιστήμη­Τεχνολογία-Κοινωνία", Μάης 1999 (Μετάφραση ή και σχολια­σμοί ή και γενικότερη παρουσίαση) Αμπαρτζίδης Γ., Ζαχαρά­κης Δ., Μπούζας Δ., Ταυρής Ν., Τσούκας B.[SAL RΕSΠνΟ,

11. 'Ά υτδ το ξι:ρατε;"

'Ή ΚΟΙΝΩΝΙΚΉ ΥΠΟΣτΑΣΗ ΤΩΝ ΜΑΘΗΜΑΠΚΩΝ",

Κεφ. 13, Εισαγωγή]- http://ypeήon.math.upatras.gr/courses/sts} • Ο αείμνηστος παιδαγωγός Γ.Κ. Μωραϊτης

( 1 925-2002), στον ανέκδοτο δεύτερο τόμο της ερ­γασίας του με τίτλο «Το Πολυτεχνικό Σχολείο Βα­σικής-Γενικής Εκπαίδευσης» γράφει για τα Μαθη­ματικά, μεταξύ άλλων: «ε) Η γνώση της γί.ώσσας και της μαθηματικής σκέψης: Αυτά τα δύο στοιχεία που λειτουργούν σαν τον πρώτο άξονα γνωστικής λειτουργίας του σχολείου, έχουν διπλή αξία, μεθοδο­λογική και φιλοσοφική. Μεθοδολογική, γιατί είναι το όργανο της επικοινωνίας και προσέγγισης της πραγ­ματικότητας, και παράλληλα είναι το σύστημα με το οποίο ταξινομείται ο κόσμος γύρω μας σε ένα οργα­νικό σύνολο και απεικονίζεται ο γύρω και μέσα μας κόσμος. Και δεν πρέπει να ξεχνάμε πως αυτή ακρι­βώς η λειτουργία είναι εκείνη που δίνει ιδιαίτερη διάσταση στον ψυχισμό του ανθρώπου. Το δίδυμο γλώσσα και μαθηματική σκέψη (η δεύτερη είναι προέκταση της πρώτης) πρέπει και σε συσχετισμό

μεταξύ τους να λειτουργούν και σε αναφορά με τους υπόλοιπους άξονες γνωστικής λειτουργίας»

[πηγή: περιοδικό «ΘΕΜΑΤΑ ΠΑΙΔΕΙΑΣ», τ. 29 (2007)]

Γ ιάννης Κερασαρ ίδης

Τι είναι η "Σάλπιγγα του Γαβριήλ" ή "τρομπέτα του Toιτicelli " ; [η απάντηση στο τέλος της στήλης]

11 I. "οι συvεργάτε� της στιίJ.ιις γράφου ιι-ερωτοι5v"

Π ρώτο Θέμα : «Η ανακάλυψη των αρχείων Καραθεοδωρή» Σ. Λ ιπορδέζη - Ν. ΛυγεροίJ ( Κομοτην ή - Λυών )

Π ρολεγόμενα. Ο Σάκης Λιπορδέζης είναι ιδρυτής την Κομοτηνή κέντρο σπουδών πάνω στα έργα του και πρόεδρος του «Συνδέσμου Φίλων Κ. Καραθεο- διάσημου Έλληνα Θρακιώτη μαθηματικού δωρή» (και για πολλά χρόνια πρόεδρος του Παραρ- Κων/νου Καραθεοδωρή . Το άρθρο που ακολουθεί τήματος της ΕΜΕ Κομοτηνής) . Ο Νίκος Λυγερός είναι κάποιες (ελάχιστες) πινελιές στο πορτραίτο σας είναι γνωστός από το προηγούμενο τεύχος (πα- του μεγάλου Έλληνα μαθηματικού, ο οποίος είναι ρουσίαση στη στήλη μας) . Αυτοί οι δύο άνθρωποι, πασίγνωστος στην αλλοδαπή και, σχεδόν, άγνω­με την αρωγή του Υπουργού ΥΠΕΠΘ Ευριπίδη στος στην πατρίδα του . Στυλιανίδη, πέτυχαν το ακατόρθωτο : να κάνουν

ΕΥΚΛΕΙΔΗΣ Β ' 66 τ.2/12

Page 15: Ευκλειδης Β 66

------------------------------ H'Af' Af�TH&AfAT/CVf -----------------------------

i\16ναzο, ΙοίJνιος 2007 Στο Μόναχο είχαμε πάει και το Νοέμβρη του

2006. Την πρώτη μέρα κάναμε ένα μεγαλόπρεπο μνημόσυνο στον τάφο του Κ. Καραθεοδωρή όπως του άξιζε . Με παπά και ψαλτική χορωδία. Με κόλ­λυβα και καταθέσεις στεφάνων. Με παρουσία υψη­λών ακαδημαϊκών προσώπων όπως ο Roland Burl­ish, ο Frits, και ο κοσμήτωρ του πανεπιστημίου οι οποίοι αναφέρθηκαν με τα καλύτερα λόγια για τον Κ. Καραθεοδωρή .

Την επομένη , νωρίς το πρωί, τα βήματά μας οδή­γησαν στα βιβλιοπωλεία παλιών βιβλίων και τα παλαιοπωλεία.

Μια παλιά γκραβούρα του 1 900 με την ακαδημία των γραμμάτων και τεχνών ήταν η πρώτη μας αγο­ρά γιατί τότε ζούσε εκεί ο Καραθεοδωρή και επειδή βραβεύτηκε σ ' αυτήν. Μετά από αρκετό ψάξιμο βρίσκουμε και ένα βιβλίο του φίλου του Sommerfeld περί θερμοδυναμικής με αρκετές αναφορές στον Καραθεοδωρή .

Την επομένη και τελευταία μέρα ξεκινάμε για την αναζήτηση αρχείων. Στην πανεπιστημιακή βι­βλιοθήκη βρίσκουμε αρκετά από τα βιβλία του και την διατριβή του την οποία και φωτοτυπούμε. Με ικανοποίηση και νέες δυνάμεις συνεχίζουμε την αναζήτηση στα αρχεία της πόλης.

Ένα κτίριο επιβλητικό εξαιρετικής αρχιτεκτονι­κής στην είσοδο του οποίου δεσπόζουν τέσσερα μεγάλα μαρμάρινα αγάλματα. Του Αριστοτέλη , του Θουκυδίδη , του Ομήρου και του Ηροδότου, καθι­σμένους σε θρόνους.

Εικόνα απείρου κάλλους την οποία για αρκετή ώρα κάναμε «σάρωση». Όσο σκεφτόμασταν ότι μέσα μπορεί να είναι οι πνευματικοί θησαυροί ενός άλλου δικού μας η καρδιά μας ανέβαζε τους ρυθ­μούς της. Η πρόσβαση τελικά βρήκε αδιέξοδο . Χρειαζόμασταν ειδική άδεια.

Έτσι μέσω του Υπουργείου Εξωτερικών και σε συνεργασία με το προξενείο της Ελλάδας στο Μό­ναχο πετυχαίνουμε πρόσβαση στα αρχεία τα οποία μας περίμεναν στο κτίριο των αρχείων της πόλης τον Ιούνιο του 2007.

Ο διευθυντής των αρχείων κ. Bemd Gδrmer μας περίμενε στο καθορισμένο ραντεβού . Αφού συ­μπληρώνουμε τα απαραίτητα έγγραφα οδηγούμα­στε σε ένα γραφείο 3χ2.

Δεν αργεί να έρθει το πρώτο κουτί σαν τις κά­σες που βάζουμε μήλα. Η έκπληξή μας προφανής όπως η αδημονία μας να αρχίσουμε το ψάξιμο . Μέσα υπήρχαν τέσσερα πακέτα δεμένα με σπάγκο χαρταετού . Περιοδικά, γράμματα, φάκελοι, κάρτες, τετράδια, μπλοκ, όλα γραμμένα δια χειρός Κ. Κα­ραθεοδωρή . Παραλήπτες επιστολών και αποστο­λείς σπουδαίοι μαθηματικοί του εικοστού αιώνα, Cartan, Courant, Schmidt, Hilbert, Landau.

Δέος, δέος, δέος. Ψαχουλεύουμε χαρτιά διπλω­μένα μέσα σε σημαντικά περιοδικά που ίσως πρώτη φορά ανοίγονταν. Πρώτη φορά κοιτούσαμε με τόσο θαυμαστό την μουτζούρα των δακτύλων μας από το ξεφύλλισμα. Παρά την αφόρητη ζέστη στον κλειστό χώρο δεν καταλάβαμε πότε πέρασαν τέσ­σερις ώρες και έπρεπε να αποχωρήσουμε. Η επόμε­νη μέρα ήταν Παρασκευή . Η βιβλιοθήκη έκλεινε στις 1 2 .00. Ευτυχώς μας δόθηκε ειδική παράταση μιάμισης ώρας.

Έτσι την επομένη ήμασταν από τους πρώτους μελετητές. Σημειωτέον ότι για κάθε τι που θα ζη­τούσαμε αντίγραφο έπρεπε να συμπληρωθεί η α­ντίστοιχη αίτηση . Ευτυχώς και σ ' αυτό την διευκό­λυνση των διευθυντών στο βλέμμα των οποίων δι­ακρίναμε την ικανοποίηση επειδή ασχολούμαστε με τον Κ. Καραθεοδωρή .

Τα χέρια μου τρέμουν όταν αντικρίζω επιστολή μαθηματικού περιεχομένου με ημερομηνία 24 Ια­νουαρίου 1950, δηλ. 9 μέρες πριν πεθάνει ! ! !

Η μία έκπληξη διαδέχεται την άλλη και να μια κριτική ανάλυση με διορθώσεις για το κατοπτρικό τηλεσκόπιο του Schmidt.

Σταχυολογώντας συγκλονιστικές στιγμές που μοιραστήκαμε με τον Δρ Ν. Λυγερό ανακαλύπτο­ντας χειρόγραφα αυτόγραφα του Κ. Καραθεοδωρή και όχι μόνο, καταγράφουμε τα εξής. • Επιστολή στον Finsler το 1 945 . • Επιστολή στον Jacobi Bemoulli για το ισοπεριμε­τρικό πρόβλημα του Lagrange . • Το λήμμα Schwarz. • Πέντε διατριβές περί μετασχηματισμού του Leg­endre . • Μαθήματα στο Μετσόβιο Πολυτεχνείο . • Καταγραφή από τον ίδιο τον Καραθεοδωρή όλου του έργου του και των εργασιών του (δύο χειρό­γραφα, το ένα με διορθώσεις και το άλλο καθα­ρό) .

• Κριτικές αναλύσεις για έργα μεγάλων μαθηματι­κών και φυσικών όπως του Courant, Hilbert, Sommerfeld κλπ.

• Σημειώσεις από τις διαλέξεις του στις 2-5 και 8 Νοεμβρίου 1 926 για τη θεωρία της σχετικότητας. • Περί των καμπυλών του στυλοβάτη του Παρθε­νώνα. • Άρθρο για τον Ε. Cartan στις 1 2 Απριλίου 1 929 στην εφημερίδα Deutche allgemeine Zeitung. • Εργασία με τίτλο : Η μεγάλη πυραμίδα του Χέο­πος. • Εργασία στα Ελληνικά με τίτλο περί Μαθηματι­κών που διδάσκονται στη μέση εκπαίδευση , το 1 924.

• Σύντομη αυτοβιογραφία. • Άρθρο του σε δελτίο της ΕΜΕ το 1 943 .

ΕΥΚΛΕΙΔΗΣ Β ' 66 τ.2/13

Page 16: Ευκλειδης Β 66

#�Af� Af�T#EArAT/CVf ----------------------------­

• Εργασία για το κατοπτρικό τηλεσκόπιο του • Ευχετήριες κάρτες προς σημαντικούς μαθηματι-Smίth. κούς γεμάτες σημειώματα επιστημονικά και με • Εργασία για την αναδιοργάνωση των πανεπιστη- επικολλημένο γραμματόσημο. μίων Αθηνών - Θεσσαλονίκης. • Ανακοίνωση Πρωσικής Ακαδημίας δια προέδρου • Το πρόβλημα του Tchebytcheν για τους χάρτες. της Oscar Peπon για την αναγγελία του θανάτου • Αλληλογραφία με τον Petric, τρεις μήνες πριν το του . θάνατό του. • Συλλυπητήρια ανακοίνωση Ακαδημίας Αθηνών • Εργασία με την οποία συμμετείχε στο συνέδριο προς τα παιδιά του . του 1 934. Α ΥΤ Α ΚΑΙ ΠΟΛΛΑ ΑΚΟΜΑ ΗΔΗ ΒΡΙΣΚΟ-• Δημοσίευση στο περιοδικό OSIRIS* το 1 937 άρ- ΝΤΑΙ ΣΤΟ ΜΟΥΣΕΙΟ ΚΑΡΑΘΕΟΔΩΡΗ (αντί­

θρου με θέμα «Οι αρχές της θεωρίας του λογι- γραφα προς το παρόν) . σμού των μεταβολών». *περιοδικό που ασχολείται με μελέτες στην ιστορία

• Αλληλογραφία με Rosenthal . και τη φιλοσοφία των επιστημών.

Δεύτερο Θέμα: «Χειραψίες» Τηλtμ. \'Ιπαλτσαβ ι{1ς, (Κ�:φαλλονιά)

Προλεγ{ιμενα. Ο Τηλέμαχος Μπαλτσαβιάς είναι ένας φίλος της στήλης μας και γνωστός στους αναγνώ­στες από προηγούμενο άρθρο του για τα «Γενέθλια» . Σ ' αυτό το άρθρο (θα δημοσιευθεί σε δύο συνέχει­ες) ασχολείται με τις «Χειραψίες»

Εκ πρώτης όψεως φαίνεται ότι η χειραψία ως τρόπος χαιρετισμού δεν παρουσιάζει κάποιο Μα­θηματικό ενδιαφέρον. Όπως όμως θα δείξουμε στο άρθρο αυτό, ο πιο διαδεδομένος τρόπος χαιρετι­σμού έχει να μας διδάξει αξιόλογες ιδέες και σκέ­ψεις. Το πρώτο, και ίσως μεγαλύτερο όφελος που μπορεί να αποκομίσει ένας μαθητής διαβάζοντας το παρόν άρθρο, είναι η κατανόηση εννοιών με χειρο­Παράδειγμα I"

Ας πούμε ότι μια παρέα αποτελείται από 1 0 άτο­μα που έχουν δώσει ραντεβού να βρεθούν, να τα πιουν και να το ευχαριστηθούν. Όλοι τα πηγαίνουν καλά με όλους τους υπόλοιπους και έτσι ο καθένας από αυτούς χαιρετάει δια χειραψίας όλους τους άλλους. Το ερώτημα είναι: «Πόσες χειραψίες έγι­ναν;» Φυσικά ο καθένας χαιρετάει τον οποιονδήποτε άλ­λο μόνο μια φορά και προφανώς ουδείς χαιρετάει τον εαυτό του .

Ο καθένας από τους 1 Ο της παρέας δίνει 9 χειρα­ψίες, άρα θα μπορούσε κάποιος εύκολα να ισχυρι­στεί ότι έγιναν 1 0 ·9 = 90 χειραψίες. Αυτό όμως εί­ναι λάθος. Γιατί;

Θυμηθείτε πως όταν ο Α δίνει χειραψία με τον Β, τότε και ο Β δίνει χειραψία με τον Α. Έτσι στον παραπάνω υπολογισμό, η κάθε χειραψία που έγινε υπολογίστηκε δυο φορές. Άρα η σωστή απάντηση είναι 1 0·9/2 = 90/2 = 45 χειραψίες.

Τώρα φίλοι μαθητές ας κάνουμε κάτι πολύ ωραίο από Μαθηματική άποψη, ας γενικεύσουμε το πα­ραπάνω συμπέρασμα. Αν αντί για 1 0 άτομα στην Παράδειγμα 2"

πιαστά παραδείγματα εκτός της καθιερωμένης «σχολικής ύλης».

Η πρώτη έννοια της οποίας η χειραψία αποτελεί παράδειγμα, είναι η συμμετρική διμελής σχέση, δη­λαδή αν ο Α δίνει χειραψία (σχετίζεται) με τον Β, τότε και ο Β δίνει χειραψία (σχετίζεται) με τον Α. Ας δούμε όμως τρία ενδιαφέροντα παραδείγματα Μαθηματικής σκέψης.

παρέα ήταν οποιοσδήποτε αριθμός, έστω v , ποια θα ήταν η απάντηση στο ίδιο πρόβλημα; Δεν είναι

πλέον δύσκολο να βρείτε πως έγιναν v ( V -1 )/ 2 χειραψίες. Βάλτε τώρα στη θέση του ν το πλήθος των ατόμων που θέλετε και προκύπτει αμέσως η απάντηση στο αντίστοιχο πρόβλημα. Μια και ήρθε στο προσκήνιο η γενίκευση , ας δούμε κάτι άλλο ωραίο στο ίδιο πρόβλημα, την ισοδύναμη γεωμε­τρική του διατύπωση .

Είναι η εξής: Δίνονται 1 Ο διακεκριμένα σημεία στο επίπεδο. Πόσα διαφορετικά ευθύγραμμα τμή­ματα ορίζονται με άκρα τα σημεία αυτά;

Η λύση είναι ολόιδια, γι ' αυτό μπορούμε να μι­λάμε για γεωμετρικό αντίστοιχο . Το παραπάνω γε­νικό αποτέλεσμα βλέπουμε ότι περιγράφεται από τη συνάρτηση f με τύπο f(x) =χ( χ - 1)/2 όταν αυτή περιοριστεί στους θετικούς ακέραιους. Ακο­λουθεί η γραφική παράσταση (σε Mathematica) αυ­τής της συνάρτησης για όλους τους πραγματικούς χ E[l,+oo). Όπως γνωρίζουμε, καμπύλη αυτή είναι τόξο παραβολής.

Συχνά οι μαθητές του Λυκείου αδυνατούν ι σουν, μια απόδειξη που γίνεται με την αποδει­να καταλάβουν, πόσο δε μάλλον να διατυπώ- κτική μέθοδο της «εις άτοπον απαγωγής». Αυ-

ΕΥΚΛΕΙΔΗΣ Β ' 66 τ.2/14

Page 17: Ευκλειδης Β 66

----------------------------- #�N� N�T#eNA.r/CVf ---------------------------­

τό γίνεται γιατί δεν ξέρουν τι ακριβώς πρέπει Ο ισχυρισμός είναι ο εξ1Ίς: να αρνηθούν και τι να δεχθούν κατά την απο- Σε μια συγκέντρωση 1 Ο ατόμων υπάρχουν δεικτική αυτή διαδικασία. τουλάχιστον δυο άτομα που αντάλλαξαν τον

Στο σημείο αυτό ας μας επιτραπεί και μια ίδιο αριθμό χειραψιών (προφανώς ο καθένας ιστορική νότα: η μέθοδος της «εις άτοπον απα- χαιρετάει τον οποιονδήποτε άλλο μόνο μια φο­γωγής» ήταν εξαιρετικά προσφιλής στους αρ- ρά και ουδείς χαιρετάει τον εαυτό του) . χαίους Έλληνες Μαθηματικούς με χαρακτηρι- Η απόδειξη είναι η εξής: στικότερο παράδειγμα την απόδειξη που έδωσε Αρνούμαστε το συμπέρασμα, δηλαδή δεχό­ο Ευκλείδης για την απειρία των πρώτων α- μαστε ότι και οι 1 Ο αντάλλαξαν διαφορετικό ριθμών, η οποία μέχρι σήμερα έχει μείνει ου- αριθμό χειραψιών. Η μόνη περίπτωση για να σιαστικά aπαράλλαχτη. Επίσης ο G. Η. Hardy, συμβεί αυτό είναι κάποιος (έστω ο 1 ος ) να έ­ένας από τους μεγάλους Μαθηματικούς του ει- δωσε Ο χειραψίες, κάποιος άλλος (έστω ο 2°ς ) κοστού αιώνα, έγραψε στην «Απολογία ενός να έδωσε 1 χειραψία, κάποιος άλλος (έστω ο Μαθηματικού» ότι η «εις άτοπον απαγωγή» εί- 3°ς ) να έδωσε 2 χειραψίες, κάποιος άλλος (έ­ναι μια «θυσία» που κάνει ένας Μαθηματικός, στω ο 4°ς ) να έδωσε 3 χειραψίες κλπ, μέχρι μακράν ωραιότερη από οποιαδήποτε «θυσία» που ο 10°ς να έδωσε 9 χειραψίες. Δηλαδή ο (gambit) που μπορεί να κάνει ένας οποιοσδή- 10°ς της παρέας χαιρέτησε όλους τους άλλους. ποτε σκακιστής, διότι ενώ ο σκακιστής προ- Αυτό όμως δεν γίνεται - εδώ έγκειται το άτοπο σφέρει ένα πιόνι ή κάποιο άλλο κομμάτι ο Μα- - αφού ο πρώτος έδωσε Ο χειραψίες (άρα δεν θηματικός προσφέρει ολόκληρο το παιχνίδι. χαιρέτησε κανένα). Άρα τι απομένει; Να δε-

Στη σύγχρονη διεθνή βιβλιογραφία η «εις χτούμε ότι υπάρχουν τουλάχιστον δυο άτομα άτοπον απαγωγή» αναφέρεται με την Λατινική που αντάλλαξαν τον ίδιο αριθμό χειραψιών. της ονομασία «Reductio ad Absurdum». Φυσικά η ιδέα της απόδειξης δεν αλλάζει αν

Το παρακάτω παράδειγμα αποτελεί χαρα- αντί για 1 Ο παρευρισκόμενους έχουμε 90 ή 952 κτηριστική περίπτωση απόδειξης με απαγωγή ή οποιονδήποτε άλλο αριθμό σκεφθείτε. σε άτοπο. [η συνέχεια στο επόμενο]

llιι. "Αυτt) το ξι3ρατι:;"!η απάντηση! Η Σάλπιγγα του Γαβριήλ (που επίσης λέγεται στη θρησκευτική παράδοση που θέλει τον Αρχάγ­

και τρομπέτα του Torricelli) είναι ένα σχήμα που γελο Γαβριήλ να φυσά τη σάλπιγγα που αναγγέλλει επινοήθηκε από τον Evangelista Toπicelli ( 1608- την Ημέρα της Κρίσης, συνδέοντας έτσι το άπειρο 1647), και το οποίο έχει "άπειρο" εμβαδά, αλλά με το θείο. πεπερασμένο όγκο . Η ονομασία του αναφέρεται

ο :z .. Β

Σχήμα μέρους της "Σάλπιγγας του Γαβριήλ"

Η Σάλπιγγα του Γαβριήλ σχηματίζεται αν πάρουμε τη γραφική παράσταση του ,y= l /x στο διάστημα χ2:1(αποφεύγοντας έτσι την aπροσδιοριστία στο σημείο χ=Ο) και περιστρέφοντάς την σε τρεις δια­στάσεις γύρω από τον άξονα των χ. Η ανακάλυψη έγινε χρησιμοποιώντας την αρχή του Καβαλιέρι και πριν την ανάπτυξη της Μαθηματικής Ανάλυσης,

όμως σήμερα η μαθηματική ανάλυση μπορεί να χρησιμοποιηθεί για να υπολογιστεί ο όγκος και το εμβαδά της Σάλπιγγας στο διάστημα από χ = 1 μέ­χρι χ = a, όπου a > Ι. Μπορείτε να μας πείτε πόση μπογιά χρειάζεται για να βάψουμε την επιφάνεια της σάλπιγγας; [πηγή: από άρθρο της αγγλικής Βικιπαίδειας]

[η συνέχεια στο επόμενο]

ΕΥΚΛΕΙΔΗΣ Β ' 66 τ.2/15

Page 18: Ευκλειδης Β 66

........... ,. q• �� ��Ιι �•ιι Αιι••Ι•ιι

Άλγεβρα Εξισώσεις 2ου βαθμού. Σύνολα.

Γενικά περί συναρτήσεων. Η συνάρτηση f(χ)=αχ+β του Βασίλη Καρκάνη

Το άρθρο που ακολουθεί, ασχολείται, με το τμήμα της Άλγεβρας, όπφς αυτό αναφέρεται στον τίτλο. Ακολουθώ­ντας τις οδηγίες για τη διδακτέα ύλη και τη διδασκαλία των Μαθηματικών του Λυκείου όπως αυτές έχουν διατυπωθεί, από το Παιδαγωγικό Ινστιτούτο επικεντρώνει τις απαιτήσεις του καθαρά στο επίπεδο αυτό. Για την κάθε παράγραφο αναφέρονται οι ερωτήσεις θεωρίας που θεωρεί ότι πρέπει να αποτελούν βασική γνώση για το μαθητή. Στη συνέχεια με ερωτήσεις του τύπου σωστό -λάθος, με ερωτήσεις πολλαπλής επιλογής καθώς και με υποδειγματικά λυμένες ασκήσεις καλύπτει (κατά το δυνατόν) σφαιρικά την αντίστοιχη ύλη. \. Ε" ' 2"" β θ ' . . ..;ισωσtις α μου

Ερωτήσεις θΕωρίας 1 . Δίνεται η εξίσωση : αχ2+βχ+γ=Ο ( 1) α, β,

γεR και α:;tΟ. Να γραφεί η διακρίνουσα της εξίσωσης ( 1 ) .

2. Να γραφούν οι σχέσεις που συνδέουν το πρόσημο της διακρίνουσας της εξίσωσης ( 1 ) και το πλήθος των ριζών της.

3. Να γραφεί ο τύπος λύσης της εξίσωσης ( 1 ) (στην περίπτωση που έχουμε λύση).

4. Αν Χι, χ2 είναι οι ρίζες της εξίσωσης ( 1 ) να γράφουν οι τύποι για το άθροισμα και το γινόμενο των ριζών.

5. Αν γνωρίζουμε τις ρίζες μιας εξίσωσης 2ου βαθμού, μπορούμε να βρούμε την εξίσωση αυτή;

6. Α ν η εξίσωση ( 1 ) έχει δυο λύσεις πώς μπορούμε να τις βρούμε χωρίς να τη λύ-

αχ2+βχ+γ=Ο έχει δυο ρίζες ετερόσημες, τότε γ<Ο. Σ - Λ

13. Για κάθε κεR, λεR * η εξίσωση χ2-κχ-λ2=0 έχει δυο ρίζες ετερόσημες.

Σ- Λ Ερωτήσης πολλαπλής επιλογής 14. Αν η εξίσωση χ2-6χ+κ=Ο έχει για διπλή

ρίζα το 3 τότε ο κ ισούται με : α. Ο β.2 γ. 9 δ. 6

15. Για κάθε κεR που η εξίσωση χ2-4χ-"-κ=Ο έχει δυο ρίζες άνισες θα ισχύει : α. κ<4 β. κ>4 γ. κ>Ο δ. κ<Ο

16. Για κάθε κεR που οι ρίζες της εξίσωσης χ2+κχ+4=0 είναι θετικές θα ισχύει: α. κ>Ο β. κ<-4 γ. κ<Ο δ. κ>4

17. Αν οι ρίζες της εξίσωσης 3χ2+(5-κ)χ-9=0 είναι αντίθετες τότε ο κ ισούται με : α. 3 β. -5 γ.Ο δ. 5

18. Αν κ+λ=3 και κ·λ= - 1 0 τότε οι αριθμοί κ, σουμε; λ είναι ρίζες της εξίσωσης:

7. Πώς μπορούμε να λύσουμε εξισώσεις της α. χ2+3χ+ 1 0=0 β. -χ2-3χ- 1 0=0 μορφής αχ2+β I χ I +γ=Ο με α:;tΟ και γ. χ2-3χ-1 0=0 δ. χ2+3χ- 1 0=0 αχ2ν+βχν+γ=Ο με α:;tΟ και νεΝ*; 19. Αν οι αριθμοί χι και χ ι 2 είναι ρίζες της ε-

Ερωτήσεις του τύπου «σωστό -λάθος». ξίσωσης χ2-6χ-27=0 τότε ο χ ι ισούται με : 8. Για κάθε α, β, γ εR με αγ<Ο η εξίσωση : α.-9 β.9 γ.3 δ.-3

αχ2+βχ+γ=Ο έχει δυο ρίζες άνισες. Σ - Λ 20. Για κάθε κ>Ο, η εξίσωση χ4+ 3χ2+κ=Ο έχει: 9. Αν η εξίσωση αχ2+βχ+γ=Ο με α:;tΟ έχει α. τέσσερις λύσεις β. δύο λύσεις

δυο ρίζες αντίθετες τότε β=Ο. Σ - Λ γ. καμιά λύση δ. δεν μπορούμε 10. Για κάθε α, β, γ εR αν χι, Xz είναι οι ρίζες να απαντήσουμε

της εξίσωσης αχ2+βχ+γ=Ο τότε οι -χι , -χ2 Λυμένες ασκήσεις

είναι οι ρίζες της εξίσωσης αχ2 -βχ+γ=Ο. Άσκηση 111 Σ - Λ Να βρεθούν οι τιμές του πραγματικού αριθ-

1 1 . Υπάρχουν πραγματικοί αριθμοί κ, λ ώστε μού λ ώστε η εξίσωση χ2 -2χ+λ=Ο να έχει: α) να ισχύει: κ+λ= 1 και κ· λ =2. Σ - Λ δυο πραγματικές ρίζες άνισες β) μια διπλή

12. Για κάθε α, β, γ εR με α:;t:Ο, αν η εξίσωση ρίζα γ) καμιά ρίζα (πραγματική)

ΕΥΚΛΕΙΔΗΣ Β' 66 τ.2/16

Page 19: Ευκλειδης Β 66

Μαθηματικά για την Α · Λυκείου

.\{Jση Η εξίσωση έχει α=1:;t:Ο, οπότε η διακρίνουσά της είναι: Δ=β2--4αγ = . . . =4( 1-λ) οπότε : α) Πρέπει και αρκεί: Δ>Ο, δηλαδή λ< 1 β) Πρέπει και αρκεί: Δ=Ο, δηλαδή λ= 1 γ) Πρέπει και αρκεί: Δ<Ο, δηλαδή λ> 1 Ά σ κηση 2 ' 1 Να βρεθεί ο πραγματικός αριθμός λ ώστε η εξίσωση (λ+3)χ2-4λχ-(5λ-6)=0 να έχει μια διπλή ρίζα. Λύση Για να έχει η εξίσωση διπλή ρίζα πρέπει και

αρκεί: α :;t: 0} (1) . Δ=Ο λ + 3 :;t: Ο } (1) � (-4λ/ +4 (λ+3) (5λ -6)=0

� λ :;t: -3 }

� 16λ 2 + 20λ 2 + 36λ - 72 = ο λ :;t: -3 }

� � 36λ2 + 36λ - 72 =ο λ:;t:-3 } λ:;t:-3 }

� � �λ=1 ή λ=-2 λ2 +λ-2=0 λ=1 ή λ=-2

ί\ σκηση J 'l Δίνεται η εξίσωση : λχ2+5χ+10=0 α) Για ποια τιμή του λ έχει μια ρίζα απλή; β) Για ποια τιμή του λ έχει μια ρίζα διπλή ; Να βρεθεί η διπλή ρίζα. . \ ί1ση

α) Πρέπει και αρκεί (α = 0) , δηλαδή λ=Ο. Τό-. β :;t: Ο τε η εξίσωση γράφεται 5χ+ 1 0=0, που είναι 1 ου βαθμού, οπότε έχει μια ρίζα απλή .

β) Πρέπει και αρκεί : (α * 0 ) . Δ=Ο

Έχουμε: Δ=β2 --4αγ=52 --4· λ· 1 0= . . . =5( 5-8λ) .

Οπότε : (α* Ο ) � [ λ* �)� λ=� Δ = Ο λ=- 8

8

Η διπλή ρίζα θα είναι η χ0 = _ _Ι_= _2_ = 2α 2λ

5 =--5 =-4 . 2 · -

8 ?\σκηση 4 ' 1 Να βρεθεί ο πραγματικός αριθμός λ ώστε η εξίσωση χ2 -2(λ-5)χ+λ2 -4=0 να έχει:

α) δυο ρίζες άνισες β) δυο ρίζες ίσες γ) καμιά ρίζα δ) δυο ρίζες θετικές

ε) δυο ρίζες αρνητικές στ) δυο ρίζες ετερόσημες ζ) δυο ρίζες αντίθετες η) δυο ρίζες aντίστροφες

.\{Jση Για την εξίσωση που δόθηκε έχουμε α = 1 :;t: Ο , οπότε η διακρίνουσα είναι: Δ=β2--4αγ = [-2(λ-5)]2 - 4· 1 · (λ2--4)= = � - 40λ+100 -� + 1 6 = - 4(10λ-29). α) Για να έχει η εξίσωση δυο ρίζες άνισες πρέ­πει και αρκεί Δ>Ο. Αλλά: Δ>Ο�

29 --4( 1 0λ-29)>0� � 10λ-29<0� λ < -10

β) Για να έχει η εξίσωση δυο ρίζες ίσες πρέπει

και αρκεί: Δ=Ο, δηλαδή λ = 29 1 0

γ) Για να μην έχει η εξίσωση ρίζες πρέπει και

αρκεί Δ<Ο, δηλαδή λ > 29 . 1 0

δ) Όπως είπαμε στο (α) ερώτημα η εξίσωση θα . δ 'ζ . . . λ 29 εχει υο ρι ες ανισες, οταν και μονο : < -

1 0 Επίσης αν Χ ι , χ2 είναι οι ρίζες τότε:

Χ1 + χ2 = -� = 2 (λ - 5 ) και

α

Υ λz 5 . Χ ι ·Χ2 =-= - , οποτε: α

Η εξίσωση έχει δυο ρίζες θετικές �

Δ > Ο β � -- > 0 α

l. > o α

λ < 29 10

� 2 (λ - 5 ) > 0

λ < 29 1 0

� λ > 5

ε) Η εξίσωση έχει δύο ρίζες αρνητικές �:

Δ > Ο λ < 29 λ < 29

β 1 0 1 0

� -- < 0 � 2 (λ - 5 ) < 0 � λ < 5 α

l_ > ο λ2 -4 > 0 lλ l > 2

α

αδύνατο

λ < 29 1 0

� λ < 5 � λ < -2 ή 2 < λ < 29 . 1 0

λ < -2 ή λ > 2

στ) Πρέπει και αρκεί: αγ < Ο . Όμως αγ < Ο � λ2 - 4 < 0 � -2 < λ < 2 .

ζ) Πρέπει και αρκεί:

ΕΥΚΛΕΙΔΗΣ Β' 66 τ.2/17

Page 20: Ευκλειδης Β 66

Μαθη ματικά για την Α ' Λυκείου

Δ > Ο} λ < 29 } λ < 29 } = 0

� 1 0 , δηλαδή 1 0 αδύνατο β β = Ο λ = 5

η) Πρέπει και αρκεί:

Δ > Ο} λ < 29} λ 29 } γ - � 10 �

<10 � λ = J5 ή λ = -15.

α - 1 1 = 1 λ2 -4 = 1 α :ι σκηση 5' 1 Δίνεται η εξίσωση : χ2+χ+λ-1=0.Να βρείτε για ποια τιμή του λ έχει άνισες ρίζες: χ ι , χ2 τέτοιες ώστε Χ ι · χ2+3(χι+χ2)+5=0 ( 1 ) . \iHίl] Για να ισχύει η ( 1 ) πρέπει και αρκεί:

α :;t: Ο Δ > Ο ( 2 ) . � + 3 ( -�) + 5 = 0

( 2) � �:�λ > ο } � λ = - 1 λ - 1 + 3 (- 1 ) + 5 = 0

Λ. σκηση 6' 1 Δίνεται η εξίσωση : χ2-λχ-λ2-5=0. α) Να δείξετε ότι για κάθε λ ε JR έχει δύο ρίζες χι , χ2 άνισες β) Να βρεθεί ο λ έτσι, ώστε να ισχύει η σχέ­ση : (χι - 2)(χ2 - 2) = -4. . \ {Jση α) Για την εξίσωση έχουμε: α= 1 :;e0 και Δ=β2--4αγ=(-λ)2--4(-λ2-5)= =5λ2+20>0, οπότε έχει 2 ρίζες χ ι , χ2 άνισες.

β)Είναι: χ 1 + χ 2 = -� = λ και α

γ λ2 5 ' χ , · χ 2 = - = - - οποτε : α

(Χ ι-2)(χz-2)= -4�Χ ιΧz-2(χ ι +Χ2)+8=0� <::::>-λ2-5-2λ+8=0�λ2+2λ-3=0�λ= -3 ή λ= 1 :\ σ Η<ηση 7' � Δίνεται η εξίσωση : χ2-2χ+λ=Ο με λ ε R. Να βρεθεί ο λ ώστε για τις ρίζες της εξίσωσης χι, χ2 να ισχύει:

3 3 9 ( ) Χι · Χ2 + Χι · Χ2 = -- - 2χι · Χ2 Χι + Χ2 · 2

. \ (Jση Αφού α= 1:;e0, θα έχουμε:

Δ>Ο�4-4λ>Ο� λ< 1 ( 1 ) . Με λ< 1 λοιπόν, έχουμε:

β χ , + χ2 = -- = 2 α

γ και χ 1 · χ 2 = - = λ . α

'Ετσι: χ� · Χ2 + χ , χ� = _2_ _ 2χ , · Χ2 (χ , + Χ2 ) � 2

� Χ ι · Χ 2 ( Χ � + χ ; ) = -� - 2χ , . Χ 2 ( χ , + χ 2 ) �

� χ , . χ 2 [ ( χ , + χ 2 )2 - 2χ , . χ 2 J =

9 = -2 - 2χ , · χ 2 ( χ , + χ 2 ) �

� λ (22 - 2λ) = -� - 2λ · 2 �

� 4λ - 2λ2 = -2. - 4λ � 2

� 2λ2 - 8λ - 2. = ο � 2

� λ2 - 4λ - 2. = ο � λ = 2. ή 4 2

λ = _ _!_ � λ = _ _!_ λόγω της ( 1 ) . 2 2 Άσκηση 8 '1 α) Αν χι , χ2 είναι οι ρίζες της εξίσωσης: χ2+βχ+γ=Ο, τότε να δειχθεί ότι: (χι- χ2)

2=Δ όπου Δ η διακρίνουσα της εξί­σωσης. β) Αν χι , χ2 είναι οι ρίζες της εξίσωσης: χ2-3χ-2=0 να υπολογιστεί η τιμή της παρά­στασης I Χι- x2 l · λίJση

α) Για την εξίσωση είναι α= 1 :;e0 και Δ=β2--4αγ= β2--4γ .

Οπότε : (χ ι-χ2)2=χ ι 2-2χ ι χ2+χ/= =χ 1 2+χ2 2 -2χ 1 χ2=(χ ι +χ2)2 -2χ ιχ2-2χ ι χ2=

=(χ ι+χ2)2--4χ ιχ2= ( -�)2 - 4 ·� = β2 - 4γ = Δ.

β) Για την εξίσωση που δόθηκε λόγω του (α) ερω­τήματος θα είναι: (χ ι- χ2)2=Δ. Όμως: Δ=β2--4αγ=(-3)2--4· 1 · (-2)= 1 7 οπότε: (χ ι- χ2)2= 1 7� I χ ι- χ2 Ι = Jl7

Ά σκηση 9'1 Να λυθεί η εξίσωση : (χ-2)2+5 1 χ-2 1 -14=0 Λ.1Jση Είναι (χ-2)2= I χ-2 1 2 , οπότε θέτοντας I χ-2 I =ψ ( 1 ) η επιλύουσα της εξίσωσης είναι η : ψ2+5ψ-1 4=0 (2) . ( 2 ) � ψ = -7 ή ψ = 2 , οπότε:

( l ) � l x - 21 = -7 ή l x - 2 1 = 2 � l x - 21 = 2 � � χ - 2 = 2 ή χ - 2 = -2 � χ = 4 ή χ = 0 . Άσκηση t () ' l Να λυθεί η εξίσωση : χ6-19χ3-216=0

ΕΥΚΛΕΙΔΗΣ Β ' 66 τ.2/1 8

Page 21: Ευκλειδης Β 66

Μαθη ματικά για την Α · Λυκείου

Λύση Θέτουμε χ3=ψ ( 1 ) , οπότε η επιλύουσα της εξί­

σωσης είναι η : ψ2 - 1 9ψ - 2 1 6 = Ο (2). (2 ) <=> ψ = -8 ή ψ = 27 , οπότε:

( 1 ) <=> χ3 = -8 ή χ3 = 27 <=> χ = -18 ή χ = ifi7 <=> χ = -2 ή χ = 3 .

Β. 2:ύνολα

Ερωτήσεις Θεωρίας I . Να δοθεί ο ορισμός του συνόλου 2. Να γράψετε με συμβολικό τρόπο τις φράσεις:

α) Το στοιχείο α ανήκει στο σύνολο Α β) Το στοιχείο β δεν ανήκει στο σύνολο Β

3 . Να εξηγηθούν οι παρακάτω τρόποι πα­ρουσίασης των στοιχείων ενός συνόλου Α. α) με αναγραφή των στοιχείων του β) με περιγραφή των στοιχείων του γ) με διάγραμμα Venn

4 . Να εκφράσετε με οποιοδήποτε τρόπο τα σύνολα: α) των φυσικών αριθμών β) των ακεραίων αριθμών γ) των ρητών αριθμών δ) των άρρητων αριθμών ε) των πραγματικών αριθμών

5 . Να ορίσετε το κενό σύνολο. Πώς το συμ­βολίζουμε;

6 . Να δώσετε τον ορισμό της ισότητας δυο συνόλων

7 . Να ορίσετε τις παρακάτω έννοιες : α) υποσύνολο συνόλου β) ένωση συνόλων γ) τομή συνόλων δ) διαφορά συνόλων ε) συμπλήρωμα συνόλου

Τις παραπάνω έννοιες να παραστήσετε με διά­γραμμα Venn Ερωτήσεις του τύπου «σωστό -λάθος» 8 . Ισχύει { 0 } = 0 . Σ - Λ 9 . Για τα σύνολα Α= { α , β, γ} και Β= {β, γ, α}

ισχύει Α ς Β . Σ - Λ 1 0. Δίνονται τα σύνολα Α= { 1 , 2, 3 , 4 } και

Β= {χ Ε Ν* I χ διαιρέτης του 4 } ισχύει Α ς Β . Σ - Λ

1 1 . Δίνονται τα σύνολα Α= {-2, -1 , 1 , 2 } και Β= {χ ΕΖ l -3 <χ:::2 } ισχύει Α ς Β . Σ - Λ

1 2. Τα υποσύνολα του συνόλου Α= {α, β } εί-ναι τα σύνολα {α} , { β } , {α, β } . Σ - Λ

Ερωτήσεις πολλαπλής επιλογής 1 3 . Για τα σύνολα: Α= {Ο, 1 ,3 } , Β= { 1 ,3 } .

Γ= {Ο } , Δ= {3 , 1 } ισχύει: α. Α ς Γ β. Γ ς Β γ. Δ ς Α δ. Α ς Β

1 4 . Για το σύνολο : Α= {κ+κ2 } ισχύει: α. Α = 0 για κ=Ο β. Α = 0 για κ= -1 γ. Α = 0 για κ= 1 δ. Α = 0 για κάθε κ Ε IR 1 5. Δίνονται τα σύνολα: Α= {2007, κ2+2008}

και Β= {2007, κ , 2008 } για τα οποία ισχύει: α) At!:B για κάθε κEIR, β) Α=Β για κ=Ο γ) Α=Β για κ= 1 δ) Α=Β για κάθε κι!:Ο

1 6 . Το σύνολο A= {x EIR/x2+x+ 1=0 } με ανα­γραφή των στοιχείων του είναι το : α. Α = {0 } β. Α = 0 γ. Α = {- 1, 1 } δ. Α = {- 1 , 2 } .

Π αρατήρηση Θεωρούμε σκόπιμο για το μαθητή της Α' Λυκεί­

ου να αναφέρουμε μια σειρά από σχέσεις και πράξεις μεταξύ των συνόλων που η γνώση τους θα τον βοη­θήσει να αντεπεξέλθει καλύτερα στην ύλη των mθα­νοτήτων που θα διδαχθεί στην Γ Λυκείου. Αξίζει μά­λιστα να αναφέρουμε ότι η αλήθεια των προτάσεων που αναφέρονται εmβεβαιώνεται και με απλή παρα­τήρηση μέσω διαγραμμάτων Venn. Έτσι αν θεωρή­σουμε το σύνολο Ω ως βασικό σύνολο και τα σύνολα Α, Β, Γ υποσύνολα του Ω τότε ισχύουν τα παρακάτω: 1 . ΑnΩ=Α, 2. ΑυΩ=Ω, 3. Α ς Α 4. 5 . ΑυΑ=Α, 6. An0=0, 7. ΑυΑ'=Ω, 8. An0=0 9. AU0=A, 10. Ω'=0 1 1 . 0 '=Ω 12. AnB=BnA, 13. ΑυΒ=ΒυΑ, 14. (AnB) nΓ= An (BnΓ)=AnBnΓ, 1 5. Αυ(ΒυΓ)=(ΑυΒ) υ Γ= ΑυΒυΓ, 16.An (ΒυΓ)=(ΑnΒ) υ (ΑnΓ), 17. Αυ (ΒnΓ)=(ΑυΒ) n (ΑυΓ), 18. (ΑnΒ) '=Α 'υΒ ' , 19. (ΑυΒ) '=Α 'nΒ ' (οι ισότητες 1 8 και 1 9 αποτε­λούν τους γνωστούς νόμους του De Morgan) . 20. Αν Α ς Β τότε ισχύουν: i)AnB=A,

ii) ΑυΒ=Β, iii) Α-Β=0, iv) Β ' ς Α' 21. Α-Β= AnB ' 22. Α-Β= A-(AnB), 23. Α-Β=(ΑυΒ)-Β, 24. (A-B)nB=0 25. (Α-Β) υ Β= ΑυΒ, 26. Α ς (Α υ Β ) 27. (Α n Β ) ς Α 28. (Α-Β) υ (AnB)=A, 29. ΑυΒ=(Α-Β) υ (AnB) υ (Β-Α),

Λυμένες Ασκήσεις Άσκη ση Ι "

Έστω Α το σύνολο λύσεων της εξίσωσης: I χ-3 1 = l 2x-3 1 και Β το σύνολο των λύσεων της εξίσωσης: χ5

= -χ2• Να βρείτε τα σύνολα ΑυΒ και ΑnΒ Λύση Είναι: I χ-3 1 = l 2x-3 1 <=> χ - 3 = 2χ - 3 ή χ - 3 = -2χ + 3 <=> χ = Ο ή χ = 2 Άρα: Α= {Ο, 2 } .

ΕΥΚΛΕΙΔΗΣ Β ' 66 τ.2/19

Page 22: Ευκλειδης Β 66

Μαθη ματικά για την Α ' Λυκείου

Επίσης: χ5= -χ2 <::::> χ5+χ2=0 <::::> χ2(χ3+ 1 )=Ο <::::> χ 2 = 0 ή χ3 + 1 = 0 <::::> χ = 0 ή Χ = - 1

οπότε : Β= {- 1 , Ο} Έτσι ΑυΒ= {- 1 , Ο, 2 } και AnB= {O } Άσκηση 2 '1 Να γράψετε με αναγραφή των στοιχείων του το σύνολο

Α = {χ ε JR 1 ( χ2 - 3χ + 2y + lx - 1 1 + .Jx2 - 1 = ο} Λύση 'Εχουμε : (χ2 - 3χ + 2 ) 2 + l x - 1 I + R"=l = 0 ( 1

Στο 1 ο μέλος της ( 1 ) έχουμε άθροισμα μη αρνητι­κών ποσοτήτων, οπότε για να ισούται αυτό με μη­δέν θα πρέπει και αρκεί κάθε όρος του αθροίσμα­τος να ισούται με μηδέν.

(χ2 -3χ +2)2 = 0 χ2 -3χ+ 2= 0} Άρα: ( 1 ) <=>lx - 11 =0 <::::> χ - 1 = 0

�χ2 - 1 = 0 χ2 - 1 = 0

Η ρίζα ρ= 1 της δεύτερης εξίσωσης επαληθεύει τις άλλες δύο, δηλαδή αποτελεί τη λύση της εξί­σωσης ( 1 ), άρα Α= { 1 } .

Α.σκηση 3 '1 Να γράψετε ποιο σύνολο παριστάνει το γραμμοσκιασμένο μέρος σε καθένα από τα παρακάτω διαγράμματα του Venn. α)

Α

β)

Β

Γ γ)

δ)

Λ1Jση α) Α-Β β) AnBnΓ γ) (AnB) ' δ) (ΑυΒ) ' :\σκηση 4 '1 Έστω το βασικό σύνολο Ω={ 1 , 2, 3, 4, 5, 6, 7, 8, 9, 10} και A={xeiN/3:::;x<7} , Β={χεΩ/χ περιττός αριθμός} . Να γράψετε τα σύνολα:

α) Α ', β) Β ', γ) Α 'υΒ, δ) Α-Β, ε) Β-Α στ) A 'nB ', ζ) (AnB) ', η) (ΑυΒ) '

Λύση Έχουμε : Ω= { 1 , 2 , 3 , 4, 5 , 6, 7, 8, 9, 1 0 } , Α= {3 , 4 , 5 , 6 } και Β= { 1 , 3 , 5 , 7 , 9 } οπότε είναι: α) Α '= { 1 , 2, 7, 8, 9, 1 0 } β) Β '= {2, 4, 6 , 8 , 1 0 } γ) Α'υΒ= { 1 , 2 , 3 , 5 , 7 , 8 , 9, 1 0 } δ) Α-Β = {4, 6 } ε) Β-Α= { 1 , 7 , 9 } στ) A'nB '= {2 , 8 , 1 0 } ζ) Είναι AnB= {3 , 5 } οπότε (AnB) ' = { 1 , 2 , 4, 6, 7 , 8 , 9, 1 0 } η ) Είναι ΑυΒ= { 1 , 3 , 4 , 5 , 6 , 7 , 9 } οπότε (ΑυΒ) '= {2 , 8, 1 0 }

Γενικά περί συναρτήσεων Ερωτήσεις Θεωρίας

l . Να δοθεί ο ορισμός της συνάρτησης από το σύνολο Α στο σύνολο Β .

2 . Τι ονομάζουμε πεδίο ορισμού ή σύνολο ορισμού για μια συνάρτηση f;

3 . Τι ονομάζουμε ανεξάρτητη μεταβλητή και τι εξαρτημένη για μια συνάρτηση f;

4 . Τι λέμε σύνολο τιμών μιας συνάρτησης f και πώς συμβολίζεται;

5 . Τι ονομάζουμε τύπο για μια συνάρτηση f; 6 . Πώς ορίζεται το πεδίο ορισμού α) για μια

πολυωνυμική συνάρτηση ; β) για μια ρητή συνάρτηση ; γ) για μια άρρητη συνάρτηση ;

7 . Πώς ορίζεται το συμμετρικό ενός σημείου Α(χ0, ψο) ως προς: α) τον άξονα χ 'χ; β) τον άξονα ψ 'ψ; γ) την αρχή των αξόνων 0(0, Ο) ; δ) τη διχοτόμο της 1 ης και 3ης γω­νίας των αξόνων;

8 . Να γράψετε τον τύπο που δίνει την από­σταση των σημείων Α(χ ι , ψ ι ) και B(xz, ψz) .

ΕΥΚΛΕΙΔΗΣ Β ' 66 τ.2/20

Page 23: Ευκλειδης Β 66

Μαθη ματικά για την Α ' Λυκείου

9 . Στο επίπεδο των αξόνων πώς αναγνωρί­ζουμε αν μια καμπύλη είναι γραφική πα­ράσταση συνάρτησης;

I Ο . Πώς βρίσκουμε τα σημεία τομής της γρα­φικής παράστασης μιας συνάρτησης με τους άξονες χ 'χ και ψ 'ψ;

Ερωτήσεις του τύπου «σωστό - λάθος» 1 1 . Το παρακάτω σχήμα παριστάνει συνάρτηση .

1 2 . Αν f(x)=x2-x+5 τότε f(x+ 1 )=χ2+χ+5 . Σ -

1 3 . Η συνάρτηση f ( χ ) = .J χ - 9 ορίζεται στο IR. Σ - Λ

1 4 . Η συνάρτηση f ( χ ) = +- ορίζεται στο χ + 1

IR. Σ - Λ 1 5 . Η απόσταση των σημείων Α(3 , 1 ) και

Β(3 ,5 ) ισούται με 5 . Σ - Λ ι 6 . Το σημείο (3 , 1 ) είναι συμμετρικό του ση­

μείου (3 ,- 1 ) ως προς τον άξονα ψ 'ψ . Σ - Λ

Ερωτήσεις πολλαπλής επιλογής ι 7. Ποιας από τις παρακάτω συναρτήσεις η

γραφική παράσταση διέρχεται από το ση-

μείο Α (.!. .!.) ·

3 ' 9 ' 1 α. f ( x ) = -2 χ 1

γ. f ( x ) = -- x 3

1 β. f ( x ) = χ + -3

1 δ. f ( x ) = -χ 3 ι 8 . Ποια από τις παρακάτω γραφικές παρα­

στάσεις δεν αντιστοιχεί σε γραφική παρά­σταση συνάρτησης; α.

ψ

ο χ

ψ'

β.

χ ο χ

ψ γ.

ψr χ

ψ δ.

ψ

χ ' χ

ψ '

1 9 . Η γραφική παράσταση της συνάρτησης f(x)=x2-5x+4 τέμνει τον άξονα χ 'χ στα σημεία: α. ( 1 ,0) και (4,0) β . (-1 ,0) και (-4,0) γ. (- 1 ,0) και (4,0) δ. ( 1 ,0) και (-4,0) .

20 . Η γραφική παράσταση της συνάρτησης

f ( ) 3χ - 1 2 ' ' ξ . χ = 2 τεμνει τον α ονα ψ ψ στο χ + 4

σημείο : α. (0,3) β. (0,-3) γ. (3 ,0) δ.(-3 ,0) . Ασκήσεις Λυμένες

Άσκηση ι ' � Να βρείτε το πεδίο ορισμού Α των συναρ­τήσεων f με τύπο:

α) f(x)=x7-7 β) r ( χ) = -ι l

x_ χ - 1

ΛίJση α) Η συνάρτηση f είναι πολυωνυμική οπότε

A=R. β) Η συνάρτηση f είναι ρητή, οπότε για να ο­

ρίζεται πρέπει και αρκεί ο παρονομαστής της να είναι διάφορος του μηδενός. Όμως: I χ l - 1i0<=> I χ I i:1 <=:>xi1 και xi- 1 , οπότε

A=R- {- 1 , 1 } ΕΥΚΛΕΙΔΗΣ Β ' 66 τ.2/21

Page 24: Ευκλειδης Β 66

Μαθη ματικά για την Α ' Λυκείου

γ) Ο τύπος της συνάρτησης f περιέχει ρητή και άρρητη έκφραση, οπότε για να ορίζεται πρέπει και αρκεί ο παρονομαστής να είναι διάφορος του μη­δενός και το υπόρριζο θετικό ή μηδέν. Δηλαδή

πρέπει και αρκεί: χ ::f. 0 }

( 1 ) � χ ::f. 0 } �

χ + 3 ;::: Ο χ ;::: -3 χ Ε [-3, 0) υ (Ο, +οο) . Άρα: Α=[-3 ,0) υ (Ο,+οο)

λ.σκηση 2η Δίνεται η συνάρτηση f με τύπο :

f (χ) = {αχ - 1, αν χ � Ο β - 2χΖ , αν χ > Ο

α) Να βρείτε τα α, β ώστε να είναι f(l)=3 και f(-2)=1 β) Για τις τιμές των α, β που βρέθηκαν στο (α) ερώτημα να υπολογίσετε την τιμή της

παράστασης Κ = 2f (Ο) - 3f (-�) + 5f ( �) . Λύση α) Έχουμε: 1 >0 οπότε: f( l )=β-2 · 1 2=β-2 Επίσης: -2<0 οπότε : f( -2)=α( -2)- 1 = -2α-1

' f ( 1 ) = 3 } β - 2 = 3 } β = 5 } Όμως: � � f (-2) = 1 -2α - 1 = 1 α = - 1

β) Λόγω του (α) είναι:

f ( x ) = {-x - 1, αν χ � Ο 5 - 2χ2 ,αν χ > Ο

οπότε : K = 2f (0) - 3f ( -�) + 5f (�) =

2 ( + ι ) - 3 (Ξ - ,) + s [s - 2 (Ξ) ' ] � . . . � 22 .

λ.σκηση 3 '1 Δίνεται η συνάρτηση

r ( χ) � {Γ' :: lx l < 2

α) Να βρείτε τις τιμές

f(-1), f(-3), r( ι;) και

β) Ν α λυθεί η εξίσωση : λχ- f(-4)=λf(O), λEIR.

.\ίJση {χ - 5, αν -2 < χ < 2 α) Είναι: f ( χ ) = 1 οπότε :

- , αν χ � -2 ή χ ;::: 2 χ

• - 1 Ε (-2,2) => f(- 1 )= -1-5= -6 1

• -3 Ε (-οο,-2] υ[2,+οο) => f(-3)= -3

• 1 2 ( 12) 1 5 S Ε ( -οο, -2] υ [2, +οο) => f S = 12 =

12 5

β) Επίσης: -4 Ε (-οο,-2] υ [2 ,+οο) => f(-4)= -..!_ και 4

το Ο Ε (-2,2) => f(0)=0-5= -5 . Έτσι: λχ- f(-4)=λf(O)

� λχ - ( -�) = λ (-5 ) � λχ = -5λ -� �

-20λ - 1 λχ = ( 1 ) με λEIR. 4

Για την εξίσωση ( 1 ) διακρίνω τις περιπτώσεις :

1. ) Α 'l _J.O • ( 1 ) -20λ - 1 ν 'vt- τοτε: � χ = ---4λ

ii) Αν λ=Ο τότε η ( 1 ) γράφεται: Ο · χ = _..!_ 4

που είναι εξίσωση αδύνατη . Άσκη ση 4η Σε ένα χάρτη έχει οριστεί σύστημα συντε­ταγμένων έτσι, ώστε οι κορυφές ενός τριγω­νικού οικοπέδου ΑΒΓ να αντιστοιχούν στα σημεία Α(4,-2), B(l ,-2) και Γ(4, 1 ). α) Να δείξετε ότι το τριγωνικό οικόπεδο εί­ναι ισοσκελές β) Ν α δείξετε ότι το τριγωνικό οικόπεδο εί­ναι ορθογώνιο γ) Να υπολογίσετε το εμβαδόν του τριγωνι­κού οικοπέδου ΛίJση α) Είναι: ψ Α= ψ8= -2, οπότε :

(ΑΒ)= I ΧΓΧΑ I = 1 1-4 1 = l -3 1 =3 . Επίσης χΑ= xr= 4, οπότε:

(ΑΓ)= I Ψr-ΨΑ I = l l-(-2) I = 1 3 1 =3 Δηλαδή : (ΑΒ)=(ΑΓ), οπότε το τριγωνικό οι­

κόπεδο είναι ισοσκελές .......:.------

β) Είναι (BΓ) =J( xr -x8 )2 +(Ψr -Ψ8 )2 = . . . =J18 οπότε (ΒΓ)2 = 1 8 .

ψ

Γ

χ ' ο χ

-2 ψ '

(α ) Επίσης: (ΑΒ)2+(ΑΓ)2 = 32+32=9+9= 1 8 . Άρα

ισχύει: (ΒΓ)2=(ΑΒ)2+(ΑΓ)2 , δηλαδή το τριγωνικό οικόπεδο είναι και ορθογώνιο .

ΕΥΚΛΕΙΔΗΣ Β ' 66 τ.2/22

Page 25: Ευκλειδης Β 66

Μαθη ματικά για την Α ' Λυκείου

1 ( α ) 1 9 γ) Θα είναι: Ε = - (ΑΒ) (ΑΓ) = -3 · 3 = -

2 2 2 Άσκηση 5'� Δίνονται τα σημεία Α(χ+1 ,2) και B(l ,2-x). α) Να βρείτε τον πραγματικό αριθμό χ ώ-στε: ( ΑΒ) = 2.J2 β) Για τις τιμές του χ που βρέθηκαν στο (α) ερώτημα να βρείτε το σημείο Γ του άξονα ψ 'ψ ώστε να ισχύει (Γ Α)=(ΓΒ). τι παρατη­ρείται; Λύση

α) Είναι: (ΑΒ) = �( χ8 - χΑ )2 + (ψ8 - ψΑ )2

=�( 1 - χ - 1)2 + ( 2 - χ - 2) = .Jx2 + χ2 = W .

Όμως ( ΑΒ) = 2.J2 , οπότε πρέπει:

w = 2.J2 <=> 2χ 2 = 8 <=> χ2 = 4 <=> χ = 2 ή χ = -2 .

β) Έστω το Γ(Ο,ψ 1 ) σημείο του άξονα ψ 'ψ. Για χ= -2 είναι: Α(- 1 ,2) και Β( 1 ,4) . Τότε : ( ΓΑ) = . . . = �ψ; - 4ψ1 + 5 και

( ΓΒ) = . . . = �ψ� - 8ψ1 + 1 7 Άρα (ΓΑ) = (ΓΒ)

<=> �Ψ; - 4ψ1 + 5 = �Ψ� - 8ψ1 + 1 7 <=> . . . <=> Ψι = 3 . Η τιμή ψ 1 =3 καθιστά τα υπόριζα θετικά οπότε

είναι δεκτή . Άρα Γ(Ο,3) Για χ=2 είναι: Α(3 ,2) και B( l ,0) και εργαζόμε­

νοι με όμοιο τρόπο βρίσκουμε Γ(Ο,3) οπότε παρα­τηρούμε ότι και στις δυο περιπτώσεις βρήκαμε το ίδιο σημείο Γ.

Λ . Η Συνάρτη ση t'(χ )=αχ+β ευθείας

Ερωτήσεις Θεωρίας 1 . Πώς ορίζεται η γωνία μιας ευθείας με τον

άξονα χ 'χ ; 2 . Πότε μια ευθεία σχηματίζει με τον άξονα

χ 'χ γωνία 0°. 3. Πώς ορίζεται ο συντελεστής διεύθυνσης

μιας ευθείας με εξίσωση ψ=αχ+β και με τι ισούται;

4 . Δίνεται ευθεία ε: ψ=αχ+β. Τι συμβαίνει με τη γραφική της παράσταση όταν : α>Ο, α<Ο, α=Ο.

5 . Τι γνωρίζετε για τη γραφική παράσταση των ευθειών : ψ=αχ και ψ=αχ+β;

6 . Ποια είναι η εξίσωση της διχοτόμου της Ι ης και 3 ης γωνίας των αξόνων καθώς και της 2ης και 4ης γωνίας των αξόνων;

7 . Ν α δοθούν οι συνθήκες παραλληλίας και καθετότητας δυο ευθειών

8. Με σχήμα να επιλυθούν γραφικά οι ανι­σώσεις : αχ+β>Ο, αχ+β:-::;0, I χ I <θ, I χ I ;?:θ Ε ρωτήσης του τίJπου «σωστό - λάΟος»

9 . Η ευθεία με εξίσωση : ψ=κ με κεΙR είναι παράλληλη στον άξονα χ ' χ και τέμνει τον άξονα ψ 'ψ στο σημείο (Ο,κ) . Σ- Λ

Ι Ο. Η ευθεία με εξίσωση χ=λ με λεΙR είναι παράλληλη στον άξονα ψ 'ψ και τέμνει τον άξονα χ 'χ στο σημείο (λ,Ο). Σ- Λ

1 1 . Η εξίσωση του άξονα χ ' χ είναι: χ=Ο και η εξίσωση του άξονα ψ 'ψ είναι: ψ=Ο. Σ- Λ

1 2 . Για κάθε ευθεία παράλληλη στον άξονα ψ 'ψ δεν ορίζεται συντελεστής διεύθυνσης. Σ - Λ

1 3 . Η ευθεία με εξίσωση ψ= -χ είναι η διχο­τόμος της 1 ης και 3ης γωνίας των αξό­νων.Σ-Λ Ε ρωτήσεις πολλαπλής επιλογt)ς

1 4 . Η ευθεία : χ = -3 έχει συντελεστή διεύθυν­σης: α. -3 , β. Ο, γ. 1 , δ. δεν ορίζεται

1 5 . Η ευθεία με εξίσωση ψ=κ+λ είναι ο άξο­νας χ ' χ όταν : α. κ=λ, β. κ= 1 /λ γ. κ= -λ δ. λ= 1 /κ

1 6 . Η γωνία που σχηματίζει η ευθεία ε: ψ=(-�-2)χ με τον άξονα χ 'χ είναι:

α. αμβλεία β. ορθή γ. οξεία δ. 45° 1 7 . Ποια από τις παρακάτω εξισώσεις δεν εκ­

φράζει συνάρτηση ; α. ψ=3 β. χ=3 γ. ψ=3χ δ. χ=3ψ

1 8 . Η ευθεία με εξίσωση ψ=2007χ+2008 τέ­μνει τον άξονα ψ ' ψ στο σημείο :

α. (0,2007) β. (0,2008) γ. (2007,0) δ. (2008,0) Ασκήσε ις Λυμένες

Άσκηση Ι ' � Να βρεθεί το σημείο τομής των ευθειών:

ει : ψ=2χ - 1 και ε2 : ψ= - χ+5 Λύση Το σημείο τομής των ευθειών ε 1 , ε2 θα έχει

συντεταγμένες (χ,ψ) που θα επαληθεύουν συγχρό­νως τις εξισώσεις και των δυο ευθειών, δηλαδή το

' ψ= 2χ- 1 } ( )

ψ= 2χ- 1 } χ = 2} συστημα: 1 <=> <=> ψ=-χ+5 2x - l = -x +5 ψ=3

Άρα το ζητούμενο σημείο είναι το Α(2 ,3) . Άσκη ση 2 ' � Να βρεθεί η εξίσωση της ευθείας ει που εί­ναι παράλληλή στην ευθεία ε2 : ψ=2χ+3 και διέρχεται από το σημείο Α(4,0) ΛίJση Έστω ε 1 :ψ=αχ+β η ζητούμενη ευθεία, οπότε:

ε ι //εz <=> (α = 2) και Α ε ( ε 1 ) <=> Ο = α · 4 + β Τε­β :;t: 3

λικά: (α = 2 ) . Άρα ε 1 :ψ=2χ-8 . β = -8

Άσκηση 3' � Να επιλυθούν γραφικά οι ανισώσεις: α) 2χ-6>0 β) 2χ-6:50

ΕΥΚΛΕΙΔΗΣ Β ' 66 τ.2/23

Page 26: Ευκλειδης Β 66

Μαθηματικά για την Α ' Λυκείου

ψ

χ ' ο χ

ΛtJση Θεωρούμε την ευθεία με εξίσωση ε: ψ=2χ-6 που τέμνει τον άξο­να χ 'χ στο σημείο Α(3 ,0) και τον ψ 'ψ στο σημείο Β(Ο,--6) και έχει τη γραφική παράσταση που φαί­νεται στο παρακάτω σχήμα. α) Είναι φανερό ότι η aνίσωση 2χ--6>0 εί-

ναι ισοδύναμη με την ψ>Ο οπότε γραφικά η λύση της δίνεται από τις τετμημένες των σημείων της aνοιχτής ημιευθείας Αζ (γιατι;). Προφανώς οι τετμημένες των σημείων αυτών είναι μεγαλύτερες του 3 . β) Όμοια σκεπτόμενοι οδηγούμαστε γραφικά ότι η λύση της ανίσωσης 2χ-6:::Ο αποτελείται από τις τε­τμημένες των σημείων της κλειστής ημιευθείας Αζ ' (γιατί;) . Προφανώς οι τετμημένες των σημείων αυτών είναι μικρότερες ή ίσες του 3 .

., :: ' f · "Γ'' : Στην ανοιχτή ημιευθεία δεν περιλαμ­βάνεται η αρχή της ενώ αντίθετα στην κλειστή πε­ριλαμβάνεται.

, (Η�ψηι Να επιλυθούν γραφικά οι ανισώσεις: α) Ι χ Ι <3 β) Ι χ Ι � 3 \{Jση Θεωρούμε την εξίσωση : ψ= I χ ι . Εφόσον

l x l --{ χ , αν χ � Ο -χ, αν χ < 0

η γραφική παράσταση της

ψ δ ' δ

ψ=3

ψ= l x l θα αποτελείται

από τις διχο­τόμους της 1 ης και 2ης γωνίας των

3 χ αξόνων ό-πως φαίνεται ψ και στο σχή­

μα. Επιπλέον στο σχήμα τοποθετούμε και την εξί­σωση της ευθείας ψ=3 που τέμνει τις δυο διχοτό­μους στα σημεία Α(3 ,3 ) και Β(-3 ,3 ) αντίστοιχα.

α) Η aνίσωση I χ I <3 είναι ισοδύναμη με την Ο:::ψ<3 οπότε γραφικά η λύση της αποτελείται από τις τετμημένες των ευθυγράμμων τμημάτων ΟΑ και ΟΒ με εξαίρεση τα σημεία Α και Β (γιατί; ) Προφανώς η λύση είναι τα χ με -3<χ<3

β) Με όμοιο τρόπο να προσδιοριστεί γραφικά από τον αναγνώστη η λύση της aνίσωση ς I χ I �3 .

λ - ι s - λ ι Δίνεται ε: ψ = -- χ - -- με λ :;t: -ι - 2λ 2λ - ι 2

Να προσδιοριστεί ο λ ώστε η ευθεία ε: α) Να είναι παράλληλη στον άξονα χ 'χ β) Να είναι παράλληλη στην ευθεία

ει : ψ=χ-ι3 γ) Να είναι κάθετη στην ευθεία

ι ε2 : ψ = - χ - 3 4

δ) Να διέρχεται από την αρχή των αξόνων ε) Να διέρχεται από το σημείο Α(ι ,-2) λ{Jση Η ευθεία ε είναι της μορφής: ψ=αχ+β με λ - 1 5 - λ 1 α = -- και β = --- , λ :;t: - οπότε: 1 - 2λ 2λ - 1 2

α) ε // χ 'χ � � 1 - 2λ � λ = 1 (α = Ο) λ - 1 = 0} β :;t: Ο λ :;t: 5

και είναι ε: ψ= --4 . β) (α = 1 ) λ - 1 λ - 5 ε // ε1 � � -- = 1 και -- :;t: -1 3 �

β :;t: - 1 3 1 - 2λ 2λ - 1 2 λ - 5 . 2 � λ = - και -- :;t: -13 . Η τιμη λ = - απορρί-3 2λ - 1 3

πτεται διότι δεν επαληθεύει την δεύτερη . Άρα α­ποκλείεται η ευθεία (ε) αυτή να είναι παράλληλη της (ε ι ) .

1 λ-1 γ) ε ..l ε2 � α- = -1 � α=--4 �-- =--4 � 4 1 - 2λ

λ - 1 = --4 + 8λ � 7λ = 3 � λ = � (και παίρνει τη 7

μορφή ε: ψ = --4χ+ 32) δ) Η ε διέρχεται από την αρχή των αξόνων

5 - λ �β=Ο � --- = Ο � λ = 5 (και παίρνει τη 2λ - 1

, 4 ) μορφη ε : ψ = -9χ .

λ - 1 5 - λ ε) Α ε (ε) � -2 = -- + -- � 1 - 2λ 1 - 2λ

3 -2 ( 1 - 2λ) = λ - 1 + 5 - λ � -2 + 4λ = 4 � λ = -2

( , , 1 7 ) και παιρνει τη μορφη ε : ψ = -4

χ -4

.

Απαντήσεις σωστού - λάθους και πολλαπλής επι­λογής Εξισ<!>σεις 2 "" /)αθμοίJ 8 . Σ, 9 .Σ, 1 0 . Σ. 1 1 . Λ, 1 2 . Λ, 1 3 . Σ, 1 4 . γ, 1 5 . α, 1 6 . β, 1 7 . δ, 1 8 .γ, 1 9 . δ, 20. γ. ΣiJνολα8. Λ, 9 . Σ, 1 0 . Λ, 1 1 . Σ, 1 2 . Λ, 1 3 . γ, 1 4. δ, 1 5 . α, 1 6 . β l:υναρτήσης 1 1 . Σ, 1 2 . Σ, 1 3 . Λ, 1 4. Σ, 1 5 . Λ, 1 6 . Λ, 1 7 . δ, 1 8 . γ, 1 9 . α, 20. β Ευ0εί r.ς9. Σ, 1 0 . Σ, 1 1 . Λ, 1 2 . Σ, 1 3 . Λ, 1 4 . δ, 1 5 . γ, 1 6 . α, 1 7 . β, 1 8 . β.

ΕΥΚΛΕΙΔΗΣ Β' 66 τ.2/24

Page 27: Ευκλειδης Β 66

ΓΕΩΜΕΤΡΙΑ ΠΑΡΑΛΛΗΛΟΓΡΑΜΜΑ

Καρδαμίτσης Σπύρος - Κυριακόπουλος Θανάσης « . . . . . Αργότερα στην ηλικία των δώδεκα χρόνων έζησα την εμπειρία μιας άλλης ·εκπληξης ·; ολότελα διαφορετικής φύ­

σης διαβάζοντας στην αρχή της καινούργιας σχολικής χρονιάς ένα μικρό βιβλίο πάνω στην Ευκλείδεια Γεωμετρία. Εκεί βρήκα αξιώματα -όπως για παράδειγμα, τα τρία ύψη ενός τριγώνου τέμνονται στο ίδιο σημείο - που αν και μα­

κριά από το να είναι βέβαια, μπορούσαν ωστόσο να αποδειχτούν με τόση βεβαιότητα, που και η παραμικρή αμφιβολία

φαινόταν ολότελα παράλογη. Αυτή η διαύγεια, κι αυτή η σιγουριά μου κάνουν μια απερίγραπτη εντύπωση. Το ότι έπρε­

πε να δεχόμαστε τα αξιώματα χωρίς να τα αποδείχνουμε δεν με ξένιζε καθόλου. Όπως και νάχει το πράγμα μου αρ­κούσε να μπορώ να θεμελιώνω τις αποδείξεις μου πάνω σε προτάσεις των οποίων η εγκυρότητα φαινόταν ολότελα

αναμφισβήτητη . . . . . . » Albert Einstein (από την αυτοβιογραφία του)

[ ΡΩΤ Η Σ Ε Ι Σ ΣΩΣΤΟ Υ - ΛΛΘΟΥΣ

1 . Υπάρχει παραλληλόγραμμο με τρεις διαδοχι­κές γωνίες οξείες.

2 . Κάθε παραλληλόγραμμο με ίσα ύψη είναι ρόμβος.

3 . Κάθε κυρτό τετράπλευρο με τρεις γωνίες ορ­θές είναι τετράγωνο.

4. Κάθε κυρτό τετράπλευρο που έχει κάθετες δι-αγώνιες είναι ρόμβος.

5 . Κάθε τετράγωνο είναι και ρόμβος. 6 . Κάθε ρόμβος είναι και τετράγωνο. 7. Κάθε διαγώνιος παραλληλογράμμου το χωρί­

ζει σε δύο ίσα τρίγωνα. 8. Σε κάθε παραλληλόγραμμο οι διαγώνιοι του

είναι ίσες. 9. Κάθε ρόμβος με μια γωνία ορθή είναι τετρά­

γωνο. 1 Ο . Κάθε παραλληλόγραμμο με δύο διαδοχικές

γωνίες ίσες είναι ορθογώνιο . 1 1 . Κάθε τετράπλευρο που έχει κάθετες διαγώνιες

και μία από αυτές είναι διχοτόμος των δύο γω­νιών του, είναι ρόμβος.

Α Π Λ Ν Τ Η Σ Ε Ι Σ

Λ - Σ - Λ - Λ - Σ - Λ - Σ - Λ - Σ - Σ - Λ Ε Ι>ΩΤ Η Σ Ε Ι l: Π ΟΛΛΑ Π Λ Η Σ Ε Π Ι Λ Ο Γ Η Σ

Ι . Ένα παραλληλόγραμμο έχει περίμετρο 30cm και η μια του πλευρά είναι τετραπλάσια της άλλης, τότε η μεγαλύτερή του πλευρά είναι ίση με : Α. l Ocm Β. 1 2cm Γ. 1 3cm

Δ. 1 4cm Ε. 1 5cm 2 . Στο σχήμα η διαγώνιος του ορθογωνίου είναι

2 J2 , η γωνία ω είναι 30° και η μεγαλύτερη πλευρά του είναι J6 , τότε η περίμετρός του είναι: Α. 2 J3 ( l+ J2 ) Δ. 4 .J6

Β . .J12 Γ. 4 J2 Ε. 2 J2 ( I+ J3 )

3 . Υπάρχει παραλληλόγραμμο που ΔΕΝ είναι ρόμβος όταν:

Α. Οι διαγώνιοί του τέμνονται κάθετα. Β. Δύο διαδοχικές πλευρές του είναι ίσες. Γ. Οι αποστάσεις των απέναντι πλευρών του είναι ίσες. Δ. Δύο απέναντι πλευρές του είναι ίσες και πα­ράλληλες. Ε. Όλες οι πλευρές του είναι ίσες.

4 . Για να είναι ένα τετράπλευρο τετράγωνο αρ-κεί:

Α. Οι διαγώνιοί του να τέμνονται κάθετα. Β. Να είναι παραλληλόγραμμο και να έχει μια γωνία του ορθή . Γ. Να είναι ορθογώνιο και ρόμβος. Δ. Να έχει τις απέναντι πλευρές του είναι ίσες και παράλληλες. Ε. Να έχει δύο γωνίες του ορθές.

ΕΥΚΛΕΙΔΗΣ Β' 66 τ.2/25

Page 28: Ευκλειδης Β 66

Μ αθηματικά για την Α ' Λυκείου

5 , Οι διαγώνιες κάθε ορθογωνίου είναι: Α. κάθετες Β. διχοτομούν τις γωνίες του ορθογωνίου Γ. δεν διχοτομούνται Δ. είναι ίσες Ε. είναι άνισες. , \[ ] ,\ :\ η-η : �<: η.: : Β - Ε - Δ - Γ - Δ \ ::: κ π-η:: ΙΗJ ! η

Δίνεται παραλληλόγραμμο ΑΒΓ Δ. Οι διχο­τόμοι των γωνιών Α, Β τέμνουν τη ΓΔ στα Ρ, Ε

Λ Λ

και οι διχοτόμοι των γωνιών Γ, Δ τεμνουν την

ΑΒ στα Σ, Τ αντιστοίχος. Να δείξετε ότι:

α) Το τετράπλευρο ΔΕΒΤ είναι παραλληλό­

γραμμο. β) Το τετράπλευρο ΚΛΗΘ που σχημα­

τίζουν οι διχοτόμοι είναι ορθογώνιο.

γ) Οι διαγώνιοι του παραπάνω ορθογωνίου

είναι παράλληλες προς τις πλευρές του ΑΒΓ Δ

και είναι ίσες με τη διαφορά των πλευρών του.

α) Το τετράπλευρο ΑΒΓ Δ είναι παραλληλόγραμμο Λ Λ

επομένως είναι ΤΒ // ΔΕ ( 1 ) και Ε ι = Βι (ως ε-Λ Λ Λ Λ

ντός εναλλάξ) Βι = Δ2 (γιατί Β = Δ επομένως Λ Λ Β Δ , Λ Λ

' Λ Λ

- = - η Β ι = Δ2 ) αρα Ε ι = Δ2 2 2

οπότε ΔΤ // ΕΒ (2) από τις σχέσεις ( 1 ) και (2) έχουμε ότι το τετρά­πλευρο ΔΕΒΤ είναι παραλληλόγραμμο.

Α Σ Τ Β

..:. Ε

β) Ανάλογα έχουμε ότι και το τετράπλευρο ΑΣΓΡ είναι επίσης παραλληλόγραμμο επομένως είναι ΚΘ // ΛΗ και ΚΑ // ΘΗ, τότε το τετράπλευρο ΚΛΗΘ είναι παραλληλόγραμμο . Επίσης είναι:

Λ Λ Λ Λ Λ Λ Α Β Α+ Β 1 80° ο Λ Αι + Δ ι = - + - = -- = -- = 90 άρα Κ =90°

2 2 2 2 επομένως το τετράπλευρο ΚΛΗΘ είναι ορθογώνιο . γ) Το τρίγωνο ΑΔ Τ είναι ισοσκελές αφού η ΑΚ εί-

ναι διχοτόμος και ύψος, άρα η ΑΚ είναι και διάμε­σος του τριγώνου, οπότε ΔΚ = ΚΤ. Ανάλογα έ­χουμε ΕΗ = ΗΒ.

Αλλά από το παραλληλόγραμμο ΔΕΒΤ έχουμε

ΔΤ = 11 ΕΒ δηλαδή ΔΤ = ΕΒ , οπότε ΔΚ =//ΕΗ 2 2

οπότε το τετράπλευρο ΔΕΗΚ είναι παραλληλό­γραμμο, συνεπώς ΚΗ//ΔΕ, δηλαδή ΚΗ//ΓΔ .

Είναι Θι = Κ ι (από το ορθογώνιο ΚΘΗΛ),

Κι = Δ2 (από το παραλληλόγραμμο ΚΗΕΔ) και

Δ2 = Δ ι (Δ τ διχοτόμος) , άρα Θ ι = Δ ι ' επομένως ΘΑ // ΑΔ // ΒΓ. Ακόμα ΚΗ=ΔΕ=ΓΔ-ΓΕ=ΓΔ-ΒΓ .

ΛL K H l: H 2 '� Σε τετράπλευρο ΑΒΓ Δ οι απέναντι γωνίες

του Β και Δ είναι παραπληρωματικές. Οι πλευ­

ρές του ΔΑ και ΓΒ τέμνονται σ_το σημείο Ε, ενώ

οι ΑΒ και ΔΓ τέμνονται στο σημείο Ζ. Να απο­

δείξετε ότι τα σημεία τομής των διχοτόμων των

γωνιών Ε και Ζ με τις πλευρές του ΑΒΓ Δ είναι

κορυφές ρόμβου.

Α Π Ο.\ Ε Ι Ξ Η Ε

..:.

Εάν Ρ, Κ, Λ και Μ τα σημεία τομής των διχο-Λ Λ

τόμων των γωνιών Ε και Ζ με τις πλευρές του τετραπλεύρου ΑΒΓ Δ θα αποδείξουμε ότι το τε­τράπλευρο ΡΚΛΜ είναι ρόμβος.

Λ Λ Ονομάζουμε τις γωνίες ΑΜΖ = ω και

ΕΚΜ = φ , τότε οι γωνίες � και φ είναι εξωτερι­κές στα τρίγωνα ΔΜΖ και ΚΒΖ αντίστοιχα, επο-

Λ Λ Λ z μένως ισχύουν: ω = Δ+ -2

ΕΥΚΛΕΙΔΗΣ Β ' 66 τ.2/26

Page 29: Ευκλειδης Β 66

Μαθηματικά για την Α ' Λυκείου

Λ Λ Λ Λ Λ z Λ Ζ Λ Ζ φ = Βι + - = 1 80° - Β+ - = Δ+ -

Λ Λ 2 2 2

( αφού Β+ Δ = 1 80° από την υπόθεση .)

άρα � = � , δηλαδή το τρίγωνο ΕΜΚ είναι ισο­

σκελές και η διχοτόμος του ΕΗ είναι διάμεσος και ύψος, άρα ΕΗ .l ΜΚ και ΜΗ = ΗΚ. Αντίστοιχα, το τρίγωνο ΡΖΛ είναι ισοσκελές και ισχύουν ΖΗ .l Ρ Λ και ΡΗ = ΗΛ. Αφού ΜΗ = ΗΚ και ΡΗ = ΗΛ, τότε το τετράπλευ­ρο ΜΡΚΛ είναι παραλληλόγραμμο στο οποίο οι διαγώνιοί του τέμνονται κάθετα, συνεπώς είναι ρόμβος.

ΛΣ Κ Ι Ι � Η J " Α ν Ε, Ζ, Η και Θ είναι τα μέσα των πλευ­

ρών ΑΒ, ΒΓ, ΓΔ και ΔΑ αντίστοιχα, τετραπλεύ­

ρου ΑΒΓ Δ και Κ, Λ είναι τα μέσα των διαγωνί­

ων του ΑΓ και ΒΔ, να δείξετε ότι:

α) τα τετράπλευρα ΕΚΗΛ και ΖΚΘΛ είναι

παραλληλόγραμμα.

Β

β) οι ευθείες ΕΗ, ΖΘ και ΛΚ συντρέχουν.

Λ Π ΟΔ Ε Ι Ξ Η

λ r::::::..::::-----i----

α) Στο τρίγωνο ΑΒΓ τα σημεία Ε και Κ είναι ΒΓ μέσα των πλευρών του, επομένως ΕΚ =// -και 2

στο τρίγωνο ΒΓ Δ τα σημεία Λ και Η είναι μέσα

11 ΒΓ , των πλευρών του, επομένως ΛΗ = 2 , αρα

ΕΚ=//ΛΗ, τότε το τετράπλευρο ΕΚΗΛ είναι πα­ραλληλόγραμμο .

ΑΒ ΑΒ Ανάλογα έχουμε ότι ΘΛ =// - και ΚΖ =// - ,

2 2

β) Επειδή το τετράπλευρο ΘΛΖΚ είναι πα­ραλληλόγραμμο, η ΘΖ διέρχεται από το μέσο της ΛΚ. Όμοια, αφού το ΕΛΗΚ είναι παραλληλό­γραμμο η ΕΗ διέρχεται από το μέσο της ΛΚ, άρα οι ευθείες ΕΗ, ΘΖ και ΛΚ συντρέχουν, επειδή διέρχονται από το κοινό κέντρο των δυο παραλλη­λογράμμων.

Al: K H l: H 4'1 α) Να αποδείξετε ότι οι διχοτόμοι των εξωτε­

ρικών γωνιών παραλληλογράμμου, τεμνό­

μενες ανά δύο σχηματίζουν ορθογώνιο.

β) Οι διαγώνιες του παραπάνω ορθογωνίου εί­

ναι παράλληλες προς τις πλευρές του πα­

ραλληλογράμμου.

γ) Το άθροισμα των διαγωνίων του ορθογωνί­

ου είναι ίσο με την περίμετρο του παραλλη­

λογράμμου.

Α Π ΟΔ Ε Ι Ξ Η

Λ Λ α) Είναι ΑΔ // ΒΓ επομένως χΑΒ+ yBA =

Λ Λ χΑΒ yBA Λ Λ Λ

1 80° ή -- + -- = 90° ή ω+ φ = 90° τότε Μ = 2 2

Λ Λ Λ 90°. Όμοια είναι Λ = Κ = Ρ = 90° συνεπώς το τε-τράπλευρο ΜΛΚΡ είναι ορθογώνιο .

β) Στο τρίγωνο ΓΒΘ η διχοτόμος του Γ Λ εί­ναι και ύψος του, άρα το τρίγωνο ΒΓΘ είναι ισο­σκελές και η Γ Λ είναι και διάμεσος, επομένως ΒΓ = ΓΘ και ΒΛ = ΛΘ. Όμοια από το ισοσκελές τρί­γωνο ΝΑΔ έχουμε ΑΝ = ΑΔ και ΝΡ = Ρ Δ. Στο τετράπλευρο ΔΝΒΘ οι απέναντι πλευρές του είναι παράλληλες άρα είναι παραλληλόγραμμο,

, ΔΝ ΒΘ , άρα ΘΛ =// ΚΖ επομένως και το τετράπλευρο οπότε : ΔΝ =// ΒΘ, δηλαδη 2 = I/ 2 ' οποτε

ΘΛΖΚ είναι επίσης παραλληλόγραμμο . ΝΡ=//ΒΛ. Επομένως το τετράπλευρο ΝΒΛΡ είναι ΕΥΚΛΕΙΔΗΣ Β ' 66 τ.2/27

Page 30: Ευκλειδης Β 66

Μαθηματικά για την Α ' Λυκείου

παραλληλόγραμμο, άρα ΑΒ I I Ρ Λ και αντίστοιχα ΚΜ ΙΙ ΑΔ

γ) Από το παραλληλόγραμμο ΝΒΛΡ έχουμε : ΡΑ = ΝΒ = ΝΑ + ΑΒ = ΑΔ +ΑΒ.

Επειδή οι διαγώνιες του ορθογωνίου είναι ίσες έ­χουμε ΚΜ = ΡΑ= ΑΔ + ΑΒ επομένως ΡΛ+ΚΜ=2(ΑΔ+ΑΒ), δηλαδή το άθροισμα των δι­αγωνίων του ορθογωνίου είναι ίσο με την περίμε­τρο του παραλληλογράμμου ΑΒΓ Δ.

λΣ Κ Η l: Η 5η Ν α αποδείξετε ότι οι προβολές του κέντρου

ενός ρόμβου στις πλευρές του είναι κορυφές ορ­

θογωνίου . . \ ΓlΟ \ Ε Ι Ξ Η

Έστω ΟΚ, ΟΛ, ΟΜ και ΟΝ οι α­ποστάσεις του κέ­ντρου του ρόμβου Ο από τις πλευρές

του ρόμβου τα Κ,Ο,Μ είναι συ­νευθειακά καθώς

και τα Λ.Ο,Ν (για­τί; ) . Επειδή το Ο είναι σημείο της

διχοτόμου ΑΓ των Γ γωνιών Α και Γ

έχουμε ότι ΟΚ=ΟΛ και ΟΜ=ΟΝ, ( 1 ) Λ Λ

Όμοια ΔΒ διχοτόμος των γωνιών Δ και Β , άρα ΟΚ=ΟΝ και ΟΑ = ΟΜ (2)

Από τις ( 1 ) και (2) έχουμε ΟΚ = ΟΜ και ΟΛ=ΟΝ άρα στο τετράπλευρο ΚΛΜΝ οι διαγώνι­ες του διχοτομούνται, συνεπώς είναι παραλληλό­γραμμο.

Επειδή οι αποστάσεις των απέναντι πλευρών ρόμβου είναι ίσες, έχουμε ΚΜ = ΑΝ, άρα το πα­ραλληλόγραμμο ΚΛΜΝ έχει ίσες τις διαγώνιές του, συνεπώς είναι ορθογώνιο.

ΛΣ Κ Η Σ Η 6η

Σε ισοσκελές τρίγωνο ΑΒΓ φέρνουμε το

ύψος του ΑΔ και προεκτείνουμε την ΒΓ κατά

τμήμα ΓΜ = ΒΓ. Α ν Κ είναι το μέσο του ΑΜ

και η ΔΚ τέμνει την προέκταση της ΑΒ στο ση-

μείο Ε, να δείξετε :

α) το τετράπλευρο ΒΚΓΕ είναι παραλλη-

λόγραμμο. β) ΕΔ =

ΑΜ .

2 γ) το σημείο Γ είναι βαρύκεντρο του τρι­

γώνου ΚΕΜ. Α Π Ο.\ Ε Ι Ξ Η

λ

Ε

J\1

α) Τα τρίγωνα ΒΕΔ και ΔΚΓ είναι ίσα γιατί Λ Λ Δι = Δz (ως κατακορυφήν), ΒΔ = ΔΓ (γιατί στο ι-σοσκελές τρίγωνο ΑΒΓ το ύψος του ΑΔ είναι και

Λ Λ διάμεσός του) και Βι = Γ ι (ως εντός εναλλάξ), α-φού ΚΓΙΙΑΒ (γιατί;) , άρα ΔΕ=ΔΚ. Στο τετράπλευ­ρο ΒΚΓΕ είναι ΔΕ = ΔΚ και ΒΔ = ΔΓ δηλαδή οι διαγώνιοι του διχοτομούνται, συνεπώς είναι πα­ραλληλόγραμμο.

β) Στο ορθογώνιο τρίγωνο ΑΔΜ η ΔΚ είναι

διάμεσος, επομένως ΔΚ = ΑΜ . Αλλά ΔΚ = ΕΔ,

2

άρα ΕΔ = ΑΜ.

2 γ) Από το παραλληλόγραμμο ΒΚΓΕ έχουμε

ΔΚ=ΕΔ, άρα στο τρίγωνο ΚΕΜ η ΜΔ είναι διά­μεσος και ΜΓ=ΒΓ=2Γ Δ. Συνεπώς το σημείο Γ εί­ναι το βαρύκεντρο του τριγώνου ΚΕΜ.

Α .. Σ Κ Η Σ Η 7'1 Έστω Ε σημείο της πλευράς ΑΒ τετραγώ-

Λ

νου ΑΒΓΔ. Αν η διχοτόμος της γωνίας ΕΔΓ τέμνει την πλευρά ΒΓ στο σημείο Ζ, να δείξετε

ότι ΔΕ = ΑΕ + ΓΖ. Λ Π ΟΛ Ε Ι Ξ Η

Προεκτείνουμε τη πλευρά του τετραγώνου ΒΑ κατά τμήμα ΑΗ = ΓΖ, οπότε ΗΕ=ΑΕ+ΓΖ. Αρκεί

ΕΥΚΛΕΙΔΗΣ Β' 66 τ.2/28

Page 31: Ευκλειδης Β 66

Μαθηματικά για την Α ' Λυκείου

λοιπόν να δείξουμε ότι ΔΕ=ΗΕ. Τα ορθογώνια τρίγωνα ΔΑΗ και ΖΔΓ έχουν ΖΓ = ΑΗ (από κατα­σκευή) και ΑΔ = ΔΓ, άρα είναι ίσα, επομένως

Λ Λ Λ Λ Η = Ζι και Δ ι = Δ2 .

Λ Λ Λ

z

Αλλά Η = Ζι = ΑΔΖ (ως εντός εναλλάξ)= Λ Λ Λ Λ ΔJ + Δ4 = ΔJ + Δ2 ( αφού η ΔΖ είναι διχοτόμος) = Λ Λ Λ Λ Λ ΔJ + Δ ι (αφού Δ ι = Δ2 ) = ΗΔΕ . Επομένως το τρίγωνο ΗΕΔ είναι ισοσκελές, άρα ΔΕ = ΗΕ.

z

Από την κορυφή Α του τετραγώνου φέρνουμε την ΑΚ κάθετη στην ΔΖ που τέμνει την ΔΕ στο σημείο Λ και την ΔΓ στο σημείο Ρ. Στο τρίγωνο ΛΔΡ η ΔΚ είναι διχοτόμος και ύψος, άρα το τρί­γωνο είναι ισοσκελές, επομένως ΔΛ = ΔΡ ( 1) και

Λ Λ Ρ ι = Λι .

Λ Λ Λ Λ Όμως Ρ ι = Α ι (ως εντός εναλλάξ) και Λι = Λ2

Λ Λ (ως κατακορυφή), άρα Αι = Λ2 οπότε το τρίγωνο ΑΕΛ είναι ισοσκελές, επομένως είναι ΑΕ = ΕΛ (2)

Στα ορθογώνια τρίγωνα ΑΔΡ και ΑΓΖ έχουμε : Λ Λ

ΑΔ = ΔΓ και Α2 = Δ2 (έχουν τις πλευρές τους κά-θετες) , άρα είναι ίσα , επομένως ΖΓ = ΔΡ (3)

Τότε από τις σχέσεις ( 1 ) , (2) και (3) έχουμε : ΔΕ = ΔΛ + ΛΕ = ΔΡ + ΑΕ = ΖΓ + ΑΕ.

Λl: Κ Η Σ J-1 8 11 Εξωτερικά τετραγώνου ΑΒΓ Δ κατασκευά­

ζουμε τα ισόπλευρα τρίγωνα ΑΒΕ, ΒΓΖ, Γ ΔΗ

και ΔΑΘ. Να δείξετε ότι το τετράπλευρο ΕΖΗΘ

είναι τετράγωνο.

Α Π ΟΛ Ε Ι Ξ Η Ε

Η Αν συμβολίσουμε με α την πλευρά του τετρα­

γώνου τότε : ΕΑ = ΕΒ = ΖΒ = . . . = ΘΑ = α, επομέ­νως τα τρίγωνα ΕΑΘ, ΕΒΖ, ΖΓΗ και ΔΗΘ είναι ισοσκελή .

Λ Λ Λ Λ Επίσης Αι = Βι = Γι = Δ ι =360°-90°-60°-60° =

= 1 50° άρα τα παραπάνω τρίγωνα είναι ίσα, επομέ­νως ΘΕ = ΕΖ = ΖΗ = ΗΘ, δηλαδή το τετράπλευρο ΕΖΗΘ είναι ρόμβος.

Λ Λ 1 80° - 1 50° Ακόμη είναι: Ε ι = Ε2 = = 1 5°, 2

Λ άρα ΘΕΖ = 60° + 1 5° + 1 5° = 90° συνεπώς το τε-τράπλευρο ΕΖΗΘ είναι ρόμβος με μία γωνία ορ­

θή, δηλαδή τετράγωνο. Λl: Κ Η Σ Η 91 1

Λ Λ Δίνεται τετράπλευρο ΑΒΓΔ με Α = Δ = 90°,

Λ ΑΒ > ΓΔ, ΒΓ = 4ΓΔ και Β = 60°. Φέρνουμε την

ΓΗ.lΑΒ και θεωρούμε τα μέσα Ε και Ζ των

πλευρών του ΑΔ και ΒΓ αντίστοιχως. Να δείξε­

τε ότι:

α) Το τετράπλευρο ΑΒΓ Δ είναι τραπέζιο.

β) ΗΒ = ΕΖ.

γ) Το τετράπλευρο ΕΗΒΖ είναι παραλλη­

λόγραμμο.

ΕΥΚΛΕΙΔΗΣ Β' 66 τ.2/29

Page 32: Ευκλειδης Β 66

Μαθηματικά για την Α ' Λυκείου

Λ Π ΟΛ Ε Ι Ξ Η Β

..λ Γ

Eo:t------+---o. Z

Λ Λ

Η Β

α) Αφού Α = Δ = 90°, τα τμήματα ΑΒ και Γ Δ είναι κάθετα στην ΑΔ, συνεπώς είναι μεταξύ τους παράλληλα, άρα το τετράπλευρο ΑΒΓ Δ είναι τρα­πέζιο .

Λ β) Στο ορθογώνιο τρίγωνο ΓΗΒ είναι Β = 60°,

επομένως Ι\ = 30° άρα ΗΒ = ΒΓ = 4ΓΔ = 2Γ Δ. ( 1 )

2 2 Στο τραπέζιο ΑΒΓ Δ η ΕΖ είναι διάμεσός του

, ΕΖ ΑΒ + ΓΔ ΑΗ + ΗΒ + ΓΔ αρα = ---

2 2

= ΓΔ + 2ΓΔ + ΓΔ = 4ΓΔ = 2ΓΔ (2)

2 2 από τις σχέσεις ( 1 ) και (2) έχουμε ότι: ΗΒ = ΕΖ

γ) Αφού η ΕΖ είναι διάμεσος του τραπεζίου, είναι ΕΖ 11 ΗΒ και λόγω του β ερωτήματος είναι ΕΖ = ΗΒ, συνεπώς το τετράπλευρο ΕΗΒΖ είναι παραλληλόγραμμο.

Λl:Κ Η Σ Η I 01 1 Λ

Δίνεται ορθογώνιο τρίγωνο ΑΒΓ ( Α = 90°) Λ

με Β = 30° και ΑΓ = 2λ. Α ν Κ, Λ είναι τα μέσα

των πλευρών του ΑΒ και ΒΓ αντιστοίχως και

Μ, Ν τα μέσα των τμημάτων ΑΚ και Γ Λ αντι­

στοίχως, να δείξετε ότι:

α) το τετράπλευρο ΚΛΓ Α είναι τραπέζιο.

β) το τετράπλευρο ΚΛΔΑ είναι ορθογώνιο,

όπου Δ είναι η προβολή του Λ στην ΑΓ.

γ) 2ΜΝ = 3ΔΝ.

Α Π ΟΔ Ε Ι Ξ Η

α) Επειδή τα Κ, Λ είναι μέσα των πλευρών ΑΒ

και ΑΓ του τριγώνου ΑΒΓ, είναι ΚΛ = /1 ΑΓ ή 2

ΚΛ // ΑΓ, άρα το τετράπλευρο ΚΛΓ Α είναι τραπέ­ζιο .

2λ β) Επειδή ΚΛ // ΑΓ, ΛΔ // ΚΑ (ως κάθετες

Λ στην ίδια ευθεία) και Α = 90°, προκύπτει ότι το τε-τράπλευρο ΚΛΔΑ είναι ορθογώνιο .

Λ γ) Στο τρίγωνο ΑΒΓ είναι Β =30°, επομένως

ΒΓ ΑΓ 2λ ΑΓ = - = ΒΛ = ΛΓ = 2λ και ΚΛ= - =- = λ.

2 ' 2 2 Στο τραπέζιο ΚΛΓ Α η διάμεσος του ΜΝ είναι ίση

με : ΜΝ = ΚΛ + ΑΓ = λ + 2λ =

� . λ 2 2 2

Στο ορθογώνιο τρίγωνο ΛΔΓ η ΔΝ είναι διά-, ΔΝ

ΛΓ 2λ λ , ΜΝ μεσος, οποτε = Τ = 2 = , επομενως

= � · ΔΝ, δηλαδή 2ΜΝ = 3ΔΝ.

2 Α Σ ΚΗ l: Η 1 1 1 1

Δίνεται τετράγωνο ΑΒΓ Δ και τα ισόπλευρα

τρίγωνα ΑΒΕ και ΒΓΖ από τα οποία το ένα βρί­

σκεται μέσα στο τετράγωνο και το άλλο έξω απ'

αυτό. Αφού υπολογίσετε τις γωνίες των τριγώ­

νων ΑΔΕ, ΒΕΓ και ΒΕΖ, να αποδείξετε ότι τα

σημεία Δ, Ε και Ζ είναι συνευθειακά. Α Π ΟΛΕ Ι Ξ Η

Για τις γωνίες του τριγώνου ΑΔΕ έχουμε : Λ Αι = 90° - 60° = 30° αφού το τρίγωνο ΑΕΒ

είναι ισοσκελές. λ

2 600 ι

Β

Γ

ΕΥΚΛΕΙΔΗΣ Β ' 66 τ.2/30

Page 33: Ευκλειδης Β 66

Μαθη ματικά για την Α ' Λυκείου

Είναι ΑΔ = ΑΕ οπότε το τρίγωνο ΑΔΕ είναι επί-Λ Λ

σης ισοσκελές, επομένως έχουμε: Δ 1 = Ε 1 = 1 80° - 30° = 750

2 Για τις γωνίες του τριγώνου ΒΕΓ έχουμε ανά­

λογα Λ Λ Λ Β1 = 30° και Γ1 = ΒΕΓ = 75°

Για τις γωνίες του τριγώνου ΒΕΖ έχουμε: Λ Λ Λ

ΕΒΖ = Β1 + Bz = 30° + 60° = 90°. Επίσης το τρί-γωνο ΒΕΖ είναι ισοσκελές γιατί ΒΕ = ΑΒ = ΒΖ

άρα EJ = ΖΙ = 1 80ο - 90ο = 45ο

2 Λ

Επιπλέον είναι Ez = 60° γιατί το τρίγωνο ΑΕΒ είναι ισόπλευρο, οπότε :

Λ Λ Λ Ε1 + Ε2 + Ε3 = 75° + 60° + 45° = 1 80° , άρα τα σημεία Δ, Ε και Ζ είναι συνευθειακά.

Π ΡΟ Τ Ε Ι :\ Ο :\1 [ � Εl: ΑΣ Κ Η Σ Ε Ι Σ Λ

I . Δίνεται ορθογώνιο τρίγωνο ΑΒΓ ( Α =90°) και ΑΔ το ύψος του . Από σημείο Ε της ΑΒ φέρ­νουμε παράλληλη προς την ΒΓ που τέμνει το ύψος ΑΔ στο Ζ. Στην συνέχεια φέρνουμε την κάθετη στην ΖΓ στο σημείο Ζ που τέμνει την ΑΒ στο Η. Να αποδείξετε ότι ΑΗ = ΒΕ.

2. Δίνεται τρίγωνο ΑΒΓ, εξωτερικά του τριγώνου κατασκευάζουμε τα τετράγωνα ΑΒΘΖ και ΑΓΔΕ. Φέρνουμε ΓΚ .l ΒΔ και ΒΛ .l ΓΖ. Να αποδείξετε ότι: α) Το ύψος ΑΗ του τριγώνου ΑΒΓ και οι ΓΚ,

ΒΛ διέρχονται από το ίδιο σημείο . β) Το ύψος ΑΗ διέρχεται από το μέσο Ν της

ΘΕ. γ) Αν ΑΜ είναι η διάμεσος του τριγώνου

ΑΒΓ, τότε ΑΜ = 2ΕΘ. 3. Δίνεται τρίγωνο ΑΒΓ και ΑΗ 1 , ΒΗ2, ΓΗ3 τα

ύψη του που τέμνονται στο Η. Α ν Μ το μέσο της ΒΓ και Λ το μέσο της Η2Η3 , να δείξετε ότι ΜΑ .l HzH3 .

4 . Δίνονται τέσσερα συνευθειακά σημεία Α, Β, Γ και Δ, τέτοια ώστε ΑΒ = ΒΓ = Γ Δ. Από τα Β και Γ φέρνουμε στο ίδιο ημιεπίπεδο παράλλη­λα τμήματα ΒΖ και ΓΕ ώστε ΒΖ=ΓΕ=2ΒΓ. Να

αποδείξετε ότι ΑΕ.lΔΖ. 5 . Δίνεται τραπέζιο ΑΒΓΔ (ΑΒ // ΓΔ) με

Λ Λ Λ Λ Α = Δ =90°, ΔΓ = 2ΑΒ και Β = 3 Γ . Φέρνουμε ΒΕ .l ΔΓ που τέμνει την διαγώνιο ΑΓ στο Μ. Φέρνουμε την ΑΕ που τέμνει την άλλη διαγώ­νιο ΒΔ στο Ν. Να δείξετε ότι: α) Το τετρά­πλευρο ΑΒΓΕ είναι παραλληλόγραμμο . β) ΑΕ .l ΒΔ. γ) ΜΝ = 1 /4 ·ΓΔ.

6 . Δίνεται τραπέζιο ΑΒΓΔ (ΑΒ // ΓΔ) με ΓΔ = 2ΑΒ, και οι μη παράλληλες πλευρές του τέ­μνονται στο σημείο Ο. Αν Κ, Λ είναι τα μέσα των διαγωνίων του ΒΔ και ΑΓ αντίστοιχα και Μ, Ν τα μέσα των ΟΑ και ΟΒ αντίστοιχα, να δείξετε ότι το τετράπλευρο ΜΝΛΚ είναι πα­ραλληλόγραμμο.

7 . Δίνεται τραπέζιο ΑΒΓΔ (ΑΒ // ΓΔ) με ΑΔ = ΑΒ + Γ Δ και Μ το μέσο της ΒΓ. Η ευθεία ΑΜ τέμνει την Γ Δ στο σημείο Ε και η ΔΜ τέμνει την ΑΒ στο σημείο Ζ. Να δείξετε ότι το τε­τράπλευρο ΑΔΕΖ είναι ρόμβος.

8. Σε ισοσκελές τρίγωνο ΑΒΓ (ΑΒ = ΑΓ) οι με­σοκάθετοι των πλευρών του ΑΓ και ΒΓ τέμνο­νται στο σημείο Ο. Θεωρούμε το συμμετρικό σημείο Μ του Ο ως προς την πλευρά ΑΓ και από στο Μ φέρνουμε την παράλληλη προς την πλευρά του τριγώνου ΑΒ που τέμνει την ΒΓ

Λ στο σημείο Δ. Να δείξετε ότι ΜΟΔ = 90°.

Λ 9 . Σε ορθογώνιο τρίγωνο ΑΒΓ ( Α = 90°) είναι

Λ Λ Γ = 5 Β . Α ν ΑΔ το ύψος του τριγώνου, να δεί-ξετε ότι ΒΓ = 4ΑΔ.

1 0 . Δίνεται ρόμβος ΑΒΓΔ. Αν Ε, Ζ σημεία των διαγωνίων του ΑΓ και ΒΔ αντίστοιχα, να δεί­ξετε ότι οι ευθείες ΕΒ, ΕΔ, ΖΑ και ΖΓ τεμνό­μενες σχηματίζουν κυρτό τετράπλευρο στο οποίο οι απέναντι γωνίες του είναι παραπλη-ρωματικές.

Σ H VI E I Ωl: H : Οι προτεινόμενες ασκήσεις θα λυθούν στο τεύχος 68 .

ΕΥΚΛΕΙΔΗΣ Β ' 66 τ.2/3 1

Page 34: Ευκλειδης Β 66

Μαθηματικά για την Α ' Λυκείου

ΕΠΙΣΤΗΜΟΝΙΚΉ ΕΠΙΤΡΟΠΉ 24ου ΠΑΝΕΛΛΉΝΙΟΥ ΣΥΝΕΔΡΙΟΥ ΕΜΕ ΚΟΖΑΝΗΣ

Δημοσιεύουμε τον πλήρη κατάλογο των μελών της επιστημονικής επιτροπής του συνε­δρίου της ΕΜΕ στην Κοζάνη, Με την ευκαιρία αυτή τους ευχαριστούμε για την πολύτιμη βοή­θεια που πρόσφεραν στην καλή οργάνωση του συνεδρίου .

ΠΑΠΑΣΤΑΥΡΙΔΗΣ ΣΤΑΥΡΟΣ, ΚΑΘΗΓΗΤΉΣ ΠΑΝΕΠΙΣΤΉΜΙΟΥ ΑΘΉΝΩΝ, Αν. ΠΡΟΕΔΡΟΣ ΕΠΙΣΤΗΜΟΝΙΚΗΣ ΕΠΠΡΟΠΗΣ ΚΑΛΟΓΕΡΟΠΟΥ ΛΟΣ ΓΡΗΓΟΡΙΟΣ, ΚΑΘΉΓΗΤΗΣ ΠΑΝΕΠΙΣΤΉΜΙΟΥ ΑΘΗΝΩΝ

ΑΝΔΡΕΑΔΑΚΗΣ ΣΤΥΛΙΑΝΟΣ, (ομ.) ΚΑΘΗΓΗΤΗΣ ΠΑΝΕΠΙΣΤΉΜΙΟΥ ΑΘΉΝΩΝ ΔΗΜΑΚΟΣ ΓΕΩΡΓΙΟΣ, ΕΠ. ΚΑΘΉΓΗΤΗΣ ΠΑΝΕΠΙΣΤΉΜΙΟΥ ΑΘΗΝΩΝ ΚΑΛΑΒΑΣΗΣ ΦΡΑΓΚΙΣΚΟΣ, ΚΟΣΜΗΤΟΡΑΣ ΑΝΘΡΩΠΙΣΠΚΩΝ ΕΠΙΣΤΉΜΩΝ ΠΑΝΕΠΙΣΤΉΜΙΟΥ ΑΙ­ΓΑΙΟΥ ΚΡΗτΙΚΟΣ ΕΜΜΑΝΟΥΉΛ, Δρ ΟΙΚΟΝΟΜΙΚΟΥ ΠΑΝΕΠΙΣΤΉΜΙΟΥ ΑΘΉΝΩΝ ΠΑΠΑΝΤΩΝΙΟΥ ΒΑΣΙΛΕΙΟΣ, ΚΑΘΗΓΗΤΉΣ ΠΑΝΕΠΙΣΤΉΜΙΟΥ ΠΑ ΤΡΩΝ ΧΡΙΣΤΟΠΟΥ ΛΟΣ ΠΑΝΑΓΙΩΤΗΣ, ΚΑΘΗΓΗΤΗΣ ΔΕΥΤΕΡΟΒΑΘΜΙΑΣ ΕΚΠΑΙΔΕΥΣΉΣ ΧΡΥΣΟΒΕΡΓΗΣ ΜΙΧΑΛΗΣ, Δρ ΜΑΘΗΜΑΠΚΟΣ, ΣΧΟΛΙΚΟΣ ΣΥΜΒΟΥ ΛΟΣ ΜΑΘΗΜΑ ΠΚΩΝ ΑΡΜΟΔΙΟΣ ΙΣΤΟΣΕΛΙΔΑΣ ΕΠΙΣΤΗΜΟΝΙΚΗΣ ΕΠΠΡΟΠΗΣ ΖΕΡΒΑΣ ΔΗΜΗΤΡΙΟΣ, ΚΑΘΉΓΗΤΉΣ ΔΕΥΤΕΡΟΒΑΘΜΙΑΣ ΕΚΠΑΙΔΕΥΣΗΣ

'\ ! Ι; : 'U � ι: ι, :: [� "Π � :'Ι Π U\ \ fl< R ii � ir: Π n �)O Π [ � � � , : i ! Π.: Τ Η \i Ο :\ ϊ ;·-. ι ι.:. Ε Π Ι "Π }Ο Ι R Ι-η.: ΑΓΓΕΛΟΠΟΥ ΛΟΣ ΕΥΓΕΝΙΟΣ, ΚΑΘΗΓΗΤΗΣ ΕΜΠ ΑΛΕΞΑΝΔΡΗΣ ΝΙΚΟΛΑΟΣ, ΚΑΘΗΓΗΤΗΣ ΠΑΝΕΠΙΣΤΉΜΙΟΥ ΠΕΙΡΑΙΩΣ ΑΝΤΩΝΙΑΔΗΣ ΙΩΑΝΝΗΣ, ΚΑΘΗΓΉΤΗΣ ΠΑΝΕΠΙΣΤΉΜΙΟΥ ΚΡΗΤΗΣ ΑΡΒΑΝΠΟΓΕΩΡΓΟΣ ΑΝΔΡΕΑΣ, ΛΕΚΤΩΡ ΠΑΝΕΠΙΣΤΉΜΙΟΥ ΠΑ ΤΡΩΝ Α ΥΓΕΡΙΝΟΣ ΕΥΓΕΝΙΟΣ, ΚΑΘΉΓΗΤΉΣ ΠΑΝΕΠΙΣΤΉΜΙΟΥ ΑΙΓ ΑΙΟΥ Α ΥΓΟΛΟΥΠΗΣ ΣΤΑΥΡΟΣ, ΚΑΘΗΓΗΤΗΣ ΠΑΝΕΠΙΣΤΉΜΙΟΥ ΘΕΣΣΑΛΟΝΙΚΉΣ ΒΕΡΥΚΙΟΣ ΠΕΤΡΟΣ, ΣΧΟΛΙΚΟΣ ΣΥΜΒΟΥ ΛΟΣ ΜΑΘΉΜΑ ΠΚΩΝ ΒΛΑΜΟΣ ΠΑΝΑΓΙΩΤΗΣ, ΕΠ. ΚΑΘΗΓΗΤΗΣ ΙΟΝΙΟΥ ΠΑΝΕΠΙΣΤΉΜΙΟΥ ΒΟΓΙΑΤΖΟΓ ΛΟΥ ΣΩΤΗΡΙΟΣ, ΚΑΘΗΓΗΤΗΣ ΔΕΥΤΕΡΟΒΑΘΜΙΑΣ ΕΚΠΑΙΔΕΥΣΉΣ MSc. Γ ΑΒΡΙΛΗΣ ΚΩΝΣΤ ΑΝτΙΝΟΣ, ΚΑΘΗΓΗΤΉΣ ΔΕΥΤΕΡΟΒΑΘΜΙΑΣ ΕΚΠΑΙΔΕΥΣΉΣ Γ ΑΓ ΑΤΣΗΣ ΑΘΑΝΑΣΙΟΣ, ΚΟΣΜΗΤΟΡΑΣ ΠΑΝΕΠΙΣΤΉΜΙΟΥ ΚΥΠΡΟΥ ΓΙΑΛΟΥΡΗΣ ΚΩΝΣΤ ΑΝτΙΝΟΣ, ΕΠ. ΚΑΘΉΓΗΤΗΣ ΓΕΩΠΟΝΙΚΟΥ ΠΑΝΕΠΙΣΤΉΜΙΟΥ ΔΑΜΙΑΝΟΥ ΧΑΡΑΛΑΜΠΟΣ, ΑΝ. ΚΑΘΉΓΗΤΗΣ ΠΑΝΕΠΙΣΤΉΜΙΟΥ ΑΘΗΝΩΝ ΔΟΡΤΣΙΟΣ ΚΩΝΣΤΑΝτΙΝΟΣ, ΣΧΟΛΙΚΟΣ ΣΥΜΒΟΥ ΛΟΣ ΜΑΘΗΜΑ ΠΚΩΝ ΔΟΥΓ ΑΛΗΣ ΒΑΣΙΛΕΙΟΣ, ΚΑΘΗΓΗΤΗΣ ΠΑΝΕΠΙΣΤΉΜΙΟΥ ΑΘΗΝΩΝ ΔΡΟΣΟΣ ΚΩΣΤΑΝτΙΝΟΣ, ΚΑΘΗΓΗΤΗΣ ΠΑΝΕΠΙΣΤΉΜΙΟΥ ΠΑ ΤΡΩΝ ΖΑΧΑΡΙΑΔΗΣ ΘΕΟΔΟΣΙΟΣ, ΑΝ. ΚΑΘΗΓΉΤΗΣ ΠΑΝΕΠΙΣΤΉΜΙΟΥ ΑΘΗΝΩΝ ΘΩΜΑΙΔΗΣ ΙΩΑΝΝΗΣ, Δρ ΜΑΘΗΜΑΠΚΟΣ, ΣΧΟΛΙΚΟΣ ΣΥΜΒΟΥ ΛΟΣ ΜΑΘΉΜΑ ΠΚΩΝ ΚΑΛΔΡΥΜΙΔΟΥ MAPIA, ΑΝ. ΚΑΘΗΓΉΤΡΙΑ ΠΑΝΕΠΙΣΤΉΜΙΟΥ ΙΩΑΝΝΙΝΩΝ ΚΑΡΑΓΕΩΡΓΟΣ ΔΗΜΗΤΡΙΟΣ, ΕΠ. ΚΑΘΉΓΉΤΗΣ ΕΚΠΑ ΚΑΡΑΖΕΡΗΣ ΠΑΝΑΓΙΩΤΗΣ, ΕΠ. ΚΑΘΗΓΗΤΗΣ ΠΑΝΕΠΙΣΤΉΜΙΟΥ ΠΑ ΤΡΩΝ ΚΑΡΑΜΠΑΤΣΑΣ ΚΩΝΣΤΑΝτΙΝΟΣ, ΚΑΘΉΓΗΤΗΣ ΔΕΥΤΕΡΟΒΑΘΜΙΑΣ ΕΚΠΑΙΔΕΥΣΉΣ DEA ΚΑΡΚΟΥ ΛΙΑΣ ΓΕΩΡΓΙΟΣ, ΣΧΟΛΙΚΟΣ ΣΥΜΒΟΥ ΛΟΣ ΜΑΘΗΜΑ ΠΚΩΝ ΚΑΣΙΜΑ τΗ ΑΙΚΑ ΤΕΡΙΝΗ, ΠΑΡΕΔΡΟΣ ΠΙ ΚΑ ΤΣΑΡΓΥΡΗΣ ΒΑΣΙΛΕΙΟΣ, ΚΑΘΗΓΗΤΗΣ ΔΕΥΤΕΡΟΒΑΘΜΙΑΣ Msc ΚΑΦΟΥΣΗ ΣΟΝΙΑ, ΕΠ. ΚΑΘΗΓΉΤΡΙΑ ΠΑΝΕΠΙΣΤΉΜΙΟΥ ΑΙΓΑΙΟΥ ΚΟΛΕΖΑ ΕΥΓΕΝΙΑ, ΚΑΘΗΓΉΤΡΙΑ ΠΑΝΕΠΙΣΤΉΜΙΟΥ ΙΩΑΝΝΙΝΩΝ ΚΟΝΤΟΓΙΑΝΝΗΣ ΔΗΜΗΤΡΙΟΣ, ΣΥΜΒΟΥ ΛΟΣ ΠΙ ΚΟΥΡΚΟΥ ΛΟΣ ΜΙΧΑΗΛ, ΕΠ. ΚΑΘΗΓΗΤΗΣ ΠΑΝΕΠΙΣΤΉΜΙΟΥ ΚΡΗΤΗΣ ΚΡΑΒΑΡΠΗΣ ΔΗΜΗΤΡΙΟΣ, ΚΑΘΗΓΗΤΗΣ ΕΜΠ ΛΑΠΠΑΣ ΔΙΟΝΥΣΙΟΣ, ΑΝ. ΚΑΘΗΓΗΤΗΣ ΠΑΝΕΠΙΣΤΉΜΙΟΥ ΑΘΗΝΩΝ ΛΕΜΟΝΙΔΗΣ ΧΑΡΑΛΑΜΠΟΣ, ΚΑΘΗΓΉΤΗΣ ΠΑΝΕΠΙΣΤΉΜΙΟΥ Δ ΥΠΚΗΣ ΜΑΚΕΔΟΝΙΑΣ

ΕΥΚΛΕΙΔΗΣ Β. 66 τ.2/32

Page 35: Ευκλειδης Β 66

Μαθηματικά για την Α ' Λυκείου

ΜΑΚΡΗ ΕΥΦΡΟΣΥΝΗ, ΑΝ. ΚΑΘΗΓΉΤΡΙΑ ΠΑΝΕΠΙΣΤΉΜΙΟΥ ΠΑ ΤΡΩΝ ΜΑΜΩΝΑ IΩANNA-DOWNS, ΑΝ. ΚΑΘΗΓΉΤΡΙΑ ΠΑΝΕΠΙΣΤΉΜΙΟΥ ΠΑ ΤΡΩΝ ΜΑΡΟΥΛΑΣ ΙΩΑΝΝΗΣ, ΚΑΘΗΓΗΤΗΣ ΕΜΠ ΜΑ ΥΡΟΓΙΑΝΝΗΣ ΝΙΚ Ο Σ, Δρ ΜΑΘΗΜΑ ΠΚΩΝ, ΕΥ ΑΓΓΕΛΙΚΉ ΣΧΟΛΗ ΣΜΥΡΝΗΣ ΜΗΛΙΩΝΗΣ ΧΡΙΣΤΟΣ, ΣΧΟΛΙΚΟΣ ΣΥΜΒΟΥ ΛΟΣ ΜΑΘΗΜΑ ΠΚΩΝ ΜΠΑΡΑΛΟΣ ΓΕΩΡΓΙΟΣ, Δρ ΜΑΘΗΜΑ ΠΚΟΣ, ΣΧΟΛΙΚΟΣ ΣΥΜΒΟΥ ΛΟΣ ΜΑΘΗΜΑ ΠΚΩΝ ΜΠΕΤΣΑΚΟΣ ΔΗΜΗΤΡΙΟΣ, ΕΠ. ΚΑΘΗΓΗΤΗΣ ΠΑΝΕΠΙΣΤΉΜΙΟΥ ΘΕΣΣΑΛΟΝΙΚΗΣ ΜΠΙΣΜΠΑΣ ΑΝΤΩΝΙΟΣ, ΚΑΘΗΓΗΤΗΣ Α ΤΕΙ Δ. ΜΑΚΕΔΟΝΙΑΣ ΜΠΟΛΗΣ ΘΕΟΔΩΡΟΣ, ΚΑΘΗΓΗΤΗΣ ΠΑΝΕΠΙΣΤΉΜΙΟΥ ΙΩΑΝΝΙΝΩΝ ΜΩΥΣΙΑΔΗΣ ΧΡΟΝΗΣ, ΚΑΘΗΓΗΤΗΣ ΠΑΝΕΠΙΣΤΉΜΙΟΥ ΘΕΣΣΑΛΟΝΙΚΗΣ ΝΕΣΤΟΡΙΔΗΣ ΒΑΣΙΛΕΙΟΣ, ΚΑΘΗΓΗΤΗΣ ΠΑΝΕΠΙΣΤΉΜΙΟΥ ΑΘΗΝΩΝ ΝΙΚΟΛΑΚΑΚΗ MAPIA, ΕΠ.ΚΑΘΗΓΗΤΡΙΑ ΠΑΝΕΠΙΣΤΉΜΙΟΥ ΘΕΣΣΑΛΙΑΣ ΝΙΚΟΛΟΥ ΔΑΚΗΣ ΕΜΜΑΝΟΥΉΛ, ΚΑΘΗΓΗΤΗΣ ΔΕΥΤΕΡΟΒΑΘΜΙΑΣ ΕΚΠΑΙΔΕΥΣΗΣ M.Ed.& M.Sc. ΝτΙΝΑΣ ΚΩΝΣΤ ΑΝτΙΝΟΣ, ΚΑΘΗΓΗΤΗΣ ΠΑΝΕΠΙΣΤΉΜΙΟΥ ΜΑΚΕΔΟΝΙΑΣ ΝΤΡΙΖΟΣ ΔΗΜΉΤΡΙΟ Σ, ΣΧΟΛΙΚΟΣ ΣΥΜΒΟΥ ΛΟΣ ΜΑΘΗΜΑ ΠΚΩΝ ΠΑΠΑΓΕΩΡΓΙΟΥ ΧΑΡΑΛΑΜΠΟΣ, ΚΑΘΗΓΗΤΗΣ ΠΑΝΕΠΙΣΤΉΜΙΟΥ ΑΘΗΝΩΝ ΠΑΠΑΔΟΠΕΤΡΑΚΗΣ ΕΥΤΥΧΙΟΣ, ΛΕΚΤΟΡΑΣ ΠΑΝΕΠΙΣΤΉΜΙΟΥ ΠΑ ΤΡΩΝ ΠΑΠΑΔΟΠΟΥΛΟΣ ΓΕΩΡΓΙΟΣ, ΕΠ. ΚΑΘΗΓΗΤΗΣ ΓΕΩΠΟΝΙΚΟΥ ΠΑΝΕΠΙΣΤΉΜΙΟΥ ΠΑΠΑΘΑΝΑΣΙΟΥ ΜΑΡΩ, ΕΠ. ΚΑΘΗΓΉΤΡΙΑ ΠΑΝΕΠΙΣΤΉΜΙΟ ΑΘΗΝΩΝ ΠΑΠΑΘΕΟΔΩΡΟΥ ΘΕΟΔΩΡΟΣ, ΚΑΘΗΓΗΤΗΣ ΠΑΝΕΠΙΣΤΉΜΙΟΥ ΠΑ ΤΡΩΝ ΠΑΠ ΙΣΤ ΑΣ ΑΘΑΝΑΣΙΟΣ, ΑΝ. ΚΑΘΗΓΗΤΗΣ ΠΑΝΕΠΙΣΤΉΜΙΟΥ ΘΕΣΣΑΛΟΝΙΚΗΣ ΠΕΤΡΑΚΗΣ ΑΝΔΡΕΑΣ, ΚΑΘΗΓΗΤΗΣ Α ΤΕΙ Δ. ΜΑΚΕΔΟΝΙΑΣ ΠΙΠΕΡΙΓΚΟΥ ΒΙΟΛΕΤΤΑ, ΛΕΚΤΩΡ ΠΑΝΕΠΙΣΤΉΜΙΟΥ ΠΑ ΤΡΩΝ ΠΝΕΥΜΑ τΙ ΚΟΣ ΣΠΥΡΙΔΩΝ, ΚΑΘΗΓΗΤΗΣ ΠΑΝΕΠΙΣΤΉΜΙΟΥ ΠΑ ΤΡΩΝ ΠΟΛ ΥΖΟΣ ΓΕΩΡΓΙΟΣ, ΠΑΡΕΔΡΟΣ ΠΙ ΠΟΤΑΡΗ ΔΕΣΠΟΙΝΑ, ΑΝ. ΚΑΘΗΓΉΤΡΙΑ ΠΑΝΕΠ. ΠΑ ΤΡΩΝ ΠΟΥ ΛΟΣ ΑΝΔΡΕΑΣ, Δρ ΜΑΘΗΜΑ ΠΚΩΝ, ΚΑΘΗΓΗΤΗΣ ΔΕΥΤΕΡΟΒΑΘΜΙΑΣ ΕΚΠΑΙΔΕΥΣΗΣ ΡΑΠΤΗΣ ΕΥ ΑΓΓΕΛΟΣ, ΑΝ. ΚΑΘΗΓΗΤΗΣ ΠΑΝΕΠΙΣΤΉΜΙΟΥ ΑΘΗΝΩΝ ΣΑΚΟΝΙΔΗΣ ΧΑΡΑΛΑΜΠΟΣ, ΑΝ. ΚΑΘΗΓΗΤΗΣ ΠΑΝΕΠΙΣΤΉΜΙΟΥ ΘΡΑΚΗΣ ΣΑΛΙΧΟΣ ΜΙΧΑΛΗΣ, Δρ ΜΑΘΗΜΑ ΠΚΟΣ, ΣΧΟΛΙΚΟΣ ΣΥΜΒΟΥ ΛΟΣ ΜΑΘΗΜΑ ΠΚΩΝ ΣΠΗΛΙΩΤΟΠΟΥ ΛΟΥ ΒΑΣΙΛΙΚΗ, ΚΑΘΗΓΉΤΡΙΑ Α.Σ.ΠΑΙ.Τ.Ε. ΣΠΥΡΟΥ ΠΑΝΑΓΙΩΤΗΣ, ΕΠ. ΚΑΘΗΓΗΤΗΣ ΠΑΝΕΠΙΣΤΉΜΙΟΥ ΑΘΗΝΩΝ ΣΤΑΘΟΠΟΥΛΟΥ ΧΑΡΑ, Εκ. ΕΠ. ΚΑΘΗΓΉΤΡΙΑ ΠΑΝΕΠΙΣΤΉΜΙΟΥ ΘΕΣΣΑΛΙΑΣ ΣΤΡΑ ΤΗΣ ΙΩΑΝΝΗΣ, ΚΑΘΗΓΗΤΗΣ ΠΑΝΕΠΙΣΤΉΜΙΟΥ ΑΘΗΝΩΝ ΤΖΑΝΑΚΗΣ ΚΩΝΣΤΑΝτΙΝΟΣ, ΚΑΘΗΓΗΤΗΣ ΠΑΝΕΠΙΣΤΉΜΙΟΥ ΚΡΗΤΗΣ ΚΡΗΤΗΣ, ΠΤΔΕ ΤΖΕΚΑΚΗ ΜΑΡΙΑΝΝΑ, ΑΝ. ΚΑΘΗΓΉΤΡΙΑ ΠΑΝΕΠΙΣΤΉΜΙΟΥ ΘΕΣΣΑΛΙΝΙΚΗΣ ΤΟΥΜΑΣΗΣ ΧΑΡΑΛΑΜΠΟΣ, Δρ ΜΑΘΗΜΑ ΠΚΟΣ, ΣΧΟΛΙΚΟΣ ΣΥΜΒΟΥ ΛΟΣ ΜΑΘΗΜΑ ΠΚΩΝ ΤΡΙΑΝΤ ΑΦΥ ΛΛΟΥ ΑΘΑΝΑΣΙΟΣ, ΣΧΟΛΙΚΟΣ ΣΥΜΒΟΥ ΛΟΣ ΜΑΘΗΜΑ ΠΚΩΝ ΤΣΙΚΟΠΟΥ ΛΟΥ ΣΤΑΜΑΤΟΥ ΛΑ, ΚΑΘΗΓΉΤΡΙΑ ΔΕΥΤΕΡΟΒΑΘΜΙΑΣ ΕΚΠΑΙΔΕΥΣΗΣ MSc. ΦΑΤΟΥΡΟΥ-ΤΡΕΣΟΥ ΕΥΑΓΓΕΛΙΑ, ΑΝ. ΚΑΘΗΓΉΤΡΙΑ ΠΑΝΕΠΙΣΤΉΜΙΟΥ ΘΕΣΣΑΛΙΝΙΚΗΣ ΦΕΛΛΟΥΡΗΣ ΑΝΑΡΓΥΡΟΣ, ΑΝ. ΚΑΘΗΓΗΤΗΣ ΕΜΠ ΦΕΡΕΝτΙΝΟΣ ΣΠΥΡΙΔΩΝ, ΣΧΟΛΙΚΟΣ ΣΥΜΒΟΥ ΛΟΣ ΜΑΘΗΜΑΠΚΩΝ ΧΑΡΑΛΑΜΠΙΔΗΣ ΧΑΡΑΛΑΜΠΟΣ, ΚΑΘΗΓΗΤΗΣ ΠΑΝΕΠΙΣΤΉΜΙΟΥ ΑΘΗΝΩΝ ΧΑΣΑΠΗΣ ΔΗΜΗΤΡΙΟΣ, ΕΠ. ΚΑΘΗΓΗΤΗΣ ΠΑΝΕΠΙΣΤΉΜΙΟΥ ΘΕΣΣΑΛΟΝΙΚΗΣ ΧΑΤΖΗΠΑΝΤΕΛΗΣ ΘΕΟΔΩΡΟΣ, ΚΑΘΗΓΗΤΗΣ ΠΑΝΕΠΙΣΤΉΜΙΟΥ ΘΕΣΣΑΛΟΝΙΚΗΣ ΧΡΟΝΑΚΗ ΑΝΝΑ, ΑΝΑΠΛΗΡΩΤΡΙΑ ΚΑΘΗΓΉΤΡΙΑ ΠΑΝΕΠ. ΘΕΣΣΑΛΙΑΣ ΧΡΥΣΑΦΙΝΟΥ OYPANIA, ΚΑΘΗΓΉΤΡΙΑ ΠΑΝΕΠΙΣΤΉΜΙΟΥ ΑΘΗΝΩΝ

Το Δ.Σ. της Ελληνικής Μαθη ματικής Εταιρείας

και η Συντακτική Επιτροπή του Ευκλείδη Β ' σας εύχονται χαρούμενο και δη μιουργικό το 2008

ΕΥΚΛΕΙΔΗΣ Β ' 6 6 τ.2/33

, , · ...o.=-=--____j

Page 36: Ευκλειδης Β 66

•-•ιιιι8-ιι8• ,., l'φl' W ��1'1 w•ιι Αιι••Ι•ιι

Άλγεβρα ΠΟΛ ΥΩΝΥΜΑ ΠΟΛ ΥΩΝΥΜΙΚΕΣ

Ε ΞΙΣΩΣΕΙΣ - ΕΞΙΣΩΣΕΙΣ ΠΟΥ ΑΝΆΓΟΝΤΑΙ ΣΕ ΠΟΛ ΥΩΝΥΜΙΚΕΣ

1 ) α) Να βρεθεί πολυώνυμο 2ου βαθμού τέτοιο ώστε P(x+l)=P(x)+x, για κάθε χ ε JR β) Πότε το παραπάνω πολυώνυμο δεν γίνε­ται γινόμενο παραγόντων με πραγματικούς συντελεστές;

Λύση α) Έστω Ρ(χ)=αχ2+βχ+γ, α, β, γ ε!R., τότε

Ρ(χ+ 1 )=Ρ(χ)+χ, για κάθε χ ε!R.<=:> <=:>α( χ + Ι )2+β(χ+ 1 )+γ=αχ2+βχ+γ+χ, για κάθε χ ε IR. <=:>αχ2+(2α+β)χ+( α+β+γ)=αχ2+(β+ 1 )χ +γ, για κάθε

1 {α = α α =

2

χε!R. <::::> 2α + Ji = � + 1 <=> β = -± α + β + j - j γ ε ΙR.

β) Πρέπει και αρκεί Δ<Ο. ( 1 )2 1 1 Αλλά: Δ < Ο <::::> -- - 4-γ < Ο <::::> - - 2γ < Ο <=> 2 2 4

1 8] <=> 2γ > - <=> γ > - . 4 8

2) Δίνονται τα πολυώνυμα.

Ρ (χ) = (-1-) χ3 + (συνα) χ2 - {συν2α) χ + 3π

ημα · 4

του Αποστόλη Κακαβά

Q (x) = (-1-) χ3 + ( ημα) χz + { ημzα ) χ + α, συν α

π α "* κπ και α * κπ + - , κ ε Ζ

2 Να βρείτε το βαθμό του πολυωνύμου P(x)+Q(x) για τις διάφορες τιμές του α.

Λύση

P (x ) + Q ( x ) = (-1- + -1 -) x3 + (ημα + συνα) χ2 + ημα συνα

+ (ημ2α - συν2α) χ + ( α + 3: ) =

= ( ημα + συνα ) χ3 + (ημα + συνα) χ2 + ημα · συνα

+ (ημ2α - συν2α ) χ + ( α + 3: ) • Παρατηρούμε ότι, με

π '71 ' α "* κπ και α "* κπ + - , κ ε IL..ι εχουμε 2

ημα+συνα*Ο<=>ημα*-συνα<::::>εφα*- 1<=>

<=:>εφα-;j; εφ (- : ) <=> I α "* κπ -�ι π π Αν α * κπ - - , κπ, κπ + - με κ ε Ζ 4 2

τότε το P(x)+Q(x) είναι 3ου βαθμού. • Εξάλλου ημα+συνα=Ο<=:>

εφα=-1 <=> I α= κπ -�' κ ε zι . Για να είναι δε-

ΕΥΚΛΕΙΔΗΣ Β' 66 τ.2/34

Page 37: Ευκλειδης Β 66

Μαθηματικά για την Β · Λυκείου

κτές αυτές οι τιμές πρέπει και αρκεί κπ-� -:ι. /.π και 4

π π . . Ι κπ-- -:ι. /.π+- , που ισχυουν, αφου κ - λ -:ι. - και 4 2 4

κ - λ -:ι. � 4

Άρα οι τιμές I α = κπ -� , κ ε Ζ I είναι δεκτές

Τότε 3π π 3π π Ρ(χ)+Q(χ)= α+- = κπ -- +- = κπ+- κ ε Ζ 4 4 4 2 '

και δεν είναι το μηδενικό πολυώνυμο για καμία τι-

. z ( . 1 μη του κε , αφου κ -:ι. -- ) . 2

Άρα είναι μηδενικού βαθμού.

3) Α ν το υπόλοιπο της διαίρεσης του πολυω­νύμου Ρ( χ) δια του χ2 -ρ2 είναι υ(χ)=αχ+β, α,β ε!R, τότε να βρεθεί το υπόλοιπο της διαί­ρεσης του Ρ(χ) : (χ-ρ)

Λύση : Η ταυτότητα της διαίρεσης Ρ(χ) : (χ2-ρ2) είναι Ρ(χ)=(χ2-ρ2)π(χ)+αχ+β ( 1 ) , όπου π(χ) το πηλί­κο της διαίρεσης Ρ(χ) : (χ2-ρ2) . Από την ( 1 ) για χ=ρ παίρνουμε Ρ(ρ )=(ρ2 -ρ2)π(ρ )+αρ+β=αρ+β. Άρα το υπόλοιπο της διαίρεσης

Ρ(χ) : (χ-ρ) είναι υ=Ρ(ρ)=αρ+β β ' τρόπος

Ρ(χ)=(χ-ρ )(χ+ρ )π(χ)+α(χ-ρ )+αρ+β= =(χ-ρ) [(χ+ρ)π(χ) +α]+αρ+β

Άρα έχουμε υπόλοιπο υ=αρ+β και πηλίκο Πι (χ)=(χ+ρ )π(χ)+α

4) Να βρεθεί το υπόλοιπο της διαίρεσης πολυ­ωνύμου Ρ(χ) : (χ2-ρ2), ρ ε!R, ρ-:ι.Ο.

Λί>ση Επειδή ο διαιρέτης είναι 2ου βαθμού το υπό­

λοιπο είναι της μορφής: υ(χ)=αχ+β α,β ε!R Η ταυτότητα της διαίρεσης Ρ(χ) : (χ2-ρ2) με πηλίκο π(χ) είναι:

Ρ( χ)= (χ2 -ρ2)π(χ)+ αχ+β Για χ=ρ από την ( 1 ) παίρνουμε :

Ρ(ρ )=(ρ2 -ρ2)π(ρ )+αρ+β=αρ+β (2) Ομοίως για χ= -ρ παίρνουμε:

Ρ(-ρ)= -αρ + β (3)

Οπότε : (2), (3) � β = Ρ (ρ ) + Ρ ( -ρ ) και 2

α = Ρ ( ρ ) - Ρ (-ρ) 2ρ

Άρα υ ( χ ) = Ρ ( ρ) - Ρ (-ρ ) χ + Ρ (ρ ) + Ρ (-ρ ) . 2ρ 2

Π αρατή ρηση : Ειδικά αν ρ=Ο και βαθΡ ( χ ) = k ::::: 2 , τότε

P ( x ) = Akxk + . . . + Α2χ2 + Α1 χ + Α0 με Ak -:ι. Ο , οπότε έχουμε : Ρ ( χ ) = χ 2 ( Ak χ k-2 + . . . + Az ) + Α, χ + Ασ . Άρα υ ( χ ) = Α1 χ + Α0 • Αν όμως k<2 ή Ρ(χ) το μηδενικό πολυώνυμο, τότε υ(χ)=Ρ(χ) .

5) Να λυθούν οι ανισώσεις:

α) (χ-2)2 (χ -1) ( 3 -χ)::; ο (1)

β) (x-2)2(x-l)(x-3)< Ο (2)

γ) (χ-2)3 (x - l)(x - 3) > Ο (3)

Λύση α) Βρίσκουμε το πρόσημο κάθε παράγοντα, εξε­τάζοντας αρχικά πότε είναι καθένας απ' αυτούς θετικός, δηλαδή λύνουμε αρχικά τις επιμέρους α-νισώσεις : ( χ - 2 )2 > Ο (i), χ - 1 > Ο (ίί) , 3 - χ > ο (iii) . (θα μπορούσαμε φυσικά να βρούμε αρχικά πότε οι παράγοντες αυτοί είναι αρνητικοί, δηλαδή να λύ­σουμε τις aντίστροφες ανισώσεις) . Έχουμε λοιπόν: (ί) � χ -::f:. 2 , (ίί) � χ Ε ( 1, +οο) , (iii) � χ Ε ( -οο, 3) . Απεικονίζουμε σε πίνακα τα συμπεράσματα αυτά απ ' όπου φαίνεται επιπλέον πότε οι παράγοντες αυτοί είναι αρνητικοί και πότε μηδενίζονται.

-00 2 3

Α=(χ-2)2 + + + +

Β=χ- 1 + + +

Γ=3-χ + + +

ΑΒΓ + +

Άρα:

( 1 ) � ΑΒΓ ::; Ο � χ Ε ( -οο, 1] υ [3, +οο ) υ {2} Β ' τρόπος :

( 1 ) � ( χ - 2 )2 ( χ - 1 ) ( χ - 3) � ο � χ = 2 ή

ΕΥΚΛΕΙΔΗΣ Β ' 66 τ.2/35

Page 38: Ευκλειδης Β 66

Μαθηματικά για την Β ' Λυκείου

(χ - 1 ) (χ - 3 ) � 0 � χ = 2 ή χ ε ( -οο, 1 ] υ [3, +οο ) � χ ε ( -οο, 1 ] υ [3, +οο ) υ { } Οι (2), (3) λύνονται ομοίως με πίνακα, αλλά και συντομότερα ως εξής: β) ( 2 ) � χ :;t: 2 και (χ - 1 ) (χ -3) < Ο � χ :;t: 2 και χ ε ( 1 , 3 ) � χ ε ( 1 , 2 ) υ ( 2, 3 ) . γ) Αφού οι (χ - 2 )3 και (χ - 2 ) είναι ομόσημοι,

θα έχουμε : ( 3 ) � (χ - 2 ) (χ - 1 ) (χ - 3 ) > Ο . Το­ποθετούμε τις ρίζες του 1 ου μέλους σε άξονα και αφού οι συντελεστές του χ είναι όλοι θετικοί τα πρόσημα του γινομένου όπως ξέρουμε εναλλάσο­νται με το (+) πάντα στο 1 ο δεξιό διάστημα.

j -00 2 3

+

Έτσι έχουμε : (3 ) � χ ε ( 1 , 2 ) υ (3, +οο ) . Στις ( 1 ) , (2) οι παράγοντες ήταν δύο, οπότε απαντήσα­με απ' ευθείας με βάση το πρόσημο τριωνύμου. l:ημε ίι•)ση : Παρατηρήσαμε ότι σε κάθε πολλαπλή ρίζα περιττής τάξεως του 1 ου μέλους αλλάζει το προσημό του, ενώ σε κάθε πολλαπλή ρίζα αρτίας τάξεως δεν αλλάζει το πρόσημο του 1 ου μέλους.

6) Ν α λυθεί η aνίσωση : (χ - 2) {-χ2 + 2χ - 1)

----;----'-------,---..,---'---7"" < ο (1) (χ - 1 ) ( χ2 - χ - 2 ) ( χ2 + χ + 2007)

λίJση Για να ορίζεται η ( 1) πρέπει και αρκεί (χ- 1 ) (χ2- χ-2) (x2+x+2007):;t:O, δηλαδή χ � { 1 , 2, - 1 } , αφού χ 2 + χ + 2007 > Ο , για κάθε χ ε JR (γιατί;) . Στο JR - {1 , 2, - 1} έχουμε:

(χ-2) ( χ2 -2χ+1) ( χ-1)2 ( 1) � > Ο� > Ο�

(χ-1)(χ-2)(χ+ 1) - (χ-1)(χ+ 1) -χ - 1

- � Ο � (χ - 1 ) (χ + 1 ) � 0 � χ + 1 � χ ε ( -οο, - 1 ] υ [ 1 , +οο ) � � χ ε ( -οο, - 1 ) υ ( 1 , 2 ) υ (2, +οο )

7) Δίνεται το πολυώνυμο

Ρ( χ) = ( συν2 �J χ4 + χ3 - ( συν2α) χ2 - ( συν2 �J χ

α) Να βρεθεί το αε [Ο,2π] ώστε το πολυώ­νυμο Ρ( χ) να έχει παράγοντα το χ+ 1

β) Για α=Ο να λυθεί η εξίσωση Ρ(χ)=Ο και η aνίσωση Ρ(χ):::; Ο

λίJση α) Το χ+ 1 είναι παράγοντας του Ρ(χ) <=>

z α z z α <::::>Ρ(- 1 )=0 <::::>συν - - 1 - συν α+συν - = 0 <::::> 2 2

<=> 2συν2 � - Ι - συν2α = Ο <=> συνα - συν2α = Ο <=> 2

<=> συν α ( 1 - συν α) = Ο <=> συν α = Ο ή συν α = Ι <=>

α = κπ + � , κ Ε Ζ ή α = 2κπ, κ Ε Ζ . 2

Εξετάζουμε για ποιες τιμές κ Ε Ζ ισχύει:

Ο ::::; κπ + � ::::; 2π ( 1 ) ή Ο ::::; 2κπ ::::; 2π (2) 2

Έχουμε: π 3π Ι 3 , ( 1 ) <=> -- ::::; κπ ::::; - <=> -- ::::; κ ::::; - <=> κ = Ο η κ = Ι 2 2 2 2

π Για κ=Ο έχουμε α = - και για κ= 1 έχουμε 2

3π α = -2

Εξάλλου ( 2) <=> Ο ::::; κ ::::; Ι <=> κ=Ο ή κ= 1

Για κ=Ο έχουμε α=Ο και για κ= Ι έχουμε α=2π.

Τελικά α Ε { 0,� , 32π , 2π}

β ) Για α=Ο έχουμε Ρ(χ)=χ4+χ3-χ2-χ = =χ4+χ3 -xz -χ=χ3(χ+ Ι )-χ( χ+ Ι )= =χ(χ+ 1 )(χ2 - 1 )=χ(χ+ Ι )2(χ- Ι ), οπότε Ρ(χ)=Ο<::::> Χ Ε {Ο, Ι ,- 1 } Εξάλλου : Ρ ( χ ) ::::; Ο <=> � χ = - 1 ή χ ( χ - 1 ) ::::; Ο � χ ε [Ο, 1 ] υ { - 1}

8) Δίνεται το πολυώνυμο Ρ(χ)=χ4+χ3 -3χ2 -αχ+β, α,β ΕIR. α) Να βρεθούν τα α,β ε ΙR ώστε το Ρ(χ) να

διαιρείται με την μεγαλύτερη δυνατή δύναμη του χ-1

β) Για τις τιμές των α και β που βρήκατε να λυθεί η εξίσωση Ρ(χ)=Ο

γ) Να λυθεί η aνίσωση Ρ(χ);:::Ο

, \ ίJση α ) Το (χ- Ι ) διαιρεί το Ρ(χ) <::::>Ρ( 1 )=0 . Με το σχή­μα Homer βρίσκω το πηλίκο και το υπόλοιπο της διαίρεσης Ρ( χ) : (χ- Ι )

ΕΥΚΛΕΙΔΗΣ Β ' 66 τ.2/36

Page 39: Ευκλειδης Β 66

Μαθη ματικά για την Β ' Λυκείου

-3 α 2 -1

β α- 1

2 -1 α- 1 α+β- 1 Άρα υ 1 = β + α - 1 και π 1 (χ ) = χ3 + 2χ2 - χ + (α - 1 )

Οπότε: Ρ ( 1 ) = Ο <=> α + β - 1 = Ο

ρ = 1

Το (χ- 1 )2 διαιρεί το Ρ(χ) <=> Ρ ( 1 ) = 0 και το (χ- 1 ) διαιρεί το π 1 (χ). Με το σχήμα Homer βρίσκω το πηλίκο και το υπόλοιπο της διαίρεσης

π 1 (χ) : (χ- 1 )

2 - 1 α- 1 ρ = 1 3 2

3 2 α+ 1 Άρα υ 2 = α + 1 και π ( χ ) = χ 2 + 3χ + 2 = (χ + 1 ) ( χ + 2 ) Οπότε: (χ- 1 )2 παράγοντας Ρ( χ) <=> υ, = Ο }

<=> β + α - 1 = Ο}<=>

α = - 1} υ2 = Ο α + 1 = Ο β = 2 β ) Για α=- 1 και β=2 έχουμε Ρ(χ)= χ4+χ3-3χ2-χ+2 =(χ+ 1 )(χ+2)(χ- 1 )2 , οπότε δεν διαιρείται με μεγα­λύτερη , από τη 211 δύναμη του (χ- 1 ) . Άρα: Ρ(χ)=Ο<::>χε { - 1 , -2, 1 }

γ) Ρ(χ)�Ο<=> χ= 1 ή (χ+ 1 )(χ+2)�0<=>χ ε( -oo,-2]u[- 1 , +οο)

9) Ν α λυθεί η εξίσωση :

.Jx2 + 8χ + 7 - .J.-x2_+

_8_x = 1 (1)

Λύση Πεδίο ορισμού της ( 1) είναι το

Α = { χ Ε ΙR Ι χ 2 + 8χ ;::: ο} θέτω x2+8x=y οπότε η επιλύουσα της ( 1) είναι η �y + 7 - JY = 1 (2)

Πεδίο ορισμού της (2) είναι το Β = [Ο, +οο ) Στο Β έχουμε: (2) <=> �y + 7 = 1 + -/Υ <=> (�y + 7 )2 = ( 1 + -/Υ)2 <=> y + 7 = 1 + 2-/Υ + y <=> 2.jY = 6 <=> JY = 3 <=> y = 9

Δεκτή τιμή αφού ανήκει στο Β. Για y=9 έχουμε : x2+8x=y<=> χ2+8χ=9<=>

χ2+8χ-9=0<=:> χ=--9 ή χ= 1

Οι τιμές αυτές προφανώς είναι δεκτές δηλαδή

ανήκουν στο Α αφού για αυτές ισχύει χ 2 + 8χ = 9 � ο

10) Δίνεται το πολυώνυμο χν+νχ+(ν-1), ν ε Ν, ν;:::2 α) Να βρεθεί το πηλίκο π(χ) της διαίρεσης

Ρ(χ) : (χ-1 )2

β) Να βρεθεί η τιμή του π(χ) για χ=1

, \ \>ση α) Ρ(χ)=χν+νχ+(ν- 1 )=χν-1-νχ+ν=χν-1-ν(χ- 1 )

=(χ- 1 )(χν- Ι+χν-2+ . . . +χ+ 1 )-ν(χ- 1 ) =(χ- 1 )(χν- Ι+χν-2+ . . . +χ 1+ 1-ν) =(χ- 1 ) [(χν- 1_ 1 )+(χν-2 -1 )+(xv-3 -1 )+ . . . +(χ2 - 1 )+(χ-1 )] =(x- 1 )[(x- 1 )(xv-2+xv-J+ . . . + 1 )+ +(χ-1 )(χν-3+χν-4+ . . . + 1 )+ . . . + (χ-1 )(χ+ 1 )+(χ- 1 )] =(χ- 1 )2 [(χν-2+χν-3+ . . . + 1 )+ +(χν-3+Χν-4+ . . . + 1 )+ . . . +(χ+ 1 )+ 1 ]

=(χ- 1 )2 [(ν- 1 )+(ν-2)χ+(ν-3)χ2+ . . . + 3χν-4+2χν-3+χν-2] άρα π(χ)=χν-2+2χν-3+3χν-4+ . . . +(ν-2)χ+(ν- 1 )

β) Έχουμε : π( 1 )= 1 +2+3+ . . . +(ν-2)+(ν- 1 ) ( 1 ) και π( l )=(ν- 1 )+(ν-2)+(ν-3)+ . . . +2+ 1 (2)

οπότε : ( 1 )+(2) �2π( l )= ν + ν + ν + . . . + ν + ν � (ν- Ι ) όροι

2π ( 1 ) = ν (ν - 1 ) � π ( 1 ) = -ν (

_,__ν

_-

--"-1 )

2

1 1) Δίνεται το πολυώνυμο Ρ(χ)=χν+2-(ν+2)χ +(ν+1) ν ε Ν*

α) Δείξτε ότι το (χ-1)2 είναι παράγοντας του Ρ(χ)

β) Διαπιστώστε ότι ο αριθμός 32ν+4-8ν-17 είναι πολλαπλάσιο του 64 .

ΛίJση α ) Ρ(χ)=χν+2-(ν+2)χ +(ν+ 1 )= =χν+2 -(ν+ 1 )χ-χ+(ν+ 1 )=χν+2 -χ-(ν+ 1 )χ+(ν+ 1 )= =χ(χν+ Ι_1 )-(ν+ 1 )(χ-1 )= =χ(χ-1 )(χν+χν- Ι+ . . . +χ+ 1 )-(ν+ 1 )(χ-1 )= =(χ- 1 ) [χ(χν+χν- 1+ . . . +χ+ 1 )-(ν+ 1 )]= =(χ- 1 ) [(χν+ Ι_ 1 )+(χν- 1 )+ . . . +(χ2- 1 )+(χ-1 )]= =(χ- 1 ) [(χ-1 )(χν+χν-Ι+ . . . +χ+ 1 )+(χ- 1 )+ +(χ-1 )(χν- Ι+χν-2 1+ . . . +χ+ 1 )+ . . . +(χ- 1 )(χ+ 1 )(χ- 1 )]= =( x-1)2 Q χν +χν-1 + . . . +χ+1)+( χν-1 +χν-2 + . . . +χ+1)+ . . . +

+ . . . +(χ+ 1 )+ 1 ]=(χ- 1 )2 ·π(χ)

β) Δείξαμε ότι: Ρ(χ)=χν+2-(ν+2)χ +(ν+ 1 )=(χ- 1 )2 ·π(χ), για κάθε χ ε!R

Οπότε : Ρ(9)=9ν+2-(ν+2)9 +(ν+ 1 )=82 ·π(8)� (3 2) ν+Ζ -9ν-1 8+ν+ 1 =64·π(8) � 32ν+4-8ν-1 7=πολ.64

ΕΥΚΛΕΙΔΗΣ Β ' 66 τ.2/37

Page 40: Ευκλειδης Β 66

Μαθηματικά για την Β ' Λυκείου

1 2) Αν P(x+l)=3x2-x, για κάθε x e iR., τότε: α) Να βρεθεί το πολυώνυμο Ρ(χ) β) Να βρεθεί το πολυώνυμο Q(x) ώστε

P(x).Q(x)=3x3-10x2+1 1 x-4, για κάθε x e iR. (1) γ) Να λυθεί η aνίσωση Ρ( I χ I ).Q(x)<O Λύση

α) Έχουμε : χ+ 1 =κ<=:>χ=κ-1 . Άρα Ρ(χ+ 1 )=3χ2-χ, για κάθε x εiR. <=:>Ρ(κ)=3 (κ-1 )2-(κ- 1 ) , για κάθε κε1R<=:>Ρ(κ)=3κ2-7κ+4, για κάθε κε!R, άρα Ρ(χ)=3χ2-7χ+4

-00 -4/3 - I I 4/3 +οο

Α=Ιχ l- 1 + + l + �. + +

Β=3 Ιχ Ι -4 - J; - - - +

Γ=χ- 1 - - - I' + +

ΑΒΓ - I + I - Ί - ? +

β) Το Q(x) είναι 1 ου βαθμού, αφού το Ρ( χ) είναι 2ου βαθμού .

Έστω Q(χ)=αχ+β. Τότε : Ρ(χ) 'Q(χ)=(αχ+β)(3χ2 -7χ+4)= =3αχ3 -7 αχ2+4αχ+ 3 βχ2 -7βχ+4β= =3αχ3+(3β-7α)χ2+(4α-7β)χ+4β

3α = 3 3β - 7α = - 1 0 ........_ {α = 1

Άρα: ( 1 ) <=> .......-4α - 7β = 1 1 β = - 1 4β = -4

γ) Έχουμε : Ι x l=y

Ρ( I χ I )=3 1 χ 1 2-7 1 χ I +4=0 = 3y2-7y+4=

= 3 ( y - 1 ) ( y -�) = 3 ( l x l - 1 ) ( l x l - �) . αφού το

τριώνυμο 3y2-7y+4 έχει ρίζες y1 = 1, y2 = 4 3

Άρα:Ρ( I χ I )(χ- 1 )<0( 1 ) <=> ( 1�-1)�� �)χ-1) <0 Θεωρώ τις επιμέρους ανισώσεις Ιχ Ι- 1 >0 (2)

4 l x l - ] > 0 (3), χ- 1 >0 (4) .

Έχουμε: (2) � χ ε ( -οο, - 1 ) υ ( 1 , +οο)

( 3 ) � χ ε ( -οο, -� )υ ( � , +οο) ( 4) � χ ε ( 1, +οο)

Απεικονίζουμε τα συμπεράσματα αυτά σε πίνακα Άρα:

( 1) � ΑΒΓ < Ο � χ ε ( --αι,-�)υ (-1, 1)υ ( ι,�)

ΓΕΩΜΕΤΡΙΑ ΜΕΤΡΙΚΕΣ ΣΧΕΣΕΙΣ

ΣΤΟΝ ΚΥΚΛΟ - ΕΜΒΑΔΑ

Επιμέλεια : ΓΙΩΡΓΟΣ ΤΡΙΑΝΤΟΣ

Στη δέσμη των δεκατριών ασκήσεων που ακολουθεί περιλαμβάνονται θέματα που αφορούν στη δύ­ναμη σημείου ως προς κύκλο και στα εμβαδά ευθυγράμμων σχημάτων, χωρίς όμως να διαχωρίζονται α­πόλυτα κατά ενότητα. Αντίθετα, εμφανίζονται συνδυασμένα με αρκετά στοιχεία προηγουμένων ενοτή­των της Γεωμετρίας όπως η ομοιότητα, οι μετρικές σχέσεις σε τρίγωνο κ.α.

Έτσι, ενώ η άσκηση (1) είναι μία απλή εφαρμογή στο θεώρημα των τεμνομένων χορδών, η άσκηση (2) , μαζί με τη δύναμη σημείου συνδυάζει τον νόμο των συνημιτόνων και άλλους τύπους για το εμβαδόν τριγώνου. Η ομοιότητα τριγώνων εμφανίζεται στις ασκήσεις (4) ,(8),(13). Ο υπολογισμός τμημάτων διαίρεσης πλευράς τριγώνου από εσωτερική διχοτόμο στην άσκηση (1 1) . Στην άσκηση (5) θεωρούνται γνωστά τα αποτελέσματα του Θεωρήματος Euler για τυχαίο τετράπλευρο και για παραλληλόγραμμο, ενώ η άσκηση (12) είναι εφαρμογή του τύπου του εμβαδού τετραπλεύρου με κάθετες διαγωνίους. Τέλος στις ασκήσεις (7),(8),(9) αξιοποιείται η ισοδυναμία τριγώνων που χωρίζει μία διαγώνιος παραλληλογράμμου το παραλληλόγραμμο, μία διάμεσος τριγώνου το τρίγωνο καθώς και των τριγώνων που έχουν κοινή βάση και ίσα ύψη .

ΑΣΚ Η Σ Η ( 1 ) : Δίνεται ημικύκλιο διαμέτρου ΑΒ = 2R και

δύο τυχαίες χορδές ΑΓ, ΒΔ του ημικυκλίου που

τέμνονται σε σημείο Ρ. Συμβολίζουμε με D1 τη δύναμη του σημείου Α ως πρός τον περιγεγραμ­μένο κύκλο του τριγώνου ΒΡΓ και με D 2 τη

ΕΥΚΛΕΙΔΗΣ Β' 66 τ.2/38

Page 41: Ευκλειδης Β 66

Μαθηματικά για την Β ' Λυκείου

δύναμη του σημείου Β ως προς τον περιγεγραμ­μένο κύκλο του τριγώνου Ρ ΑΔ . Να αποδειχθεί ότι: D1 + D2 = 4R2 • Ζ

Δ

ο

/

λπ{ιδε ιξη : Έστω Η η προβολή του σημείου Ρ πάνω

Λ Λ

στην ΑΒ . Επειδή Ρ ΔΑ+ ΡΗΑ = π και Λ Λ

ΡΓΒ+ ΡΗΒ = π οι περιγεγραμμένοι κύκλοι των τριγώνων ΑΔΡ και ΒΓΡ έχουν διαμέτρους ΑΡ και ΒΡ αντιστοίχως και διέρχονται από το σημείο Η .

Έχουμε : D 1 = ΑΡ · ΑΓ = ΑΗ · ΑΒ (1) και D2 = ΒΡ · ΒΔ = ΒΗ · ΒΑ (2) .

με : Με πρόσθεση των σχέσεων (1) , (2) παίρνου-

D1 + D2 = ΑΡ · ΑΓ + ΒΡ · ΒΔ = = ΑΗ · ΑΒ + ΒΗ · ΒΑ = = ΑΒ(ΑΗ + ΒΗ) = ΑΒ · ΑΒ = = ΑΒ2 = 4R2

ΑΣ Κ Η Σ Η ( 2 ) : Δίνεται οξυγώνιο τρίγωνο ΑΒΓ εγγεγραμ­

μένο σε κύκλο (O, R) . Θεωρούμε τα σημεία Δ, Ε, Ζ συμμετρικά των Α, Β, Γ ως προς τις πλευρές ΒΓ, ΑΓ, ΑΒ αντιστοίχως. Ονομάζουμε D1 , D2 , D3 τις δυνάμεις των σημείων Δ, Ε, Ζ ως

προς τον κύκλο (O, R) αντίστοιχως. Να αποδει­

χθεί ότι ισχύει 1 . D1 + 02 + 03 = α 2 + β 2 + γ2 ,

όπου α, β ,γ τα μήκη των πλευρών του τριγώνου ΑΒΓ . 2. α · ΗΕ · ΗΖ + β · ΗΔ · ΗΖ + γ · ΗΔ · ΗΕ = 4R(ΔΕΖ) , όπου Η το ορθόκεντρο του τριγώνου ΑΒΓ.

Δ Απ{Jδ�: ιξη : 1 ) Σύμφωνα με την υπόθεση η πλευρά ΒΓ

είναι μεσοκάθετος του τμήματος ΑΔ , του οποίου έστω Ρ το μέσον. Άρα D =0Δ2 - R2 = ΟΔ2 - ΟΑ2 = 2ΑΔ · ΚΡ όπου ι ' ΟΚ l_ ΑΔ (2° θεώρημα διαμέσων στο τρίγωνο ΟΑΔ ). Αν Μ είναι το μέσον της πλευράς ΒΓ τότε είναι ΟΜ l_ ΒΓ και ΟΜ=ΚΡ

Οπότε : D 1 = 2 · 2ΑΡ · ΟΜ = 4 · υ α · ΟΜ ( 1 ) Επειδή είναι 2Ε Λ υα = -, ΟΜ = RσυνΜΟΓ = RσυνΑ , όπου Ε

α το εμβαδόν του τριγώνου ΑΒΓ , η σχέση (1) γράφεται:

4ER D 1 = 2 · -- συνΑ (2) . α

β2 + 2 _ α2 Όμως, ισχύουν αβγ=4ΕR και συνΑ = γ

2βγ (από τον νόμο των συνημιτόνων) . Οπότε η (2) γράφεται :

β β2 2 2 Dι = 2 · α Υ . + γ - α = βz + γz - αz (3) .

α 2βγ Κυκλικά έχουμε :

D2 = γ 2 + α z - β 2 ' D3 = α z + β2 - γ 2 . Με πρόσθεση κατά μέλη παίρνουμε:

D ι + D 2 + D3 = α z + β z + γ z .

ri ιφατ1) pηση : Γνωρίζοντας ότι: ΟΜ = ΗΑ βρί-2

σκουμε κ .λ. π. , οπότε

ΕΥΚΛΕΙΔΗΣ Β ' 66 τ.2/39

Page 42: Ευκλειδης Β 66

Μαθηματικά για την Β ' Λυκείου

α2 + β2 + γ2 υα · ΗΑ + υβ · ΗΒ + υγ · ΗΓ = ----=--2-�

Να δοθεί και δεύτερος τρόπος απόδειξης της τελευταίας σχέσης, ανεξάρτητος.

Λ Λ

2) Επειδή ΖΗΕ+ Α = π (πλευρές κάθετες μία προς μία) από το θεώρημα των εμβαδών έχουμε (ΗΖΕ) = ΗΖ · ΗΕ => (ΗΖΕ) = ΗΖ · ΗΕ (4) . (ΑΒΓ) ΑΒ · ΑΓ (ΑΒΓ) βγ

, , (ΗΖΔ) ΗΖ · ΗΔ Ομοιως εχουμε : = (5) (ΑΒΓ) αγ και (ΗΔΕ) = ΗΔ · ΗΕ (6) . (ΑΒΓ) βα

�ε πρόσθεση κατά μέλη των σχέσεων (4),(5) ,(6) παίρνουμε (ΗΖΕ) (ΗΖΔ) (ΗΔΕ) ....:....______.:... + + = (ΑΒΓ) (ΑΒΓ) (ΑΒΓ)

ΗΖ · ΗΕ ΗΖ · ΗΔ ΗΔ · ΗΕ --- + + ----

βγ αγ βα (ΗΖΕ) + (ΗΖΔ) + (ΗΔΕ)

<=> = (ΑΒΓ)

α · ΗΖ · ΗΕ + β · ΗΖ · ΗΔ + γ · ΗΔ · ΗΕ = --------�--------�-----αβγ

(ΔΕΖ) α · ΗΖ · ΗΕ + β · ΗΖ · ΗΔ + γ · ΗΔ · ΗΕ � ---- = --------�--------�-----

(ΑΒΓ) αβγ � α · ΗΖ · ΗΕ + β · ΗΖ · ΗΔ + γ · ΗΔ · ΗΕ =

= αβ . (ΔΕΖ) = 4(ΑΒΓ) · R · (ΔΕΖ) = 4R · (ΔΕΖ) γ (ΑΒΓ) (ΑΒΓ

Δύο κύκλοι C, , C2 τέμνονται και έστω Α , Β τα κοινά τους σημεία. Ένας τρίτος κύκλος C3 έχει το κέντρο του στο μέσον Ο της διακέ-

ντρου Κ1Κ2 των c, , C2 και διέρχεται από τα σημεία Α, Β . Θεωρούμε τυχαίο σημείο Ρ στο επίπεδο των κύκλων αυτών και ονομάζουμε D1 , D2 , D3 τις δυνάμεις του Ρ ως προς τους κύ-κλους c, , C2 , C3 αντιστοίχως. Να αποδειχθεί ό­

τι: D, + D2 = 2D3 •

λπιiδηξη : Ονομάζουμε R 1 , R υ R 3 τις ακτίνες των κύ-

κλων C 1 , Cυ C3 αντιστοίχως. 'Εχουμε : D1 + D2 = ΡΚ1 2 - R � + ΡΚ/ - R ; =

= (ΡΚ1 2 + ΡΚ/ ) - (R � + R ; ) (1)

Από το πρώτο θεώρημα των διαμέσων: Α ) Στο τρίγωνο ΡΚ 1Κ 2 έχουμε:

2 ΡΚ 1 2 + ΡΚ/ = 2 · Ρ0 2 + Κ 1Κ 2 (2) . 2

Β) Στο τρίγωνο ΑΚ 1Κ 2 έχουμε : 2

ΑΚ 2 + ΑΚ 2 = 2 · ΑΟ2 + K IK 2 Ι 2 2 κ κ 2

R 2 + R 2 = 2R 2 + _ι _2_ � I 2 3 2

� Κ,Κ/ = R2 + R2 - 2R2 (3)

2 I 2 3

δηλ.

Από τις σχέσεις (1) , (2), (3) παίρνουμε

D + D = 2 · Ρ02 + Κ,Κ/ - (R2 + R2 ) = I 2 2 I 2 - 2 · ΡΟ2 + R 2 + R 2 - 2R 2 - (R 2 + R 2 ) - I 2 3 I 2 = 2(Ρ02 - R ; ) = 2D3

ΛΣ Κ Η Σ Η ( 4 ) Δίνεται κύκλος (O, R) και σημείο Ρ στο

εσωτερικό του κύκλου. Τυχαία ευθεία ε διέρχε­ται από το σημείο Ρ και τέμνει τον κύκλο στα σημεία Α,Β . Θεωρούμε τις εφαπτόμενες Ax, By του κύκλου και ονομάζουμε Κ, Λ τις προβολές του Ρ πάνω στις Αχ, By , αντιστοίχως.

Ν δ θ . . ' θ ι ι . α απο ει χ ει οτι το α ροισμα - + - ειναι ΡΚ ΡΛ

σταθερό. ( δηλ. ανεξάρτητο της ευθείας ε ).

Λπόδ�: ιξη : Έστω ΟΡ = d > Ο Τότε για την ειδική περί-

πτωση της ευθείας ΡΟ = Α1Β1 έχουμε 1 1 1 1 2R -- +-- = -- +-- = = PKI ΡΛΙ R + d R - d R 2 - d 2

2R Θ δ 'ξ ' ' 'δ = � α ει ουμε οτι ισχυει το ι ιο και ΡΑ · ΡΒ για τυχαία ευθεία ΑΒ από το Ρ . Για αυτό θα υπο­

ΕΥΚΛΕΙΔΗΣ Β' 66 τ.2/40

Page 43: Ευκλειδης Β 66

Μαθη ματικά για την Β ' Λυκείου

λογίσουμε τα ΡΚ, ΡΛ ως συνάρτηση των ΡΑ, ΡΒ . Αν Γ το aντιδιαμετρικό σημείο του Α. Είναι

ΑΒΓ = π ( βαίνει σε ημικύκλιο) . 2

Χ ι

; ; ; ,. "' R ; - -

; - -; - -- -

y ι Λ Λ Λ

Επειδή είναι ΡΑΚ = ΡΒΑ = ΑΓΒ (υπό χορ­δής ΑΒ και εφαπτομένης οι δύο πρώτες - εγγε­γραμμένη στο τόξο ΑΒ η τρίτη ) καθένα από τα τρίγωνα ΡΑΚ , ΡΒΛ είναι όμοιο πρός το τρίγωνο ΑΓΒ . Από την ομοιότητα των τριγώνων: α) ΡΑΚ και ΑΓΒ έχουμε :

ΑΒ = ΑΓ δ λ. _Ι_ = 2R (Ι) ΡΚ ΡΑ η ΡΚ ΡΑ · ΑΒ β) ΡΒΛ και ΑΓΒ έχουμε :

ΑΒ = ΑΓ δ λ. _Ι_ = 2R ΡΑ ΡΒ η ΡΑ ΡΒ · ΑΒ (2) .

Με πρόσθεση κατά μέλη των σχέσεων (Ι) , (2) παίρνουμε:

Ι Ι 2R Ι Ι 2R Ρ Α + ΡΒ ΡΚ

+ Ρ Λ

= ΑΒ

(Ρ Α

+ ΡΒ

) = ΑΒ

. Ρ Α · ΡΒ

=

σταθερό .

2R ΑΒ 2R

Τέλος και για d=O έχουμε :

Β 'τρόπος

Ι Ι 2 2R -· - + - = - = ----,,---------,-

ΡΚ ΡΑ R R2 - d2

πόν ΑΒ = 2R, ή ΑΒ=2Rημθ, που ισχύει από το ημθ

θεώρημα ημιτόνων σε οποιοδήποτε τρίγωνο ΑΒΓ του σχήματος

ΛΣ Κ Η l: Η (5 ) Δίνεται κυρτό τετράπλευρο ΑΒΓ Δ . Έστω

Κ,Λ,Μ,Ν τα μέσα των πλευρών ΑΒ, ΒΓ, Γ Δ, ΔΑ αντιστοίχως, Τ το μέσον της ΑΓ και Σ το μέσον της ΒΔ. Θεωρούμε τυχαίο σημείο Ρ εσωτερικό του τετραπλεύρου. Ονομάζουμε 01 , 02 , 03 , 04 τις δυνάμεις του Ρ ως προς τους κύκλους με δι­αμέτρους τις πλευρές ΑΒ, ΒΓ, ΓΔ, ΔΑ του τε­τραπλεύρου, αντιστοίχως και Ο τη δύναμη του σημείου Ρ ως προς τον κύκλο με διάμετρο ΤΣ. Να αποδειχθεί ότι: 01 + 02 + 03 + 04 = 40 .

C ι

Απόδ�: ιξη : Είναι :

Ο = PKz - (AB )z Ο = ΡΛz - ( ΒΓ )z ι 2 , 2 2 ,

Ο = PMz - ( ΓΔ )z Ο = PNz - (ΑΔ )z 3 2 , 4 2 Με πρόσθεση κατά μέλη των παραπάνω σχέ­

σεων παίρνουμε : Οι + 02 + 03 + 04 = ΡΚ2 + ΡΛ2 + ΡΜ2 + ΡΝ2 -

_ _!_ (ΑΒ2 + ΒΓ2 + ΓΔ2 + ΑΔ2 ) 4

= (ΡΚ2 + ΡΜ2 ) + (ΡΛ2 + ΡΝ2 )

_ _!_ (ΑΒ2 + ΒΓ2 + ΓΔ2 +ΑΔ2 ) 4

Από το πρώτο θεώρημα των διαμέσων τρίγωνα ΡΚΝ και ΡΝΛ έχουμε αντιστοίχως:

ΡΚ2 + ΡΜ2 = 2 · ΡΟ2 + ΚΜ2 (2) , 2 (3)

(Ι)

στα

ΕΥΚΛΕΙΔΗΣ Β ' 66 τ.2/41

Page 44: Ευκλειδης Β 66

Μαθηματικά για την Β ' Λυκείου

Σύμφωνα με το γνωστό θεώρημα του Euler, (Άσκηση 4 αποδεικτική της σελίδας 1 98 του σχο­λικού εγχειριδίου) για το τετράπλευρο ΑΒΓ Δ έ­χουμε : ΑΒ +Br +ΓΔ +ΑΔ =Ar +ΒΔ +4ΣΤ (4) .

Η ισότητα (1) , βάσει των ισοτήτων (2) , (3) , (4) γράφεται:

ΚΜz D, + D2 + D3 + D4 = 2 · Ρ02 + -- + 2 · Ρ02 + 2

+ ΛΝ2 _ _!_ (ΑΓ2 + ΒΔ2 + 4ΣΤ2 ) =

2 4

=4Pd +_!.(ΚΜ2 +ΝΝ)-_!.(ΑΓ2 +Μ)-ΣΓ2 2 4

Pd 1 2 2 1 z 2 Σf z =4 +- (ΚΜ +ΝΛ )-- (4 ·ΚΛ: +4 · ΜΛ ) -4 · (-) 2 4 2

=4[Pd -(ΣΓ)2 ] +_!. (ΚΜ2 +NN)-(I<N +ΜΝ) 2 2

=4D+_!.[ΚΜ2 + ΝΝ -2(1<Ν + ΜΝ)] (5) 2

Αρκεί να δειχθεί ότι: _!_ [ΚΜ 2 + ΝΛ2 - 2(ΚΛ2 + ΜΛ2 )] = Ο

,

ή 2 ΚΜ 2 + ΝΛ2 - 2(ΚΛ2 + ΜΛ2 ) = Ο

, ή ΚΜ2 +ΝΝ - κΝ -ΜΝ -ΝΜ2 -ΚΝ2 = 0 , ή ΚΛ2 + ΜΛ2 + ΝΜ 2 + ΚΝ 2 = ΚΜ 2 + ΝΛ2

σχέση που ισχύει (ως γνωστόν) σε κάθε παραλλη­λόγραμμο .

ΑΣ Κ Η 1: Η ( 6 ) Τρίγωνο ΑΒΓ είναι εγγεγραμμένο σε κύκλο.

Η διάμεσος ΑΜ = μα τέμνει τον κύκλο στο ση-

μείο Ρ :Εστω ΒΝ = μ Ρ η διάμεσος που αντιστοι­

χεί στην πλευρά ΑΓ του τριγωνου και G το κέ­ντρο βάρους του τριγώνου ΑΒΓ. Ν α δειχθεί ότι:

2 4 2 3 2 α) ΜΡ = � β) GP = μα + α

4μa 1 2μα

γ) (GBP) = 2(β2 + γ2 + α2 ) (GAN) 2β2 + 2γ2 - α2

Α___. __

Ρ

Απόδειξη : α ) Από το θεώρημα των τεμνομένων χορδών,

για τις χορδές ΑΡ,ΒΓ έχουμε:

β)

α α ΜΑ · ΜΡ = ΜΒ · ΜΓ ::::::> μ · ΜΡ = - · - ::::::> a 2 2

αz ::::::> ΜΡ = -4μα

1 α2 GP = GM + MP = -μ + - =

3 α 4μα

(2)

(1)

γ) Επειδή τα τρίγωνα BGP, AGN έχουν τις κατά Λ

κορυφήν γωνίες G ίσες, από το θεώρημα των εμ-βαδών στα τρίγωνα αυτά έχουμε:

2 4μ� + 3α2 (BGP) GB · GP 3 μΡ

. 1 2μα

__;___.;_ = = ___ ____;___,:....__

(AGN) GA · GN 2 1 ) μα . J μβ

2βz + 2γz - αz + 3αz 2(βz + γz + αz ) 2βz + 2γz - αz 2β2 + 2γz - αz

ΑΣ Κ Η Σ Η ( 7 ) Δίνεται τρίγωνο ΑΒΓ. Έστω ΑΜ, ΒΝ, ΓΡ οι

διάμεσοι του τριγώνου. Θεωρούμε το παραλλη­λόγραμμο ΒΜΤΝ. Να αποδείξετε ότι: α) η Α Τ είναι παράλληλη και ίση προς την τρί­τη διάμεσο ΓΡ του τριγώνου ΑΒΓ.

β) (ΑΜτ) = �(ΑΒΓ) . 4

Α

τ

Β

Γ Απόδειξη α) Αφού το τετράπλευρο ΒΜΤΝ είναι παραλ­

ληλόγραμμο, θα έχουμε :

ΝΤ 11 = ΒΜ = ΒΓ = ΡΝ . Δηλ. το Ν είναι κοινό

2 μέσο των ΡΤ και ΑΓ, οπότε το τετράπλευρο ΑΤΓΡ είναι παραλληλόγραμμο με αποτέλεσμα να ισχύει ΑΤ 11 = ΓΡ , που είναι το ζητούμενο. β) Επειδή η ΤΝ είναι διάμεσος του τριγώνου Α ΤΓ ισχύει

ΕΥΚΛΕΙΔΗΣ Β ' 66 τ.2/42

Page 45: Ευκλειδης Β 66

Μαθη ματικά για την Β ' Λυκείου

( ΑΝτ) = _!_ (Α ΤΓ) = _!_ (ΑΓΡ) = _!_ · _!_ (Α Β Γ) . 2 2 2 2 δηλ. (ΑΝτ) = _!_ (ΑΒΓ) (1) 4

Επειδή η ΜΝ είναι διάμεσος του τριγώνου ΑΜΓ ισχύει:

(ΑΝΜ = � (ΑΜ) = � ·� (ΑΙΓ) =� (ΑΗ) (2) Τέλος, είναι

(ΜΝΤ) = (ΡΜΝ) = (ΜΝΓ) = _!_ (ΑΒΓ) (3) , 4 α ού (ΜΝΓ) = ( ΜΝ ) 2 = ( _!_) 2 = _!_ . φ

(ΑΒΓ) ΑΒ 2 4 Με πρόσθεση των ( 1 ),(2),(3) κατά μέλη έχουμε :

(ΑΝτ) + (ΑΝΜ) + (ΜΝτ) = � (ΑΒΓ) � 4

� (ΑΜτ) = � (ΑΒΓ) 4

ΛΣ Κ Η Σ Η ( 8 ) Δίνεται κυρτό τετράπλευρο ΑΒΓ Δ . Έστω

Κ,Λ,Ρ,Τ τα μέσα των πλευρών ΑΒ,ΒΓ,ΓΔ,ΔΑ αντιστοίχως. Από το μέσον Μ της ΑΓ φέρουμε ευθεία παράλληλη προς την διαγώνιο ΒΔ. Από το μέσον Ν της ΒΔ φέρουμε ευθεία παράλληλη προς την διαγώνιο ΑΓ και ονομάζουμε Σ το σημείο τομής των ευθειών αυτών. Φέρουμε τις ΣΚ, ΣΛ, ΣΡ, ΣΤ. Να αποδειχθεί ότι οι ευθείες αυτές διαιρούν το τετράπλευρο ΑΒΓ Δ σε τέσσε­ρα ισοδύναμμα τετράπλευρα.

Δ

Α

Απόδε ιξη : Αρκεί να δείξουμε ότι:

(ΣΛΓΡ) = (ΣΛΒΚ) = (ΣΚΑτ) =

= (ΣΤΔΡ) = _!_ (ΑΒΓΔ) . 4

Γ

Είναι: (ΣΛΓΡ) = (ΣΛΡ) + (ΡΛΓ) (1) . Επει-

δή ΣΜ // ΒΔ και ΡΛ // ΒΔ είναι ΣΜ // ΡΛ . Άρα τα τρίγωνα ΣΡ Λ και ΜΡ Λ έχουν κοινή βάση Ρ Λ και ύψη ίσα προς την απόσταση των παραλ­λήλων ΣΜ, Ρ Λ , οπότε ισχύει: (ΣΛΡ) = (ΜΛΡ) (2) . Οι πλευρές του τριγώνου ΜΛΡ είναι μία προς μία παράλληλες προς τις πλευρές του τριγώνου ΑΔΒ και ίσες προς το μι­σό της κάθε μιάς. Άρα τα τρίγωνα ΜΛΡ και

ΔΑΒ είναι όμοια με λόγο ομοιότητας � , οπότε

ισχύει: (ΜΛΡ ) __ (_!_) 2 __ _!_ (ΔΑΒ) 2 4 '

(ΜΛΡ) = _!_ (ΔΑΒ) (3) . Επειδή τα 4

δηλ.

σημεία

Ρ, Λ είναι τα μέσα των πλευρών ΔΓ, ΒΓ αντί­στοιχα του τριγώνου ΔΒΓ ισχύει:

(ΡΛΓ) = ( ΡΓ ) 2 = (_!_) 2 = _!_ (ΔΒΓ) ΔΓ 2 4 δηλ. (ΡΛΓ) = _!_ (ΔΒΓ) (4) . 4

Βάσει των ισοτήτων (2), (3), (4) η ισότητα (1) γράφεται:

(ΣΛΓΡ) = _!_ (ΔΑΒ) + _!_ (ΔΒΓ) =

4 4

= _!_ [(ΔΑΒ) + (ΔΒΓ)] = _!_ (ΑΒΓ Δ) 4 4

Εντελώς ανάλογα προκύπτουν και οι υπόλοι­πες προς απόδειξη ισότητες .

ΑΣ Κ Η Σ Η (9) Δίνεται παραλληλόγραμμο ΑΒΓ Δ. Τυχαία

ευθεία που διέρχεται από την κορυφή Γ τέμνει την προέκταση της ΑΒ στο σημείο Ε και την προέκταση της ΑΔ στο σημείο Ζ. Έστω Ρ η το­μή των ΔΕ,ΒΖ. Ν α αποδειχθεί ότι το εμβαδόν του τετραπλεύρου ΑΒΡΔ είναι ίσο προς το εμ­βαδόν τουτριγώνου ΡΕΖ.

Ε Απόδε ιξη :

z

Αρκεί να αποδειχθεί ότι: (ΑΒΡΔ) = (ΡΕΖ) , ΕΥΚΛΕΙΔΗΣ Β ' 66 τ.2/43

Page 46: Ευκλειδης Β 66

Μαθηματικά για την Β ' Λυκείου

ή το αυτό : (ΑΒΡΔ) + (ΡΔΖ) = (ΡΕΖ) + (ΡΔΖ) δηλ. αρκεί να ισχύει: (ΑΒΖ) = (ΕΔΖ) . Επειδή εί­ναι ΔΖ I I ΒΓ τα τρίγωνα ΒΔΖ και Γ ΔΖ έχουν κοι­νή βάση ΔΖ και ύψη ίσα προς την απόσταση των παραλλήλων ευθειών ΔΖ,ΒΓ.

Άρα, (ΒΔΖ) = (Γ ΔΖ) (1) . Επειδή είναι Γ Δ I I ΒΕ τα τρίγωνα ΒΓ Δ και

ΕΓ Δ έχουν κοινή βάση Γ Δ και ύψη ίσα προς την απόσταση των παραλλήλων ευθειών ΕΒ, Γ Δ.

Άρα, (ΒΔΓ) = (ΕΔΓ) και επειδή είναι (ΒΔΓ) = (ΑΒΔ) , ισχύει (ΑΒΔ) = (ΕΔΓ) (2) . Προσθέτουμε τις ισότητες (1) , (2) κατά μέλη και έχουμε: (ΒΔΖ) + (ΑΒΔ) = (ΓΔΖ) + (ΕΔΓ) και τελικά (ΑΒΖ) = (ΕΔΖ) , δηλ. το ζητούμενο. Β 'τρόπος

(ΑΒΖ) ΑΒ·ΑΖ ΑΒ Λ

Έχουμε: -( --) = = - (κοινή γωνία Α ) ΑΕΖ ΑΕ·ΑΖ ΑΕ

(ΕΔΖ) ΖΔ ·ΖΕ ΖΔ Λ

-( --

) = = - (κοινή γωνία Ζ ) .

ΑΕΖ ΖΑ·ΖΕ ΖΑ , ΑΒ ΖΔ ΓΔ ΖΔ

Αρκει - = - , ή - = - , που ισχύει α-ΑΕ ΖΑ ΑΕ ΖΑ Δ Δ

φού ΖΓΔ � ΖΕ Α Π αρατή ρηση : Αν τα Β,Γ,Δ είναι μέσα των πλευ-

ρών του Α Ε Ζ τότε (ΑΒΡΔ ) = l_ (ΑΕΖ ) (γιατί;) 3 ΛΣ Κ Η Σ Η ( 1 0 ) Δίνεται παραλληλόγραμμο ΑΒΓ Δ εμβαδού

k2 • Έστω Μ,Ν,Ρ,Σ τα μέσα των πλευρών ΑΒ,ΒΓ ,Γ Δ, ΔΑ αντίστοιχα.

Οι ευθείες ΜΓ, ΜΔ, ΝΑ, ΝΔ, ΡΑ, ΡΒ, ΣΒ, ΣΓ σχηματίζουν οκτάγωνο. Να υπολογισθεί το εμ­βαδόν του οκταγώνου αυτού συναρτήσει του k.

Απόδειξη Θεωρούμε τις διαγωνίους ΑΓ, ΒΔ του παραλ­

ληλογράμμου και ονομάζουμε Ο το κέντρο του. Έστω Ε η τομή των ΣΒ , ΔΜ και Ζ η τομή των ΝΑ, ΣΒ. Επειδή ΣΝ I I = ΑΒ το ΑΒΝΣ είναι πα­ραλληλόγραμμο με διαγωνίους ΑΝ , ΣΒ που διχο­τομούνται στο Ζ. Προφανώς, το σημείο Ζ είναι το μέσον της ΟΜ . Στο τρίγωνο ΑΣΝ οι ΑΟ , ΣΖ εί­ναι διάμεσοί του οπότε το σημείο Ε είναι το κέ­ντρο βάρους του .

Άρα ισχύουν: ΟΕ = l.Ao και ΟΖ = ..!.οΜ . 3 2

Μ Β ...... ιf' 1 ' ' Ι I I I ' ' , Ι I I I ' ' ..._ Ι 1 I I \ ">' ..._ 1 ..λ_.. ι' Ι I ' I ..... I ι' Ι

Ι � ιΖ Ι , "' ι Ι ..-

· ""'t E Ι .r�' , / ..λ _.. ' I "' Π Ι' _.. .... ..- I Τ ' ι ,"' I ' ,

Σ ... ,- - -, - - - ;'lit - -e •- - :.-;; Ν ' , ι ι' 0' I _.. /' .... . �"' ,' ! .. .... .... � .... ι "' r Κι ι"'\ 1

I , "' Ι ...... Ι ' I Ι ι' \..- _.. I ../ ' I I ι' _.. Ι ..._ ' ι "' _.. .... I Ι Ι , ' I I ι' _.. _.. I I ι ' ..._ ' I ��'_.. ..- I Ιι ' ,'Ι

Δ p Γ Από το θεώρημα των εμβαδών για τα τρίγωνα

ΟΕΖ και ΟΑΜ έχουμε :

(ΟΕΖ) ΟΕ · ΟΖ = ----

(ΟΑΜ) ΟΑ · ΟΜ ΟΑ · ΟΜ δηλ. είναι

1 1 1 (ΟΕΖ) = "6(0ΑΜ) = "6 · l(OAB) =

1 1 1 1 ο = - · - · - (ΑΒΓΔ) = -k-6 2 4 48

1 6

Εντελώς όμοια αποδεικνύεται ότι καθένα από τα υπόλοιπα επτά τρίγωνα του σχηματιζομένου οκταγώνου ΕΖΗΘΙΚΛ Τ που έχουν κοινή κορυφή

ο ' β δ ' ' 1 k 2 , εχει εμ α ον ισο προς το - . 48

Τελικά, είναι:

(ΕΖΗΘΙΚΛΤ) = 8 · -1 k 2 = l_ k 2 • 48 6

Λ Σ Κ Η Σ Η ( 1 1 ) Δίνεται τρίγωνο ΑΒΓ ορθογώνιο στο Α με

πλευρές α,β,γ. Έστω ΒΔ,ΓΖ οι εσωτερικές διχο­τόμοι των γωνιών Β και Γ και Ι το έγκεντρο του τριγώνου. Να αποδειχθεί ότι: (ΒΓΔΕ) = 2(ΒΙΓ) .

Απόδειξη Είναι:

1 1 1 Ε (ΒΙΓ) = - · ΒΓ · ΙΚ = -αρ = -α- = 2 2 2 τ

1 1 2 βγ αβγ = -α- = - (1) 2 τ 4τ

1 1 (ΒΓΔΕ) = (ΑΒΓ) - (ΑΔΕ) = -βγ - -ΑΕ · ΑΔ = 2 2

= ..!. βγ - ..!. . _1r_ . _1r_ = 2 2 α + β α + γ

= ..!. βγ[1 -βγ ] =

2 (α + β)(α + γ)

ΕΥΚΛΕΙΔΗΣ Β ' 66 τ.2/44

Page 47: Ευκλειδης Β 66

Μαθη ματικά για την Β ' Λυκείου

= _!_ βγ · (α + β)(α + γ) - βγ = 2 (α + β)(α + γ)

= βγ α2 + αγ + αβ = 2(α + β)(α + γ)

= βγ α(α + β + γ)

2α2 + 2αβ + 2βγ + 2αγ

= βγ α(α + β + γ) = α2 + α2 + 2αβ + 2βγ + 2αγ

= βγ α(α + β + γ) α2 + β2 + γ2 + 2αβ + 2βγ + 2αγ

= βγ α(α + β + γ) = αβγ αβγ (2) (α + β + γ)2 α + β + γ 2τ

Από τις ισότητες ( 1 ) και (2) προκύπτει το ζη­τούμενο.

Γ \ \ \

β

Δ

Α

\ \ \ \ \ \ \ \ \ \ \ \ \

· --�

\ \ \ \ \ ,.

' ..... .... \ "' ' , .... .... .... .... \ [ , "' "' ρ

_ _ .,_ _ _ _ ,. . 'f \ ... .... .... ' ' .... ..... ' \ .... ..... ' \ '

' , \ ' \

' \ ' \ ' · '

Ε

Β ' τρόπος Αρκεί να δείξουμε ότι:

γ

( ΙΔΕ ) + ( ΙΕΒ) + ( ΙΔΓ) = ( ΙΒΓ) , ή

( ΙΔΕ ) + ( ΙΕΒ) + ( ΙΔΓ) =

Ι ( ΙΒΓ) ( ΙΒΓ) ( ΙΒΓ) .

Β

Οι λόγοι αυτοί υπολογίζονται ως συνάρτηση των πλευρών α,β,γ, αφού τα αντίστοιχα τρίγωνα έχουν ίσες ή παραπληρωματικές γωνίες.

ΑΣ Κ Η Σ Η ( 1 2 ) Δίνεται οξυγώνιο τρίγωνο ΑΒΓ εγγεγραμμέ­

νο σε κύκλο (0, R) και έστω ΑΔ,ΒΕ,ΓΖ τα ύψη του. Να αποδειχθεί ότι:

α) Οι πλευρές ΕΖ, ΖΔ, ΔΕ του ορθικού τρι­γώνου ΔΕΖ είναι κάθετες στις ακτίνες ΟΑ, ΟΒ, ΟΓ αντιστοίχως.

β) (ΑΒΓ) = R · τ ' , όπου 2τ' η περίμετρος του ορθικού τριγώνου ΔΕΖ.

Απόδειξη α) Φέρουμε την εφαπτομένη Αχ του κύ­

κλου. Τότε είναι ΟΑ .l Αχ , οπότε για να είναι Λ

ΟΑ .l ΖΕ , αρκεί ΖΕ// Αχ. Επειδή η γωνία ΒΑχ είναι υπό χορδής ΒΓ και εφαπτομένης Αχ ισχύει

Λ Λ

ΒΑχ = Γ (αντίστοιχη εγγεγραμμένη στο τόξο Λ Λ π

ΑΒ). Επειδή ΒΕΓ = ΒΖΓ = - , το τετράπλευρο 2 ΒΖΕΓ είναι εγγράψιμμο σε κύκλο, οπότε ισχύει

Λ Λ Λ Λ

Γ = ΕΖΑ . Άρα, είναι ΒΑχ = ΕΖΑ γεγονός, που σημαίνει ότι οι ευθείες Αχ και ΖΕ είναι παράλλη­λες ( αφού σχηματίζουν τις εντός εναλλάξ γωνίες ίσες). Όμοια αποδεικνύονται οι υπόλοιπες καθετό­τητες.

β) Παρατηρούμε ότι καθένα από τα τετρά­πλευρα ΟΖΑΕ, ΟΖΒΔ, ΟΔΓΕ έχει καθέτους δια­γωνίους και άρα εμβαδόν ίσο με το ημιγινόμενο των διαγωνίων του.

Έχουμε : (ΑΒΓ) = (ΟΖΑΕ) + (ΟΖΒΔ) + (ΟΔΓΕ)=

= _!_ΟΑ · ΖΕ + _!_ΟΒ · ΔΖ + _!_ΟΓ · ΔΕ 2 2 2

= ..!_ R · ΖΕ + ..!_ R · ΔΖ + ..!_ R · ΔΕ = 2 2 2 1 1 = -R · (ΖΕ + ΔΖ + ΔΕ) = -R · 2τ' = Rτ' 2 2

ΑΣ Κ Η Σ Η ( 1 3 ) Δίνεται κύκλος (O,R). Θεωρούμε μία διάμε­

τρό του ΒΓ και την ακτίνα ΟΑ κάθετη στην ΒΓ. Α ν Μ είναι τυχαίο σημείο του ελάσσονος τόξου

ΑΓ δ θ . . 1 1 2

να απο ειχ ει οτι: -- = --(ΜΑΓ) (ΜΑΒ) (ΜΒΓ)

ΕΥΚΛΕΙΔΗΣ Β ' 66 τ.2/45

Page 48: Ευκλειδης Β 66

Μαθηματικά για την Β ' Λυκείου

Απόδε ιξη

Αρκεί να δείξουμε ότι: (ΜΒΓ) - (ΜΒΓ) = 2 (ΜΑΓ) (ΜΑΒ)

Έχουμε: ( βλέπε σχήμα ) Λ Λ π Λ Λ

ΑΒΓ = ΑΓΒ = - , ΑΒΜ = ΑΓΜ = φ . 4 Α

Τα τρίγωνα ΜΒΓ , ΜΑΓ έχουν: Λ π Λ

ΜΒΓ = - - φ = ΜΑΓ . 4 Από το θεώρημα των εμβαδών έχουμε :

(ΜΒΓ) =

ΒΓ · ΒΜ = 2R · BM = fi ΒΜ (1) (ΜΑΓ) ΑΓ · ΑΜ Rfi · AM ΑΜ Τα τρίγωνα ΜΒΓ, ΜΑΓ έχουν:

Λ Λ Π 3π ΜΓΒ+ ΜΑΒ = (4 + φ) + (4 - φ) = π . Από το θεώρημα των εμβαδών έχουμε:

(ΜΒΓ) =

ΒΓ · ΓΜ =

2R · ΓΜ = fi ΓΜ (2) (ΜΑΒ) ΑΒ · ΑΜ Rfi · AM ΑΜ

Αναιρούμε κατά μέλη τις ( 1 ),(2) και παίρνου­με :

(ΜΒΓ) _ (ΜΒΓ) = J2 ΒΜ _ J2 ΓΜ =

(ΜΑΓ) (ΜΑΒ) ΑΜ ΑΜ

= J2

(ΒΜ - ΓΜ) (3) ΑΜ

Για να ισχύει η αποδεικτέα, αρκεί να ισχύει (λόγω της (3)) :

ΒΜ - ΓΜ = J2 · ΜΑ (4) .

Πάνω στην ΜΒ θεωρούμε τμήμα ΜΔ=ΜΓ, οπότε είναι ΒΜ-ΓΜ=ΒΔ και άρα αρκεί να ισχύει :

ΒΔ = J2 · ΜΑ (5) .

Συγκρίνουμε τα τρίγωνα ΒΔΓ, ΑΜΓ.

Έχουν Λ Λ Λ 3π Λ

ΒΓ Δ = φ = ΑΓΜ και ΒΔΓ = - = ΑΜΓ . 4 Άρα, τα τρίγωνα αυτά είναι όμοια με αποτέ­

λεσμα να ισχύει:

ΔΒ = ΒΓ =>

ΔΒ = _.!!.__ = J2 => ΔΒ = J2 . ΑΜ ΑΜ ΑΓ ΑΜ RJ2 , δηλ.το ζητούμενο .

Β ' τρόπος (για την (4) ) : Από Θ. Πτολεμαίου έχουμε :

ΜΒ · ΑΓ = ΜΓ · ΑΒ + ΜΑ · ΒΓ =>

ΜΒ · R J2 = ΜΓ · R J2 + ΜΑ · 2R =>

ΜΒ - ΜΓ = J2ΜΑ

Κατεύ θυνση Ε υ θ ε ί α

Θανάσης Τζιώτζιος - Κώστας Κουτρουμπέλας - Βαγγέλης Τσατούρας Η μελέτη της ευθείας καθώς και άλλων επίπεδων σχημάτων γίνεται και με την χρήση της Άλγεβρας. Αυ­

τή η αναλυτική θεώρηση διευκολύνει την επίλυση προβλημάτων που με την Ευκλείδεια Γεωμετρία ήταν αρ­κετά πιο σύνθετα.

Η Αναλυτική Γεωμετρία βασίζεται στο ότι: η εξίσωση φ(χ, y) = Ο παριστάνει μια καμπύλη, αν και μόνο αν, η καμπύλη αυτή είναι το σύνολο των σημείων του επιπέδου, που οι συντεταγμένες τους επαληθεύουν την εξίσωση. Στο άρθρο που ακολουθεί παρουσιάζονται μορφές ασκήσεων και οι τρόποι επίλυσης αυτών, με στόχο την καλύτερη κατανόηση από τους μαθητές των σχέσεων και των μεθόδων εργασίας του κεφαλαίου αυτού.

1 ) Για τον μη μηδενικό πραγματικό αριθμό τέμνει την ευθεία y = 2χ - α στο σημείο α, ορίζουμε τα σημεία Α(Ο, α) και Γ. Να δειχθεί ότι το τρίγωνο ΑΒΓ είναι Β(4α, -2α). Η κάθετη στην ΑΒ στο Α ισοσκελές.

ΕΥΚΛΕΙΔΗΣ Β ' 66 τ.2/46

Page 49: Ευκλειδης Β 66

Μαθη ματικά για την Β ' Λυκείου

Ο συντελεστής διεύθυνσης της ΑΒ είναι λΑ8 = - 3α = _i _ Αφού ΑΓ l_ ΑΒ ισχύει ότι :

4α 4 λΑΒ · λΑr = -1 => λΑr = _i . Η ευθεία ΑΓ είναι: 3

4 4 y - α = 3 (χ - 0), => y = 3 χ + α. Οι συντεταγμένες του σημείου Γ δίνονται από {Υ = i χ + α} την λύση του συστήματος : 3 (Σ) .

y = 2χ - α ο , (Σ) {� χ + α = 2χ - α} {χ = 3α} ποτε: <:::::> 3 <:::::> y = 5α y = 2χ - α Άρα Γ(3α, 5α) . Το μήκος του ΑΒ είναι : (ΑΒ)=�(4α -0) 2 + ( - 2α-α)2 = �25α2 = 5lα l και του ΑΓ είναι: (ΑΓ) = �r-(3

_α ___ Ο

_) 2-+-(-Sα---α )-2 = � 25α 2 = 5 lα l .

Επομένως ΑΒ = ΑΓ, δηλαδή το τρίγωνο ΑΒΓ είναι ισοσκελές.

2) Από σημείο Μ της ευθείας y=x φέρουμε τυχαία ευθεία που τέμνει τους θετικούς η­μιάξονες Οχ και Oy στα σημεία Α και Β α-

, Ν δ θ ' ' 1 1 ντιστοιχα. α απο ει χ ει οτι: -- + --

(ΟΑ) (ΟΒ) είναι σταθερό.

Στην y = χ θεωρούμε σταθερό σημείο Μ( α, α) με α > Ο, ώστε η τυχαία ευθεία να μπορεί να τέμνει τους θετικούς ημιάξονες.

Η τυχαία ευθεία που φέρουμε θα έχει εξί­σωση : y - α = λ(χ - α) με λ<Ο, ώστε να είναι δυνατό να τέμνει και τους δύο θετικούς ημιά-ξονες. Για y = Ο , έχουμε : χ = α - α = λ = α(λ - 1) _ Άρα (ΟΑ) = ι α(λ - 1) 1 = α(λ - 1) . λ λ λ Για χ = Ο έχουμε y = α - αλ = α( 1 - λ) . Επομέ­νως (ΟΒ) = lα( l - λ) l = α( 1 - λ) . Οπότε :

1 1 λ 1 λ - 1 1 -- + -- = + = = (ΟΑ) (ΟΒ) α(λ - 1) α(Ι - λ) α(λ - 1) α Δηλαδή σταθερό .

0(0, Ο) θεωρούμαι τα σημεία Α(α,Ο}, Β(Ο,β) και Γ(β,α). Η διάμεσος ΓΜ του τριγώνου ΑΒΓ τέμνει τον άξονα χχ ' στο Κ(κ, Ο) ενώ τον άξονα yy ' στο Λ(Ο, λ).

Αν ισχύει: (ΜΚΑ) = Υ.. , τότε :

ί. Ν α δειχθεί ότι : κ = α ± _!_ . β

ίί. Επίσης, αν (ΟΚΛ) = 2(ΑΚΜ), τότε να δειχθεί ότι οι απόλυτες τιμές των κ, λ είναι aντίστροφοι αριθμοί.

i. Οι συντεταγμένες του μέσου Μ της ΑΒ εί-α + Ο α Ο + β β ναι: χ Μ = -2- = 2 και ΥΜ = -2- = 2 . Συ-

νεπώς προκύπτουν τα διανύσματα: � α β �

ΑΜ = ( - - , - ) και ΑΚ = (κ - α, Ο) . Άρα 2 2 � �

έχουμε : (ΜΚΑ)= lf4=> Yzjdet( AM , AK ) I = lf4 κ - α Ο 1 1 => I α β 1= 2 =>1(κ - α)β 1 = 1=>κ = α ± β .

2 2

Β ω . β Ι

. \1.0 . i. Ι

.-\ω .. ΟΙ Κ 1.1ι. .Ο 1

� �

i i . Επειδή ΟΚ = (κ, Ο) και ΟΑ = (0, λ) θα είναι: (ΟΚΛ) = Yz Ι κλΙ . Όμως: (ΜΚΑ) = li4 => Yz Ι κλl = 2 lf4 => Ικ Ι Ιλl = 1 .

4 ) Να βρεθεί η ευθεία που διέρχεται από τα

σημεία Α(-! , α) και Β(-_!_ , β) με α;i:β α β

μη μηδενικούς πραγματικούς αριθμούς. Α ν η ευθεία ΑΒ τέμνει τον άξονα χχ ' στο σημείο Γ και τον yy ' στο Δ, να απο­δειχθεί ότι ΑΓ = ΒΔ και ακολούθως ότι: 3 ) Σε ορθοκανονικό σύστημα με κέντρο

ΕΥΚΛΕΙΔΗΣ Β' 66 τ.2/47

Page 50: Ευκλειδης Β 66

Μαθη ματικά για την Β ' Λυκείου

{ΟΑΓ) = {ΟΒΔ) = _!__ Ι α + β Ι , όπου Ο η

2 β αρχή των αξόνων.

Ο συντελεστής διεύθυνσης της ΑΒ είναι: λ = β - α = αβ(β - α) = αβ .

_ _!_ + _!_ β - α β α

Οπότε η εξίσωση της γίνεται: y - α = αβ(χ + _!_ ) ή y = αβχ + α + β ( 1 ) . α Για y = Ο στην ( 1 ) έχουμε : χ = - α + β , α β

ενώ για χ = Ο έχουμε : y = α + β. Δηλαδή οι συντεταγμένες των Γ, Δ είναι: Γ( - α + β , 0), α β Δ(Ο, α + β) . Το μήκος του ΑΓ ισούται με :

1 α + β 2 2 F2 (ΑΓ)= ( - - + --) + α = - + α = α αβ β 2 �r-1_+_α_2_β_2 , ___:_____ ___ . Επισης το μήκος του ΒΔ είναι:

l β l 1 2 rr::-; (ΒΔ)= β2 + Cα + β - β) = νβ2 + α =

�1 + α2β 2 ___:_____ ___ . Επομένως ΑΓ = ΒΔ.

l β l Τα τρίγωνα ΟΑΓ και ΟΒΔ έχουν ίσες βάσεις και ίσα ύψη, την απόσταση d του Ο από την ευθεία ΑΒ, που είναι: d = Ι α + β l . Άρα

�1 + α 2β 2

(ΟΑΓ)=(ΟΒΔ)=_!_ �,-----1 +-α-2β-2 . Ια+β l =_!_Ι α+β Ι . 2 l β l �1 +α2β2 2 β

--+ --+

5) Αν τα α, β είναι δύο μη μηδενικά δια-νύσματα, τότε να δειχθεί ότι: i . Η εξίσωση :

--+ --+ --+ --+ --+ --+ --+ --+ --+---+

{ lα-t-β i -Ι Ι α Ι - 1 β Ι Ι )�{Iα-t-β i - Ι α Ι - 1 β l)yt-αβ =Ο {1 ) παριστάνει πάντα ευθεία με συντελεστή διεύθυνσης {όταν υπάρχει) μη αρνητικό.

--+ --+

i i . Αν α t .J.. β τότε η {1) είναι παράλλη-λη του άξονα χχ ' .

--+ --+

ί ί ί . Αν α tt β τότε η {1) είναι παράλ-

ληλη του άξονα yy'. i ν . Όταν η {1 ) διέρχεται από το σημείο

--+ --+

0{0, Ο) τότε: α _l β .

ι. Για να παριστάνει η ( 1 ) ευθεία αρκεί : --+ --+ --+ --+ --+ --+ --+ --+

l α+ β l - l l α l - l β l l -:/: Ο ή Ι α+ β i - Ι α l - l β l -:/: Ο . --+ --+ --+ --+ --+ --+

Ισχύει όμως: Ι Ι α Ι - Ι β 1 1 � Ι α+ β Ι � Ι α l + Ι β I => ----t � � ----t --+ ----t ----t ----t

Ι α+ β i - Ι Ι α Ι - Ι β l l � ο και Ι α+ β i - Ι α i - Ι β l � ο, χωρίς να μπορεί να είναι συγχρόνως και τα δύο μηδέν, αφού τα διανύσματα θα έπρεπε να είναι συγχρόνως ομόρροπα και αντίρροπα ή κάποιο από αυτά μηδενικό . . Επομένως η ( 1 ) είναι πά­ντα εξίσωση ευθείας. Επίσης επειδή οι συντε­λεστές των μεταβλητών χ και y είναι ετερόση­μοι ή μηδέν, ο συντελεστής διεύθυνσης της ευ­θείας (όταν υπάρχει) θα είναι μη αρνητικός.

--+ ---t ---t ---t ---t ---t

i i . Αν α t.J.. β τότε l l α l - l β l l = l α+ β l άρα η ( 1 ) είναι ευθεία παράλληλη του άξονα χχ ' .

---t ---t ---t ---t ---t ---t

i i i . Αν α tt β τότε Ι α+ β i = Ι α l + l β l οπότε η ( 1 ) είναι ευθεία παράλληλη του άξονα yy ' . iν. Όταν η ( 1 ) διέρχεται από την αρχή των α­ξόνων επαληθεύεται για χ = y = Ο. Συνεπώς

--+ --+ --+ --+

ισχύει: α β = Ο , άρα α _l β .

6 ) Δίνεται η εξίσωση : {2λ + 1)χ - {λ - 1)y - 3 = Ο, λ e 9i {1 ).

ί. Να αποδείξετε ότι η {1) παριστάνει ευ­θεία για κάθε λε9i και όλες αυτές οι ευ­θείες διέρχονται από σταθερό σημείο Μ.

ίί. Ποια από τις ευθείες αυτές απέχει την μέγιστη απόσταση από την αρχή των αξόνων.

ίίί. Αν η {1) τέμνει τους άξονες στα ση­μεία Α και Β, να βρεθεί το λ ώστε το Μ να είναι μέσον του τμήματος ΑΒ.

i) Για να παριστάνει ευθεία η ( 1 ) για κάθε λ ε 9i, αρκεί 2λ + 1 -:/: Ο ή λ + 1 -:/: Ο .

Αυτό ισχύει αφού 2λ + 1 = Ο <=> λ = - _!_ 2

και λ + 1 = Ο <=> λ = - 1 .

Εξετάζουμε αν υπάρχει σημείο Μ(χο, Υ ο) τέ­τοιο ώστε για κάθε λε9i να ισχύει

ΕΥΚΛΕΙΔΗΣ Β' 66 τ.2/48

Page 51: Ευκλειδης Β 66

Μαθη ματικά για την Β ' Λυκείου

(2λ+ 1 )χο - (λ - 1 )yο - 3 = 0 (2) . (2) � (2χο - Υο)λ + χο + Υο - 3 = Ο, για {2χ 0 - Υ ο = Ο } κάθε λε9{ �

_

� Χ ο + Υο - 3 - 0 {3χ 0 = 3 } � {Χ ο = 1 } Άρα όλες οι ευθείες Υο = 2χ ο Υ ο = 2 διέρχονται από το Μ( 1 , 2) . i i) Η απόσταση του 0(0, Ο) από την ( 1 ) είναι: d = 1 (2λ + 1)0 - (λ - 1)0 - 31 = 3

�(2λ + 1) 2 + (λ - 1) 2 -J5λ2 + 6λ + 2 Η μέγιστη τιμή της d βρίσκεται μόνο όταν το τριώνυμο 5λ2 + 6λ +2 παίρνει την ελάχιστη τι­μή του . Επειδή το λ2 έχει θετικό συντελεστή , θα παρουσιάζει ελάχιστη τιμή μόνο όταν λ= - � = _ i _ Αντικαθιστώντας στην ( 1 ) , η 2 · 5 5 ζητούμενη ευθεία είναι η : - χ + 8y - 1 5 = Ο . iii) Για λ =F -1 και λ =F _ __!_ η ( 1 ) τέμνει τον 2 άξονα χχ ' στο Α με ΥΑ = Ο και ΧΑ = -3- , 2λ + 1

3 ενώ τον yy ' στο Β με ΧΒ = Ο και ΥΒ = -- . 1 - λ

3 -- + 0

1 = 2λ + 1 2

_3_ + 0 2 = 1 - λ

2

� {4λ + 2 = 3}� jλ = �) -4 - 4λ = 3 λ = _!_

4

Άρα μόνο για λ = __!_ το Μ είναι μέσον του ΑΒ . 4

7) Αφού δειχθεί ότι η εξίσωση : χ2 + y2 + 2xy - 4χ - 4y + 3 = Ο παριστάνει δύο ευ­θείες παράλληλες μεταξύ τους, να υπο­λογισθεί κατόπιν το εμβαδό του τετρά­πλευρου που σχηματίζεται από τις πα­ραπάνω ευθείες και τους άξονες.

Η ε1ίσωση γράφεται και ως εξής: (χ + y) - 4(χ + y) + 3 = Ο ( 1 ) . Το τριώνυμο που προκύπτει ως προς t = χ + y έχει δύο ρίζες

τις : 1 , 3 , άρα: ( 1 ) � (x + y - 1 )(x + y - 3) = 0 � χ + y - 1 = Ο ή χ + y - 3 = Ο . Επομένως η αρχική εξίσωση παριστάνει τις ευθείες (ε 1 ) : x + y - 1 = 0 ή (εz) : x + y - 3 = 0, που επειδή έχουν τον ίδιο συντελεστή διεύθυνσης, λ = - 1 , είναι παράλληλες μεταξύ τους.

Στην (ε 1 ) για χ = Ο έχουμε y = 1 , ενώ για y = Ο είναι χ = 1 , δηλαδή τα σημεία τομής της με τους άξονες είναι A( l , Ο) και Β(Ο, 1 ) . Αντίστοιχα στην ( εz) για χ = Ο έχουμε : y = 3 και για y = Ο είναι: χ = 3 , που σημαίνει ότι τέμνει τους άξονες στα Γ(3 , Ο) και Δ(Ο, 3) . Ε­πειδή τα ζεύγη (Α, Γ) και (Β, Δ) βρίσκονται στους ημιάξονες Οχ, Oy αντλιστοιχα και ΑΒ//ΓΔ, το ΑΒΔΓ είναι τραπέζιο . Τα μήκη των βάσεων του είναι: (ΑΒ) = �(1 - 0) 2 + (1 - 0) 2 = J2 και (ΓΔ) = �(3 - 0) 2 + (3 - 0) 2 = 3-fi. Το ύψος του τραπεζίου, δηλαδή η απόσταση των πα­ραλλήλων ευθειών, από εφαρμογή του σχολι-κού βιβλίου είναι: υ = β - 1 1 = J2 . Επομέ--J1 z + 1 z

νως (ΑΒΓ Δ) = (3-Ji + ..fi) J2 = 4. 2

8) Θεωρούμε ένα σημείο Α που ανήκει στην ευθεία (ε) : χ - y + 1 = Ο. Επίσης δί­νεται και η ευθεία (ε ') : χ + 2y - 2 = Ο. i) Να βρεθεί η εξίσωση της κάθετης

από το Α στην (ε '). ii) Να δειχθεί ότι το συμμετρικό του Α

ως προς την (ε ·) κινείται σε ευθεία.

i) Έστω Α(χο, yo) . Αφού ανήκει στην ευθεία (ε) θα είναι: Α(χο, χσ+ 1 ) . Για τον συντελεστή λκ της κάθετης στην (ε ' ) θα ισχύει ότι: λκ· (-Υz) = -1 => λκ = 2. Άρα η ζητούμενη κά­θετος είναι (κ) : y - (χο + 1 ) = 2(χ - χο) δηλα­δή (κ) : y = 2χ - Χο + 1 .

"-, ·

ΕΥΚΛΕΙΔΗΣ Β' 66 τ.2/49

Page 52: Ευκλειδης Β 66

Μ αθηματικά για την Β ' Λυκείου

ii) Οι συντεταγμένες του σημείου τομής Β, της (ε ' ) με την (κ) είναι η λύση του συστήματος : {χ + 2y = 2 } (Σ) . Έχουμε : 2x - y = χ0 - 1

{x + 2y = 2 } {χ = � Χο } (Σ) <=> <=> . 4x - 2y = 2X0 - 2 - � Υ - Χο 5

Άρα Β( � χ0 , 4 χ 0 ) . Αν θέσουμε ως Α '(χ ι , Υ ι ) 5 5 το συμμετρικό του Α ως προς την (ε ' ) τότε το Β είναι το μέσον του ΑΑΌ Επομένως ισχύει: 2 χ 0 + Χ ι 1 - χ 0 = � Χ ι = -- χ 0 και 5 2 5 4 _ χ σ + 1 + Υ ι _ i _ 1 Χ ο - � Υ ι - Χ ο · 5 2 5

Απαλείφοντας την μεταβλητή χ0 παίρνουμε ότι: yι = -3χ ι - 1 . Δηλαδή το Α ' κινείται στην ευθεία: 3χ + y + 1 = Ο .

9) Δύο ευθείες με συντελεστές διεύθυνσης αντίθετους αριθμούς σχηματίζουν γωνία 60° και τέμνονται στο σημείο (0, κ), κ ε 91. Επίσης η ευθεία που έχει συ­ντελεστή θετικό διέρχεται από το σημείο (-3, 0). Να βρεθούν: ί) Οι εξισώσεις των δύο ευθειών. ίί) Οι εξισώσεις των διχοτόμων των δύο

γωνιών που σχηματίζουν οι ευθείες.

i) Έστω ότι οι ζητούμενες ευθείες είναι: (ε ι ) : y = λχ +β ι και (εz) : y = -λχ + βz . με λ>Ο. Αφού διέρχονται από το σημείο (0, κ) θα εί­ναι: β ι = βz = κ, οπότε γίνονται (ε ι ) : λχ - y + κ = Ο και (εz) : -λχ-y+κ=Ο.

-+ -+

Ορίζουμε τα διανύσματα ν, u παράλληλα με -+ -+

τις ευθείες. Το ν // (ε ι ) με ν = (-1 , -λ) και -+ -+

το u 11 (ε2) με u = (- 1 , λ) . Συνεπώς η γωνία θ των διανυσμάτων θα είναι 60° ή 1 20° και

-+ -+ � ν u 1 - λ2 ο συν θ = -- =-- . Επομένως: θ= 60 � ,� , , � , 1 + λ2

1 - λ 2 1 -- -- � 2 - 2λ2 = 1 + λ2 � 3λ2 = 1 � 1 + λ2 2

λ= J3 . ( αφού λ>Ο) 3 Τότε η ευθεία (ε ι ) είναι η : y = J3 χ + κ. Αφού 3 διέρχεται από το (-3 , Ο) ισχύει: Ο= J3 (-3) + κ 3 � κ = .J3 . Άρα οι ζητούμενες ευθείες είναι: (ε ι ) : y = J3 χ + J3 � .J3y - x - 3= 0 και 3 (εz) : y = - J3 χ + J3 , � .J3y + x - 3 = 0. 3

1 - λ2 Ομοίως: θ= 1 20° � συνθ = - 1 /2 � -- = 1 + λ2 = - � � λ = .J3 (αφού λ>Ο), οπότε : κ = 3 J3 .

Οι ευθείες στην περίπτωση αυτή είναι οι:

(ε3 ) : J3 χ - y + 3 J3 = Ο και (ε4) : .J3 x + y - 3 .J3 = 0.

ii) Αν θεωρήσουμε ένα τυχαίο σημείο M(xo,yo) και (δ ι ) , (δz) τις διχοτόμους των γω­νιών που σχηματίζουν οι (ε ι ) και (εz) τότε: Μ ε (δ ι )υ(δz) <=> d(M, ε ι ) = d(M, εz) <=> Ι .fJΥο - Χσ - 3 1 Ι .fJΥο + Χσ - 3 1 .:....._�==------'- = <=> J3+i J3+i <=>I .J3Yo - χο - 3 1 = Ι .J3Υο + Χ ο - 3 1 <=>.J3Yo - Χ ο - 3 = ..fjyo + χ σ - 3 <=> χσ = Ο ή -.J3y0 - x0 - 3 = .J3y0 - x 0 + 3 <=> yo= .J3 .

Οι εξισώσεις των διχοτόμων είναι : (δ ι ) : χ= Ο και (δz) : y = J3 . Όμοια εργαζόμαστε για το ζεύγος των ευθειών (ε3) , (ε4) και προκύπτουν ως διχοτόμοι οι ευθείες: (δ ι ) : χ = Ο και (δz) : Υ = 3 J3 .

1 Ο) Δίνονται τα σημεία A(l , Ο) και Β(Ο, 2). Από τυχαίο σημείο Κ του άξονα yy' φέ­ρουμε παράλληλη στην ΑΒ που τέμνει το άξονα χχ ' στο σημείο Γ και κάθετη στην ΑΒ που την τέμνει στο σημείο Δ.

ΕΥΚΛΕΙΔΗΣ Β ' 66 τ.2/50

Page 53: Ευκλειδης Β 66

Μ αθηματικά για την Β ' Λυκείου

Να βρεθεί η εξίσωση της γραμμής πάνω στην οποία κινείται το μέσον Μ του τμήματος Γ Δ.

Ο συντελεστής της ΑΒ είναι λΑ8 = -2 και η εξίσωση της είναι: y= -2χ + 2. Αφού ΚΓ // ΑΒ θα ισχύει λκr = λΑΒ = -2 και αν θεωρή­σουμε ως Κ(Ο, yo) , yo ε!R, το μεταβλητό ση­μείο, τότε η ΚΓ είναι: y - Υο = -2χ ( 1 ) . Θέτοντας στην ( 1 ) y = Ο έχουμε χ=� , άρα 2 Γ( Ι2_ , 0) . Εξάλλου : ΚΔ l_ ΑΒ, οπότε αν λκΔ 2 είναι ο συντελεστής διεύθυνσης της ΚΔ, τότε :

Η εξίσωση λοιπόν της ΚΔ είναι: y-y0=Yz χ.

Οι συντεταγμένες του σημείου Δ θα είναι η λύση του συστήματος των δύο ευθειών. Ε­πομένως έχουμε : {Υ = -2χ + 2 } { 1 } - x + y = -2χ + 2

1 � 2 ο � y = 2 X + Yo y = -2x + 2

ΒΙ ΒΛΙΑ Π Ο Υ ΛΑΒΑΜΕ : Σ. Π. ΖΕΡΒΟΣ

ΚΑΘΗΓΗΤΗΣ

ΠΑΝΕΠΙΣΤΉΜΙΟΥ ΑΘΗΝΩΝ

'" Β

χ

Το σημείο Μ(χ ι , Υ ι ) ως μέσον του θα έχει συντεταγμένες ίσες με :

4 - 2yo Υο ----'- + -5 2 Υο + 8 χ 1 = -�-----==-- = ---=--- και 2 20

4yo + 2 + 0 5 2y0 + 1 Υ I = --=---- = 2 5

ΓΔ

Απαλείφοντας την παράμετρο yo από τις δύο σχέσεις έχουμε : 40χ 1 - 1 6 = 5y 1 - 1 . Άρα το μέσον Μ του Γ Δ ανήκει στην ευθεία με ε­ξίσωση : 8χ - y - 3 = Ο .

ΠΕΤΡΟΣ Β. ΚΡΙΚΕΛΗΣ

ΛΕΚΤΩΡ

ΠΑΝΕΠΙΣΤΉΜΙΟΥ ΑΘΗΝΩΝ

ΠΩΣ ΜΕΤΑΒΑΙΝΟ ΥΜΕ ΑΠΟ ΤΑ ΚΛΑΣΙΚΆ ΜΑ ΘΗΜΑ ΤΙΚΆ ΣΤΑ ΝΕΏΤΕΡΑ

«Με αρχικές γνώσεις μόνο της Β ' Λ υκείου, φτάνει στα σύνορα Έρευνας»

Μαθη ματικά: σελίδες 3 84 πρόλογος και συνοπτικό ιστορικό

Νεοελληνικών Μαθη ματικών,

και αρκετές φωτογραφίες. Έκδοση δεύτερη , επαυξη μένη . Αθήνα, 2007 Κεντρική πώληση : Ετόϊσα, Ε. Μπενάκη 42, τηλ. : 2 1 0 3839739 Τιμή 1 8€

ΕΥΚΛΕΙΔΗΣ Β ' 66 τ.2/51

Page 54: Ευκλειδης Β 66

•-•ιιιι-�ι-• ,.. nι• r ""'' ,., Α•••Ι••

Α. ΠΡΟΒΛΗΜΑ Τ Α ΕΥΡΕΣΗΣ ΜΕΓΙΣΤΗΣ ΚΑΙ ΕΛΑΧΙΣΤΗΣ ΤΙΜΗΣ ΜΙΑΣ ΣΥΝΑΡΤΗΣΗΣ

Α. Ε ΙΣΑ ΓΩΓΉ

Ολοκληρώνοντας το 1 ο κεφάλαιο στα Μαθημα­τικά της Γενικής Παιδείας της Γ ' Λυκείου συνειδη­τοποιούμε ότι ο στόχος του ήταν η αντιμετώπιση προβλημάτων εύρεσης της μέγιστης ή της ελάχι­στης τιμής μιας συνάρτησης (ενός μεγέθους) . Επί­σης και της τιμής της μεταβλητής, από την οποία εξαρτάται η συνάρτηση (το μέγεθος), για την οποία παρουσιάζει τη μέγιστη ή τη ελάχιστη τιμή αυτή .

Στα προβλήματα αυτά τα βήματα που ακολου­θούμε για την επίλυσή τους είναι:

ι '"') Κατασκευάζουμε τη συνάρτηση του μεγέ­θους που θέλουμε να μεγιστοποιήσουμε ή να ελαχι­στοποιήσουμε ως προς τη μεταβλητή της οποίας η τιμή ζητείται. Α ν στο πρόβλημα υπεισέρχονται και άλλες μεταβλητές εκφράζουμε αυτές ως συνάρτηση της ζητούμενης. Στο βήμα αυτό προσδιορίζουμε και το πεδίο ορισμού της συνάρτησης αυτής.

2"') Με τη βοήθεια των «γνώσεών» μας, αλλά κυρίως των θεωρημάτων του Διαφορικού Λογι­σμού προσδιορίζουμε την τιμή της μεταβλητής στην οποία η συνάρτηση παρουσιάζει την ακρότα­τη τιμή της καθώς και την τιμή αυτή .

Όταν αναφέρουμε: «γνώσεις» εννοούμε αυτές που προέρχονται από τις πρώτες τάξεις του Λυκεί­ου όπως είναι: η γραφική παράσταση, η μέγιστη

και ελάχιστη τιμή των Τριγωνομετρικών συ­

ναρτήσεων κ.α. (βλέπε παράδειγμα 1 και 2) . Όμως υπάρχουν προβλήματα στα οποία οι

γνώσεις αυτές δεν επαρκούν. Για την αντιμετώπι-

του Κώστα Βακαλόπουλου

σή τους μάθαμε το παρακάτω θεώρημα. Θ ΕΩ Ρ Η Μ Α ( K_Qt'!!li>ω ι 'Ε παραγώγου) .

Έστω συνάρτηση f παραγωγίσιμη στο (α, β) και χ0ε (α, β) με Γ(χο)=Ο.

• Αν f'(x) > Ο στο (α, χ0) και f'(x) < Ο στο (χ0, β) τότε η f παρουσιάζει στο χο τη μέγιστη τιμή της στο (α, β)

• Αν Γ(χ) < Ο στο (α, χ0) και Γ(χ) > Ο στο (χο,β) τότε η f παρουσιάζει στο χ0 την ελάχιτη τιμή της στο (α, β) Βάσει του παραπάνω θεωρήματος, για να

προσδιορίσουμε τα ακρότατα της συνάρτησης που είναι παραγωγίσιμη στο (α, β) αρκεί:

ι "'Ό Να βρούμε τις τιμές της μεταβλητής χ

στις οποίες μηδενίζεται η παράγωγος της συνάρτησης, επιλύοντας την εξίσωση : Γ(χ) = Ο

2"'" . Να προσδιορίσουμε το πρόσημο της πα­ραγώγου συνάρτησης εκατέρωθεν του σημείου μηδενισμού, επιλύοντας τις α­νισώσεις : f' (x) > Ο και f'(x) < Ο.

Ανάλογα με τον τρόπο αλλαγής του πρόσημου της f' εκατέρωθεν των σημείων μηδενισμού χαρακτη­ρίζουμε τα ακρότατα μέγιστα ή ελάχιστα στο (α, β).

π.χ. Να βρεθούν τα ακρότατα της συνάρτη­

σης και να προσδιοριστεί το είδος τους:

f(x) = 2χ3 + 1 5χ2 + 36χ - 2008, x ε iR

Λ ΥΣΗ

Έχουμε : Γ(χ)=6χ2 - 30χ+36=6(χ2 - 5χ+6), xεiR • f' (x) = Ο <::::> χ2-5χ+6 = Ο <::::> χ = 2 ή χ = 3

ΕΥΚΛΕΙΔΗΣ Β ' 66 τ.2/52

Page 55: Ευκλειδης Β 66

Μαθη ματικά για την Γ Λυκείου

• f' (x) > Ο <::::> χ2-5χ+6 > Ο <::::> χ < 2 ή χ > 3 • f' (x) < Ο <::::> χ2-5χ+6 < Ο <::::> 2 < χ < 3

χ -00 2 3 +οο

6 6 f' (x) + - + Ί I

f / τ.1μ.� τ !ε./ Άρα η συνάρτηση παρουσιάζει μέγιστο τοπικό

στο 2 το f(2) = - 1 950 και ελάχιστο (τοπικό) στο 3 το f(3 ) = - 1 98 1 .

1 3 2 β) Αν f(x) = - χ - 2χ + 4χ - 2008 x EIR τό-3

τε f '(x) = χ2 - 4χ + 4 = (χ - 2)2 • Επειδή Δ = 1 6 - 1 6 = Ο ισχύει f'(x) > Ο για

κάθε xEIR-{2 } ( f'(2) = 0 ) .

I �χ) I � +

2

λρα η συνάρτηση f δεν παρουσιάζει ακρότα­τεc; τιμέc; στο R.

ΑΞ Ι ΟΛΟΓΕΣ Ε Π Ι � Η Μ Λ :\ Σ F: η.: 2 '1 • Μια συνάρτηση μπορεί να παρουσιάζει ακρό-

1 •ι Αν υπάρχει χ0 Ε ( α, β ) c Ar (Ar το πεδίο ορι- τατες τιμές και σε άλλα σημεία εκτός από αυτά στα οποία μηδενίζεται η παράγωγος και εκατέρω­

σμού της συνάρτησης t) στο οποίο η f' μηδενίζεται θέν τους αλλάζει πρόσημο !

τότε η f δεν παρουσιάζει οπωσδήποτε ακρότατη ιι_----=-----='----'---'-'-----------Τέτοια σημεία μπορεί να είναι: τιμή σ ' αυτό.

3 IR

, Ι 1 1 " . Τα σημεία στα οποία η συνάρτηση δεν πα-π.χ. Για τη συνάρτηση f(x)=x , ΧΕ ισχυει:

2 ραγωγίζεται. f'(x) = 3χ , x EIR και f'(O)=O ενώ η συνάρτηση f

π.χ. Δίνεται η συνάρτηση : f(x) = I χ- 1 I , xEIR. δεν παρουσιάζει στο Ο ακρότατη τιμή.

Στο χ0 = 1 η συνάρτηση παρουσιάζει την ελά-

χ

Γενικά :

Μια παραγωγίσιμη συνάρτηση f στο R, δεν παρουσιάζει ακρότατη τιμή στο IR

• αν f'(x) i- Ο για κάθε x EIR ή • αν μηδενίζεται ίσως η f' σε κάποια σημεία,

εκατέρωθέν τους να διατηρεί σταθερό πρόσημο. Ε ιδ ικ{ι , για τις πολυωνυμικές συναρτήσεις 3ου

βαθμού, δηλαδή για τις συναρτήσεις της μορφής: f(x) = αχ3 + βχ2 + γχ + δ, α, β, γ, δ EIR με α i- Ο με Γ(χ) = 3αχ2 + 2βχ + γ (τριώνυμο) ισχύει ότι:

Δεν παρουσιάζουν ακρότατες τιμές αν η δι­

ακρίνουσα: Δ = 4β2 - 12αγ της παραγώγου τους

είναι μικρότερη ή ίση με το μηδέν». Π αραδείγματα

α) Αν f(x) = χ3 + 4χ2 + 7χ + 1 x EIR τότε f'(x) = 3χ2 - 8χ + 7 Επειδή Δ = 64 - 84 = - 20 < Ο ισχύει f' (x) i- Ο για κάθε xEIR. Άρα η συνάρτηση f

δεν παρουσιάζει ακρότατες τιμές στο R.

χιστη τιμή της (f( l )=O) ενώ δεν παραγωγίζεται σ ' αυτήν.

ο 1 χ

2 1 1' · . Τα άκρα του πεδίου ορισμού της 2 π.χ. Δίνεται η συνάρτηση : f(x) = χ με χ :<:; 1 .

Η συνάρτηση παρουσιάζει στο 1 (τοπικό) μέ­γιστο ενώ f' (x) = 2χ, χ :<:; 1 οπότε f' ( l ) = 2 i- Ο

Σημεί(Ι)ση:

\z ο 1 χ

Στα Μαθηματικά της Γενικής Παιδείας αντιμε­

τωπίζονται συναρτήσεις που είναι παραγωγίσιμες

στο πεδίου ορισμού τους, το οποίο συνήθως δεν έχει

κλειστά άκρα.

ΕΥΚΛΕΙΔΗΣ Β ' 66 τ.2/53

Page 56: Ευκλειδης Β 66

Μαθηματικά για την Γ Λυκείου

Έτσι η εύρεση των ακρότατων τιμών περιορίζε­

ται στην εφαρμογή του θεωρήματος που προαναφέ­

ραμε.

Β. Λ Υ Μ Ε�Α Π Α ΡΑ Λ Ε Ι ΓΜΑΤ Α (Ας Παρακο­

λουθήσουμε τη μεθοδολογία που ακολουθούμε

στην επίλυσή τους Ι)

Π ΡΟ ΒΛ Η Μ Α 1

Ένας aνθοπώλης σε μια ανθοκομική έκθεση

πρέπει με 64μ. φράκτη (που διανέμεται στην εί­

σοδο της έκθεσης) να φράξει ένα χώρο σχήμα­

τος ορθογώνιου, χρησιμοποιώντας ως μια πλευ­

ρά τον τοίχο της έκθεσης, για να εκθέσει τα

λουλούδια του.

...:..ι.«=?<'""lί"'(('""( <;..;.«;..;.«::.::.«::.::.«::.::.«::.::.«::.::.«�1"'"':::.::.«:..;.::;« χ

Προφανώς θέλει να σχηματίσει ορθογώνιο

με το μεγαλύτερο δυνατό εμβαδόν. Μπορείτε να

τον βοηθήσετε;

ΛΥΣΗ

Έστω x,y οι διαστάσεις του «βέλτιστου» ορ­θογώνιου . Το εμβαδόν του ορθογώνιου είναι: E(x,y)= x .y (συνάρτηση δυο μεταβλητών) .

Ό 64 - χ

μως: χ + 2y = 64 <::::> y =--<=> 2

χ χ Υ = 32 - 2 ( y > Ο <=> 32 - 2 > Ο <=> χ < 64 ) .

Άρα:

Ε (χ ) = χ (32 - ; ) = -Ξ χ 2 + 32χ με Ο < χ < 64.

• Με γνώσεις Α ' Λυκείου :

Η συνάρτηση Ε(χ) είναι τριώνυμο της μορφής: 2 1 αχ + βχ + γ με α = -2 < Ο , β = 32 και γ = Ο.

Ως γνωστόν η συνάρτηση αυτή παρουσιάζει , , , β 32 μεγιστη τιμη μονο στο χ 0 = -- = = 32 .

2α 2 ( -Ξ) Υ

Άρα: Για χ = 32 ε (0, 64) και y = 32 - 32 = 1 6 2

το εμβαδόν γίνεται μέγιστο και ίσο με Δ Ε = -- = 5 1 2 τ.μ. 4α

• Με γνώσεις Γ Λυκείου : Για κάθε χε (Ο, 64) ισχύει: Ε '(χ)= - χ + 32 Ε '(χ) = Ο <=> χ = 32 Ε ' (χ) > Ο <=> χ < 32 Ε ' (χ) < Ο <=> χ > 32

χ ο 32 64

Ε ' (χ) + 6 -τ

Ε / τι � Άρα χ = 32 η συνάρτηση Ε παίρνει τη μεγαλύ­

τερή της τιμή δηλαδή το εμβαδόν γίνεται μέγιστο και ίσο με Ε(32) = 5 1 2 τ.μ .

Π ΡΟ ΒΛ Η Μ Α 2 Με δυο σχοινιά μήκους 200 μ. και 300 μ. α­

ντίστοιχα, θέλουμε να οριοθετήσουμε μια τρι­

γωνική περιοχή σε παραλία της Ζακύνθου για

την αναπαραγωγή της careta-careta. τι είδους

τρίγωνο πρέπει να σχηματίζουμε ώστε το εμβα­

δόν του να είναι το μέγιστο δυνατό; Α

χ 200μ 300μ

Β Γ

.\ΥΣ Η

Αν είναι χ η (ζητούμενη) γωνία BAr του τρι­γώνου ΑΒΓ που σχηματίζεται τότε το εμβαδόν τους δίνεται από τη συνάρτηση :

1 1 Ε ( χ ) = - · ΑΒ · ΑΓ · ημχ = - 200 · 300 · ημχ = 2 2

= 30 .000 . ημχ , ο < χ < π • Με γνώσεις Α ' και Β ' Λυκείου :

Η συνάρτηση Ε παίρνει τη μεγαλύτερή της τι­μή μόνο αν ημχ = 1 (αφού : Ο < ημχ � Ι για κάθε

χε (Ο, π)) . Όμως ημχ = 1 <::::> χ = Ξ (90° ) .

Άρα Το τρίγωνο για να έχει το μεγαλύτερο

ΕΥΚΛΕΙΔΗΣ Β' 66 τ.2/54

Page 57: Ευκλειδης Β 66

Μαθη ματικά για την Γ Λυκείου

εμβαδόν πρέπει και αρκεί να είναι ορθογώνιο . • Με γνώσεις Γ Λυκείου :

Έχουμε: Ε ' (χ) = 30 .000συνχ , Ο < χ < π

• Ε '(χ) = Ο <=:> χ = 2:. 2

• Ε '(χ) > 0 <=:> 0 < χ < 2:. 2

• Ε '(χ) < 0 <=> 2:. < χ < π 2

ο π χ 2 π

Ε ' (χ) + 6 -τ

Ε / τ!μ. � π

Άρα: Μόνο για χ = - η συνάρτηση παρου-2

σιάζει μέγιστη τιμή της, δηλαδή το εμβαδόν γίνε­ται μέγιστο.

Π ΡΟ Β Λ Η Ι\-Ι Α 3 Το υπουργείο εμπορείου επιβάλλει τα «κου­

τάκια» με τα αναψυκτικά να έχουν χωρητικό­τητα 628cm3• Να βρεθούν οι διαστάσεις τους ώστε οι βιοτεχνίες που τα κατασκευάζουν να έ­χουν το μικρότερο δυνατό κόστος;

ΛVΣΗ

Έστω χ η ακτίνα της βάσης και h το ύψος. Προφανώς το κόστος κατασκευής εξαρτάται από το πόσο υλικό (λαμαρίνα) χρησιμοποιούμε στην κατασκευή του. Δεδομένου ότι χρησιμοποιούμε λαμαρίνα συγκεκριμένου πάχους το κόστος εξαρ­τάται από τη συνολική επιφάνεια Ε του κουτιού .

Έτσι, Ε(χ, y) = 2πχ2 + 2πχ · h, χ > Ο, h > Ο (συνάρτηση δυο μεταβλητών)

Όμως αν V: ο όγκος του κουτιού θα ισχύει:

ν = 628 <=> πχ2 . h = 628 <=> h = 628 === 200 πχ 2 χ 2

(π =:: 3 , 1 4)

Άρα: Ε( χ) = 2 · 3, 1 4χ 2 + 2 · 3 , 1 4 · χ . 200 , χ > Ο χ2

(συνάρτηση μιας μεταβλητής) (Στο παράδειγμα αυτό οι γνώσεις της Α ' και

Β ' Λυκείου δεν επαρκούν για να βρούμε πότε η συνάρτηση αυτή παρουσιάζει ελάχιστο ! )

'Εχουμε: Ε '(ψ [ 2π ( χ ' + 2:)] '

= 2π ( 2χ - 2�0 ) = 4π ( χ - 1�2°} χ > 0

• Ε '(χ) = Ο <=> χ -1 0�

= Ο <=> χ3 = 1 00 <=> χ

<=> χ = ifiOO <=> χ === 4, 64 cm .

• Ε'(χ) >Ο<:::>χ- 1� >0<=:>χ3 > 100<=:>χ>4,64. χ

• Ε'(χ) <Ο<=:> . . . <=:>χ <4,64 .

χ ο 4,64 +οο

Ε ' (χ) - 6 + τ

Ε � τ � τ.ε.

Άρα: Το εμβαδόν (η επιφάνεια) άρα και το κόστος

γίνεται ελάχιστο όταν και μόνον η ακτίνα της βά­σης είναι χ === 4,64 και στο ύψος του κουτιού

200 h === 2 === 9, 29 cm . ( 4, 64 ) Γ. ΑΣ Κ Η Σ Ε Ι Σ

1 . Να βρεθεί η ευθεία που διέρχεται από το σημείο A(l,2) και σχηματίζει με τους ημιά­ξονες Οχ και Oy το μικρότερο εμβαδόν. ΛΥΣΗ

Η ζητούμενη ευθεία θα έχει εξίσωση της μορ­φής: Υ = λχ + β ( 1 ), λ < Ο, β < Ο (αφού ω>90°) (η περίπτωση να μην έχει συντελεστή διεύθυνσης (//yy ') απορρίπτεται

Επειδή διέρχεται από το Α θα ισχύει: 2 = λ + β <=> β = 2 - λ.

Έτσι η μορφή ( 1 ) γίνεται: y = λχ + 2 - λ (2) Για τα σημεία Κ(χ ι , Ο) και Λ(Ο, y2) που η ευ­

θεία ε τέμνει τους ημιάξονες ισχύει: λ - 2 Ο = λ · χ 1 + 2 - λ <=> λχ 1 = λ - 2 <=> Χ 1 = -­λ

ΕΥΚΛΕΙΔΗΣ Β ' 66 τ.2/55

Page 58: Ευκλειδης Β 66

Μαθη ματικά για την Γ Λυκείου

(προφανώς λ i- 0). Επίσης: Υ2 = λ · Ο + 2 - λ <=> Υ2 = 2 - λ

Άρα: κ ( λ � 2 , Ο} Λ (Ο, 2 - λ)

ο χ

Το εμβαδόν του τριγώνου ΟΚΛ που σχηματί­ζεται δίνεται από τη συνάρτηση :

Ι λ - 2 Ι (λ - 2)2 Ε (λ) = -2 · -λ-(2

- λ) <=> Ε(λ) = -2 · λ

, λ < Ο

'Εχουμε Ε'(λ) = _.!. . 2(λ - 2)λ - (λ - 2)2 = 2 λ2

Ι 2λ2 - 4λ - λ2 + 4λ - 4 2 λ2

Με λ < Ο έχουμε: • Ε '(λ) = Ο <::::> λ2 - 4 = Ο <::::> λ = -2 • Ε ' (λ) > Ο <::::> λ2 - 4 < Ο <::::> 2 < λ < Ο • Ε ' (λ) < Ο <::::> λ2 - 4 > Ο <::::> λ < - 2

-00 -2 ο

Ε '(λ) + 6 -Ί

Ε �τ.1ε. /

Άρα: Μόνο για λ = - 2 η ευθεία y = - 2χ + 4, σχηματίζει με τους ημιάξονες τρίγωνο με το μι­κρότερο εμβαδόν. 2. Ποιες διαστάσεις πρέπει να έχει το «τελάρο»

ΜΝΚΛ μιας ορθογώνιας κορνίζας ώστε να έχει τη μεγαλύτερη δυνατή επιφάνεια και οι κορυφές του Μ, Ν να ακουμπούν στο παρα­βολικό άνοιγμα ενώ η βάση του στο δάπεδο. (Η παραβολή του σχήματος είναι η γραφική

2 παράσταση της συνάρτησης f(x) = - χ + 9).

Υ

κ ο Λ χ

ΛΥΣ Η 2 Έστω Μ(χ, y) με y = - χ + 9 η μια κορυφή

του ορθογώνιου που σχηματίζεται , 2 Άρα: Μ( χ, - χ + 9) με Ο < χ < 3 Η επιφάνεια της κορνίζας δηλαδή το εμβαδόν

του ορθογωνίου δίνεται από τη συνάρτηση 2 3

Ε(χ) = 2χ(- χ + 9) <::::> Ε(χ) = -2χ + Ι 8χ, Ο < χ < 3

Ε ' (χ) = - 6χ2 + Ι 8 = - 6(χ2 - 3), Ο < χ < 3 Με χε(Ο, 3) έχουμε: • Ε '(χ) = Ο <::::> χ = .J3 • E '(x) > O <::::> x2 - 3 < 0 <::::> 0 < x < .J3 • Ε '(χ) < Ο <::::> χ2 - 3 > Ο <::::> .J3 < χ < 3

ο ..J3 3

Ε ' (χ) + 6 -τ

Ε / τ.ι �

Άρα: Τη μεγαλύτερη επιφάνεια θα έχει η κορ­νίζα με διαστάσεις 2.J3 και 6 .

( ΚΛ = 2 · χ = 2.J3 , ΛΜ = -χ 2 + 9 = 6 ) 3. Έστω ορθογώνιο ΑΒΓ Δ στις πλευρές του

οποίου εφαρμόζουμε εξωτερικά ημικύκλια διαμέτρου ίση με κάθε πλευρά του. Α ν η πε­ρίμετρος του σχήματος είναι 400 μ. δείξτε ότι το ορθογώνιο αποκτά το μεγαλύτερο εμ­βαδόν όταν γίνει τετράγωνο !

Α 2χ

2y

Δ Γ

ΛΥ2:Η

Έστω ΑΒ = 2χ και Γ Δ = 2y (χ, y > Ο) Η περίμετρος του σχήματος είναι 2πχ + 2πy

ΕΥΚΛΕΙΔΗΣ Β ' 66 τ.2/56

Page 59: Ευκλειδης Β 66

Μαθη ματικά για την Γ Λυκείου

Άρα: 2πχ + 2πy = 400 200 - πχ 200

<=> y = <=> y = - - x ( 1 ) π π

200 ( y > Ο <=> 200 - πχ > Ο <::::> χ < - ) π

• Το εμβαδόν του ορθογωνίου είναι: ( Ι ) ( 200 ) 800 E = 2x · 2y = 4xy = 4x -;- - χ = -4χ2 + -;- χ ,

200 Ο < χ < - .

π

' 800 200 Άρα: Ε (χ ) = -4χ2 + - χ, Ο < χ < -π π

Έχουμε : ' 800 ( 200 ) Ε ( χ ) = -8χ + --;- με χ ε Ο,--;-

έχουμε :

' ( 1 00 ) 1 00 Ε ( χ ) = Ο <=> -8 χ - --;- = Ο <::::> χ = --;-

, ( 1 00 ) 1 00 Ε ( χ ) > Ο <=> -8 χ ---;- > Ο <::::> Ο < χ < --;-

, ( 1 00) 1 00 200 Ε (χ) < Ο <::::> -8 χ -- < 0 <::::> - < χ < -π π π

Για

ο 1 00 200 - -π π

Ε ' (χ) + 6 -I

Ε / )μ. � 1 00 χ = - οπότε π

200 1 00 1 00 y = - - - = - το π π π

ορθογώνιο γίνεται τετράγωνο και έχει το μεγαλύ­τερο εμβαδόν του.

4. Θέλουμε να διαμορφώσουμε ένα θερινό θε­ατράκι σχήματος κυκλικού τομέα ακτίνας ρ και γωνίας θ με επιφάνεια 225τ.μ. Να βρεθεί η ακτίνα ρ ώστε η περίφραξη να στοιχίσει το λιγότερο δυνατό.

ΛΥΣΗ

• Α ν Ε το εμβαδόν έχουμε : θ 2 θρ2 2Ε Ε = -πρ <::> Ε = - <=> θ = - . 2π 2 ρ2

Για Ε = 225 έχουμε: θ = 452°

, ρ > Ο ( 1 ) ρ

• Αν Π η περίμετρος τότε : θ ( Ι ) π = 2ρ + -2πρ = 2ρ + θρ = 2π 450 450

= 2ρ + -2 ρ = 2ρ + -, ρ > Ο ρ ρ

'Ετσι: Π (ρ) = 2ρ + 450 , ρ > Ο είναι η συνάρτη­ρ

ση του δίνει την περίμετρο Π συναρτήσει της α­κτίνας ρ.

Έχουμε :

Π(ρ) = 2 - 450 = 2[ 1 - 225 ) = 2 ρ2 - 225 ρ2 ρ2 ρ2

Με ρ > Ο έχουμε :

• π (ρ) = ο <=> ρ2 - 225 = ο <=> ρ = 1 5 • π (ρ) > ο <=> ρ2 - 225 > ο <=> ρ > 1 5 • π (ρ) < ο <=> ρ2 - 225 < ο <=> ο < ρ < 1 5

χ ο 1 5 +οο

Π(ρ) - 6 + I

π � τ !ε. / Άρα :

Για να έχουμε το ελάχιστο κόστος πρέπει και αρκεί η ακτίνα να είναι 1 5 μέτρα.

ΕΥΚΛΕΙΔΗΣ Β' 66 τ.2/57

Page 60: Ευκλειδης Β 66

Μαθηματικά για την Γ Λυκείου

Β. ΣΤΑΤΙΣΤΙΚΉ του Κώστα Βακαλόπουλου

Στο άρθρο αυτό θα παρουσιάσουμε μια μικρή συλλογή ασκήσεων οι οποίες καλύπτουν τις έννοιες που

υπάρχουν στο κεφάλαιο της Στατιστικής.

Σε κάθε άσκηση αναφέρεται το κομμάτι της θεωρίας που αντιστοι­

χεί. Όπου χρειαστεί σχολιάζουμε . . .

ΑΣ Κ Η Σ Η J 'l .

(ΣΥΧΝΟΤ Η Τ ΕΣ - Λ Ι Α Γ ΡΑ Μ Μ ΑΤΑ)

Έστω χ1 , χ2, χ3, χ4 οι τιμές μια μεταβλητής · 2

Χ με σχετικές συχνότητες: f; = -1- , ί=1 ,2,3,4 5κ

α) Να προσδιοριστεί η τιμή του κ β) Αν ν4 = 64 (απόλυτη συχνότητα) να βρεί­

τε το μέγεθος v του δείγματος γ) Να χαράξετε το κυκλικό διάγραμμα της

παραπάνω κατανομής. ΛΥΣΗ

α) Ισχύει: ltr, = 1 1 (δηλ. f, + f, + f, + f, = I )

Άρα 1 2 22 32 42 1 + 4 + 9 + 1 6 - + - + - + - = 1 <=:> = 1 <=:> 5κ 5κ 5κ 5κ 5κ <=:> 5κ = 30 <=:> κ = 6 .

1 4 9 1 6 (Ετσι: f1 = - , f2 = - , f3 = - , f4 = - ) . 30 30 30 30

β) Ισχύει: l f; = � I · i = 1 ,2,3 ,4 Άρα:

f = v 4 <=> .!.i =

64 <=> v = 30 . 64 <=> v = 1 20 4 v 30 ν 1 6 γ) Ισχύει: α; = f; · 360° , i = 1 ,2,3 ,4

:ti α; 1 /30 1 2° 4/30 48° 9/30 1 08° 1 6/30 1 92°

Κυκλικίι δ ι ιί:γραμμα κατανομι]ς συzνοη]των. ΑΣ Κ Η Σ Η 2 '1

1 20

(Δ Ι Α Γ ΡΑ Μ VΙ ΑΊΆ- Π Ι Ν Α ΚΑΣ ΚΑΊΆ Ν Ο Μ Η Σ

ΣΥΧΝΟΤΉΤΩΝ- Μ Ε'Γ ΡΑ ΘΕΣ Η Σ)

Στον πίνακα παρουσιάζεται η κατανομή του αριθμού των ημερών που παρέμειναν οι ένοικοι

σ' ένα ξενοδοχείο της Κέρκυρας την φετινή κα-λοκαιρινή σαιζόν.

Α ριθμός ημερών Αρ ιθμός Ενο ίκων

0-4 9 5-9 10

1 0- 14 1 2 1 5-19 23 20-24 2 1 25-29 17 30-35 8

Να βρεθεί: α) Η μέση τιμή των ημερών παραμονής στο

ξενοδοχείο β) Η διάμεσος των ημερών αυτών. ΑΥΣΗ

Έστω: χ ; το κέντρο κλάσεων ν; οι συχνότητες κάθε κλάσης και Ν; οι αθροιστικές συχνότητες F; % οι αθροιστικές σχετικές συχνότητες επί τοις εκατό ( i= 1 ,2, . . . ,7) α) Έχουμε τον ακόλουθο πίνακα:

Χ; V; X;V; Ν; 2,5 9 22,5 9 7,5 1 0 75 1 9 1 2 ,5 1 2 1 50 3 1 1 7 ,5 23 402,5 54 22,5 2 1 472,5 75 27,5 1 7 467,5 92 32,5 8 260 1 00

1 00 1 850 7

Σχίνί 1 850 Άρα χ = i = 17 = - = 1 8 5

1 00 ,

Σ ν; i = l

Fi% 9

1 9 3 1 54 75 92

1 00

β) Κατασκευάζουμε το πολύγωνο αθροιστικών σχετικών συχνοτήτων επί τοις εκατό.

ΕΥΚΛΕΙΔΗΣ Β' 66 τ.2/58

Page 61: Ευκλειδης Β 66

Μαθη ματικά για την Γ Λυκείου

1 00

90

80

70

60

50

40

30

20

1 0

Α · -

Ε Δ . .

-

ο 5 ι ο 1 5

· · · · · · · · · · · · · · · ·---

'

Β

20 25 30 Δ Δ

-

i I !

'

I I I

i 35

Από τα όμοια τρίγωνα Γ Α Β � Γ Δ Ε έχουμε : ΔΕ = ΑΒ Ά α : ΔΕ = ΑΒ · ΓΔ

= 5 · 1 9 :::: 4 1 3

ΓΔ ΑΓ ρ

ΑΓ 23 '

Άρα: Η διάμεσος είναι : δ = 1 5 + 4, 1 3 � 1 9 , 1 3

ΑΣ Κ Η Σ Η 3 '� ( Π Ι :\ λ ΚΕΣ ΣΥΣΧΕ τΙ Σ Η Σ)

Στον παρακάτω πίνακα καταγράφεται η η-

λικία και η ομάδα που υποστηρίζει (0: ΟΛ Υ-ΜΠΙΑΚΟΣ, Π: ΠΑΝΑΘΗΝΑΪΚΟΣ) δείγμα 40

φιλάθλων από μια κερκίδα του γηπέδου.

1 7/0 1 5/Π 1 4/Π 1 9/0 23/0 2 1 10 27/Π 1 1 /Π

1 9/0 20/Π 1 9/Π 1 7/0 1 1 /Π 1 3/0 1 3/Π 1 5/Π

2 1 /Π 1 9/0 20/Π 1 9/0 1 7/Π 1 7/0 1 8/0 1 3/0

1 9/0 20/0 2 1 /0 1 8/0 1 7/0 1 6/Π 1 6/0 1 5/0

1 4/Π 1 9/0 2 1 /Π 1 8/0 23/0 22/0 2 1 /Π 1 9/Π

α) Να συμπληρώσετε τον πίνακα:

ΟΛΥΜΠΙΑΚΟΙ

ΠΑΝΑΘΉΝΑΪΚΟΙ

ΉΛΙΚΙΑ

< 1 8 � 1 8

(ΑΝΉΛΙΚΟΙ) (ΕΝΉΛΙΚΟΙ)

β) Να βρεθεί το ποσοστό επί των ανηλίκων που

είναι Ολυμπιακοί

γ) Να βρεθεί το ποσοστό επί των Παναθηναϊ­

κών που είναι ανήλικοι

δ) Να βρεθεί το ποσοστό επί των φιλάθλων

που είναι ενήλικοι - Ολυμπιακοί

Λ ΥΣ Η

β) < 1 8

α) < 1 8 � 1 8 ο 8

ο 8 1 5 π 9

π 9 8 ΣΥΝ ΟΛΟ 1 7

Επειδή 8 : 1 7=0,4705 το ζητούμενο ποσοστό εί­ναι 47 05% , γ < 1 8 ;::: 1 8 ΣΥΝΟΛΟ

π 9 8 1 7

Επειδή 9 : 1 7=0,5294 το ζητούμενο ποσοστό εί­ναι 52,94% δ) < 1 8 � 1 8 ΣΥΝΟΛΟ

ο 8 1 5 23

π 9 8 1 7

ΣΥΝΟΛΟ 1 7 23 40

Επειδή 1 5 :40=0,375 το ζητούμενο ποσοστό εί­ναι 37 ,5%

ΑΣ Κ Η Σ Η 4'� ( Μ ΕΣ Η τ Ι Μ Η )

Η μέση ηλικία των θαμώνων ενός INTERNET CAFE στο ΜΑΡΟΥΣΙ είναι 15 χρόνια. Σήμερα το πρωί ήρθε ένας ηλικιωμένος κύριος 62 χρονών και η μέση τιμή ανέβηκε στα 15,5 χρόνια. Πόσοι ήταν οι θαμώνες του cafe πριν έρθει ο κύριος αυτός;

Λ ΥΣ Η

Αν ν ο αριθμός των θαμώνων του cafe πριν έρθει ο κύριος ισχύει:

Με την είσοδο του κυρίου στο cafe έχουμε : ν

Σ χί + 62 � , = i = ι <=> 1 5 5 =

1 5 · ν + 62 <=> ν + 1 ' ν + 1

<=> 1 5 , 5 (ν + 1 ) = 1 5 · ν + 62 <=> Ο, 5 · ν = 46, 5 <=> ν = 93

ΑΣ Κ Η Σ Η 5' � ( Μ ΕΣΗ τΙ Μ Η- Δ Ι Α ΚΥ Μ Α Ν Σ Η )

Η μέση τιμή και η διακύμανση τριών αριθ-

, ' 15 SO ' Α θ ' μων ειναι και - αντιστοιχα. ν προστε ουν 3

ΕΥΚΛΕΙΔΗΣ Β ' 66 τ.2/59

Page 62: Ευκλειδης Β 66

Μαθη ματικά για την Γ Λυκείου

δυο ακόμα αριθμοί η μέση τιμή και η διακύ­μανση γίνονται 20 και 50 αντίστοιχα. Να βρε­θούν οι δυο νέοι αριθμοί.

ΛΥΣΗ

Έστω χ 1 , χ2, χ3 ο ι τρεις πρώτοι αριθμοί και χ4 = χ, xs = y οι δυο νέοι. Θα ισχύει:

χ + χ + χ I 2 3 = Ι 5 <=:> Χ + Χ + Χ = 45 ( Ι ) 3 I 2 3

Επίσης: Ι ( 2 2 2 ) 2 50 2 2 2 - Χ 1 + Χ 2 + Χ3 - Ι 5 = - <=> χ 1 + χ 2 + χ3 = 725 (2)

3 3 (Θυμίζουμε ότι η διακύμανση :

52 = Σ(χ ί -�γ = _!_ Σ χ� - �2

ν ν , ( χι +� +x3 ) +x+y ( Ι ) 45+x+y Επισης: =20<:::> =20<:::>

5 5 <=:> x + y = 55 (3 )

Ενώ: � [( χ � + χ; + χ� ) + χ 2 + y 2 ] - 202 = 50

( 2 ) <=> 725 + χ2 + y2 = 250 + 2000 <=> χ2 + y2 = Ι 525 (4)

Λύνουμε το σύστημα των εξισώσεων (3) και (4) {x + y = 55 {x + y = 55 2 2 <=> 2 <=>

χ + y = Ι 525 ( χ + y) - 2xy = 1 525

{x + y = 55 <=> ( χ, y ) = ( 25, 30) ή ( 30, 25 ) .

xy = 750

Άρα: Οι ζητούμενοι αριθμοί είναι οι : 25 , 30 . ΛΣ Κ Ι-η:: Η 6 '� «ΤΥ Π Ο Π Ο Ι Η Σ Η τ t iVI ΩN» Έστω Χι , χ2, Χ3, . . . , Χν ν αριθμοί με μέση τι-

μή χ και τυπική απόκλιση s , . Να δείξεται ότι

αν από κάθε αριθμό αφαιρέσουμε την μέση τιμή (�) και το αποτέλεσμα της διαφοράς του κάθε

αριθμού το διαιρέσουμε με την τυπική απόκλι­ση ( s , ) θα προκύψουν κ νέοι αριθμοί με μέση

τιμή Ο και τυπική απόκλιση 1 . ΛΥΣΗ

Έστω Υ ι , Yz , y3 , . . . , Yv ο ι ν νέοι αριθμοί με μέ-ση τιμή y και τυπική απόκλιση : sy

Θ . ΕΒί - χ . 1 2 α ισχυει: Yi = - - , ι= , , . . . ,ν s ,

- Υι + y2 + . . . + yv 1 ( Χι -� Χ2 -� Χν -� • y = =- --+--+ . . . +--ν ν s, s, s,

= -1 - [ ( Χ ι + χ2 + . . . + Χ ν ) - ν . �] ν · s,

= -1- [ ν · � - ν · �] = ο Άρα: ly = o l . ν · s,

, - Χι + χ2 + . . . + χν -(Υποψιν: χ <=:>χ1 + χ2 + . . . + χν = ν · χ ) . ν

• 52 = ( Υ ι - y)

2 + ( y2 - y)

2 + . . . + (Yv - y/ Υ ν 1 ( 2 2 2 ) = - Υι + y2 + . . . + yv =

ν

Σχόλιο : Την 6'� άσκηση μπορούμε να λύσουμε και εφαρμόζοντας τα συμπεράσματα της 1 ης εφαρ­μογής σελ. 99 του σχολικού βιβλίου .

Σύμφωνα με αυτή αν Χ ι , Xz, . . . , Χν, οι τιμές μιας μεταβλητής Χ με μέση τιμή χ και τυπική α­πόκλιση : Sx και: • Yi = c · χί , (c ε!R), i = 1 ,2 . . . , ν οι τιμές μιας με­

ταβλητής Υ με μέση τιμή y και τυπική από-κλιση Sy τότε y = c · χ και Sy = lc l · sx ( 1 ) ή

• Yi = Xi + c, (c ε!R), i= 1 ,2 . . . , ν οι τιμές μιας με­ταβλητής Υ με μέση τιμή y και τυπική από-κλιση Sy, τότε y = χ + c και sy = Sx (2)

Έτσι για την άσκηση 6 έχουμε: χ ί - χ Ι χ . Ι 2 3 Yi = -- = - · x i - - , ι = ' , , . . . , ν

s, s , s x

Θεωρούμε: ωi = _!_ x i , i= I ,2 ,3 , . . . , ν αριθ­s χ

μούς με μέση τιμή ω και τυπική απόκλιση Sω

Σύμφωνα με τους τύπους ( 1 ) θα ισχύει: - Ι - χ I I I s ωί = - · Χ = - και S 00 = - · S x = ----"- = 1 .

s x s , s x s ,

Και: Yi = ωi - � . i= I ,2,3 , . . . , ν αριθμούς με s,

μέση τιμή y και τυπική απόκλιση : Sω

Σύμφωνα με τους τύπους (2) θα ισχύει:

ΕΥΚΛΕΙΔΗΣ Β' 66 τ.2/60

Page 63: Ευκλειδης Β 66

Μαθη ματικά για την Γ Λυκείου

- - χ χ χ y = ω - - = - - - = 0 και s y = sω = 1 .

s x s x s x Πριν συνε-χίσουμε θα κάνουμε ένα ακόμα σ;(όλιο:

Όταν σε μια κατανομή συχνοτήτων συμβαίνει μεταβολή στις συχνότητες (νi) των τιμών της (xi) , i= 1 .2 . . . , κ τότε επέρχεται αντίστοιχη μεταβολή και στον πληθυσμό του δείγματος.

π. χ. Αν αυξήσουμε τις συχνότητες κατά μ τότε ο νέος πληθυσμός (ν ') θα είναι:

κ κ κ

ν ' = Σ( ν ; + μ ) = Σ ν; + Σ μ = ν + κμ i = l i = l i = l

ΑΣ Κ Η � Η 7 11 Έστω χ;, ί= 1 ,2 . . . , 1 0 οι 1 0 τιμές μιας μετα­

βλητής Χ σ' ένα δείγμα μεγέθους ν = 50 με χ; = ί, ί =1 ,2 . . . , 1 0, ν;, ί=1 .2 . . . , 10 οι αντίστοιχες συ­χνότητές τους και χ = 1 1 , 8 η μέση τιμή τους.

Αν όλες οι συχνότητες αυξηθούν κατά 2 να υπολογίσετε τη νέα μέση τιμή των τιμών αυτών.

ΛΥΣ Η Ο πληθυσμός θα γίνει:

1 0 1 0 1 0 ν ' = Σ ( ν , + 2 ) = Σ ν ; + Σ 2 = ν + 1 0 · 2 = 70 · i = l i = l i = l

Η νέα μέση τιμή χ ' είναι:

χ ' = -Σχ ; (v ; + 2 ) = - Σ χί vί + 2Σ χί - 1 1 0 1 ( 1 0 1 0 )

70 ι = Ι 70 i = l i = l ( 1 )

Όμως: Αν χ η αρχική μέση τιμή, τότε: \ 0 1 0

Σ χ;ν ; Σχ;V; ι ο ;z = l.=.L__ άρα 1 1, 8 = l.=.L__ <:::::>Σ χ. ν. = 590 και ν 50 i= l ι ι

\ Ο

Σχ; = Χ ι + Χ2 + Χ3 + . . . + Χ10 = 1 + 2 + 3 + . . . + 1 0 = i = l

1 =2

10( 1 + 1 0) = 55

(Θυμηθείτε το άθροισμα των ν πρώτων όρων αριθ­

μητικής προόδου: Σ ν = k ν ( α1 + αν )J

'Ετσι έχουμε : ;z . = _!_(590 + 2 · 55 ) = 700 = 1 0 70 70 ΑΣΚΗΣΗ 8'1

(l:Υ :\ΤΕΛ Εl:ΊΉ Σ :\"I ΕΊΊ\ ΒΟ Α Η Σ - Ο νΙ Ο Ι ΟΓΕ'\ E I A )

Στα δυο τμήματα της Γ Τάξης ενός επαρχι­ακού σχολείου στο διαγώνισμα στο μάθημα της Στατιστικής η βαθμολογία έχει κατανομή περί­που κανονική.

Στο 1 ο τμήμα το 50% των μαθητών έχει βαθμολογία πάνω από 12 ενώ το 49,85% από αυτούς έχει βαθμολογία μέχρι 18 .

Στο 2° τμήμα το 16% των μαθητών έχει βαθμολογία μέχρι 10 ενώ από 1 0 μέχρι 19 έχει το 8 1 ,5% των μαθητών.

Μπορείτε να συγκρίνετε τα δυο τμήματα ως προς την ομοιογένειά τους.

Λ ΥΣ Η ( 1 " τμήμα) Έστω � , Sx η μέση τιμή και η τυπική απόκλιση της κατανομής της βαθμο­λογίας του τμήματος.

' ' - - - - - - 1- - - - - - - - - - - - - - - - - - - L - - - - -

' ' ' ' ' ' ' '

- - - - - - - - - - - - - j - - - - - - j - - - - - - - - - - - - - - - - - - 1 - - - - - - [ - - - - - - - - - - - -

χ - 35, χ - 25, χ - 5, χ χ + 5, χ + 25, χ + 35, 68% 95%

99,7% 49,85% {χ = 12 {�= 12 Από τα δεδομένα προκύπτουν: _ <:::::> x+3sx = 18 sx =2

Άρα: CV1 = _3_ = ο, 1 66 δηλ. 1 6 .6%. 1 2

(2" τμή μα) Έστω Ύ , Sy η μέση τιμή και η τυ-πική απόκλιση της κατανομής της βαθμολογίας του τμήματος.

' ' _ _ _ _ _ _ ,_ _ _ _ _ _ _ _ _ _ _ _ _ _ _ _ _ _ _ ,ι _ _ _ _ _

' ' ' ' ' ' ' ' - - - - - - - - - - - - - : - - - - - - [ - - - - - - - - - - - - - - - - - - 1 - - - - - - ; - - - - - - - - - - - -

Υ - 35, Υ - 25,. Υ - S,. Υ y + 5, y + 25, y + 35, 68% 95%

99,7% 1 6% 8 1 ,5%

Από τα δεδομένα προκύπτουν: {y - sY = 1 0 {Ύ = 1 3

(Ύ + 2sY ) - (Ύ - sY ) = 1 9 - 1 0 <:::::>

s Y = 3

Άρα: CV2 = 1._ :::: Ο, 2307 δηλ. 23 ,07%. 1 3

Άρα: Το 1 ο τμήμα παρουσιάζει μεγαλύτερη ομοιογένεια.

ΕΥΚΛΕΙΔΗΣ Β' 66 τ.2/61

Page 64: Ευκλειδης Β 66

Μαθη ματικά για την Γ Λυκείου

Κατεύθυνση ΘΕΩΡΗΜΑΤΑ ΣΥΝΕΧΩΝ

ΣΥΝΑΡΤΉΣΕΩΝ Ψύχας Βαγγέλης

Οι συνεχείς συναρτήσεις είναι μία σημαντική κλάση των πραγματικών συναρτήσεων μιάς πραγματικής μεταβλητής. Τα βασικά θεωρήματα των συνεχών συναρτήσεων σε συνδυασμό με τη μονοτονία, μας βοη­θούν να βγάλουμε σημαντικά συμπεράσματα γιά τη συμπεριφορά των συνεχών συναρτήσεων. Στόχος του άρθρου αυτού είναι κυρίως η ανάδειξη του σημαντικού ρόλου που παίζει το σύνολο τιμών μίας συνεχούς συνάρτησης.

1 . Θεώρη μα Bolzano Αν μία συνάρτηση f είναι συνεχής στο [α,β ]

και f(α) · f(β) < Ο , τότε υπάρχει ένα τουλάχιστον ξ Ε (α,β) τέτοιο ώστε : f(ξ) = Ο .

Δηλαδή η εξίσωση f(x) = Ο , έχει μία τουλάχι­στον λύση στο (α,β) . (Σχήμα 1)

f(β)

f(α)

Σχήμα 1

+ Ειδικά αν f(α) · f(β) � Ο τότε οι αριθμοί α και β , μπορεί να είναι λύσεις της εξίσωσης.

2 . Θεώρη μα Ενδιάμεσων Τιμών

Αν μία συνάρτηση f είναι συνεχής στο [α,β ] και f (α) -φ f (β) , τότε για κάθε Κ μεταξύ των f(α) και f(β) υπάρχει ένα τουλάχιστον ξ Ε (α, β) τέτοιο ώστε: f(ξ) = κ .

Δηλαδή η εξίσωση f(x) = κ , έχει μία τουλά­χιστον λύση στο (α,β) . (Σχήμα 2)

f(β)

κ

f(α)

α

Σχήμα 2

3 . Θεώρημα Μ έγιστης & Ελάχιστης Τιμής Αν μία συνάρτηση f είναι συνεχής στο [α,β]

τότε παρουσιάζει μέγιστη και ελάχιστη τιμή σ 'αυτό.

+ Δηλαδή υπάρχουν χε , χμ Ε [α, β ] τέτοια ώστε f(χε ) � f(x) � f(χμ ) για κάθε χ Ε [α, β ] .

Αν θέσουμε f(χε ) = m (την ελάχιστη τιμή της f ) και f(χμ ) = Μ (τη μέγιστη τιμή της f ) , τότε το σύνολο τιμών της συνάρτησης f με πεδίο ορι­σμού το [α, β] , είναι το [m,M] . (δηλαδή f([α, β]

)= [m,M] ) .

Άρα για κάθε κ Ε [m,M] υπάρχει ένα τουλά­χιστον ξ Ε [α, β] τέτοιο ώστε: f(ξ) = κ .

Δηλαδή η εξίσωση f(x) = κ , έχει μία τουλά­χιστον λύση στο [α, β] . (Σχήμα 3)

ΕΥΚΛΕΙΔΗΣ Β ' 66 τ.2/62

Page 65: Ευκλειδης Β 66

Μαθη ματικά για την Γ Λυκείου

f(β)

f(α)

m

Π ιφαηΊ ρηση 1 Με τη βοήθεια του Θεωρήματος του Bolzano,

μπορούμε να αποδείξουμε ότι μία εξίσωση έχει ρί­ζα σε ένα συγκεκριμένο διάστημα των πραγματι­κών αριθμών και στη συνέχεια με τη μέθοδο της διχοτόμησης να την προσεγγίσουμε, όσο κοντά ε­πιθυμούμε.

Π αρατή ρηση 2 Αν μία συνάρτηση f είναι συνεχής και γνησί­

ως μονότονη στο [α,β ] και f(α) · f(β) < Ο , τότε υπάρχει �:να ακριβώς ξ Ε (α, β) τέτοιο ώστε: f(ξ) = ο . Δηλαδή η εξίσωση f(x) = Ο , έχει μία ακρ ιβώς

λύση στο (α,β) .

Π αρατή ρηση 3 Αν μία συνάρτηση f είναι συνεχής [α, β ] και

f(x) '* Ο για κάθε χ Ε (α, β) , τότε η συνάρτηση f θα διατηρεί σταθερό πρόσημο στο (α, β ) .

Δηλαδή θα ισχύει: f(x) > Ο για κάθε χ Ε (α, β)

ή f(x) < Ο για κάθε χ Ε (α, β) .

Π αρατή ρηση 4 Αν η συνάρτηση f είναι ορισμένη και συνε­

χής στο ανοικτό διάστημα (α, β ) , lim f(x) = Α χ�α+

και lim f(x) = Β , τότε σύνολο τιμών της συνάρ-χ�β-

τησης f είναι το ανοικτό διάστημα (Α, Β) (όταν η f είναι γνησίως αύξουσα) ή το ανοικτό διάστη­μα (Β, Α) (όταν η f είναι γνησίως φθίνουσα) .

Παρακάτω λύνουμε μία κλασική άσκηση με τρείς διαφορετικούς τρόπους. Με ένα τουλάχιστον

από αυτούς τους τρόπους αντιμετωπίζεται η πλειο­νότητα των ασκήσεων που αναφέρονται στα Θε­ωρήματα των Συνεχών Συναρτήσεων.

Αl: Κ Η Σ Η l Αν η συνάρτηση f είναι ορισμένη και συνε­

χής στο [α, β ] με f(α) '* f(β) και κ, λ είναι θε-τικοί πραγματικοί αριθμοί, τότε υπάρχει ένα τουλάχιστον ξ Ε (α,β) τέτοιο ώστε:

Λύση

f(ξ) = κ f(α) + λ f(β)

. κ + λ

1 ο; τρόπος (Θέλουμε να αποδείξουμε ότι υπάρχει ένα

τουλάχιστον ξ Ε (α,β) τέτοιο ώστε

f(ξ) =κ f(α) + λ f(β)

κ + λ (κ + λ )f(ξ) = κ f(α) + λ f(β)

(κ + λ )f(ξ) - κ f(α) - λ f(β) = Ο . ) Η τελευταία ισότητα μας οδηγεί να θεωρή­

σουμε κατάληλη συνάρτηση g : [α, β ] � JR με g(χ) = (κ + λ)f(χ) - κf(α) - λ f(β) και να εφαρ­μόσουμε το Θεώρημα του Bolzano

Αρκεί να αποδείξουμε ότι υπάρχει ένα τουλά­χιστον ξ Ε (α,β ) τέτοιο ώστε g(ξ) = Ο .

Υπολογίζουμε τα g(α) και g(β) . g(α) = (κ + λ)f(α) - κ f(α) - λ f(β) =

= κf(α) + λf(α) - κf(α) - λ f(β) = = λ(f(α) - f(β)) .

g(β) = (κ + λ)f(β) - κ f(α) - λ f(β) = = κf(β) + λf(β) - κ f(α) - λ f(β ) =

= -κ(f(α) - f(β)) .

Η συνάρτηση g είναι συνεχής στο κλειστό διάστημα [α,β ] και

g(α) · g(β) = -κλ(f(α) - f(β)) 2 < Ο .

Άρα σύμφωνα με το Θεώρημα του Bolzano, θα υπάρχει ένα τουλάχιστον ξ Ε (α, β) τέτοιο ώστε g(ξ) = Ο .

2"' τρόπος Εφόσον f(α) '* f(β) , υποθέτουμε χωρίς βλάβη

της γενικότητας ότι f(α) < f(β) . Τότε πολλαπλα­σιάζοντας τη ανισότητα αυτή με τους θετικούς α­ριθμούς κ και λ έχουμε:

{κf(α) < κ f(β) λf(α) < λ f(β)

ΕΥΚΛΕΙΔΗΣ Β' 66 τ.2/63

Page 66: Ευκλειδης Β 66

Μαθη ματικά για την Γ Λυκείου

Προσθέτουμε το λf(β) στη πρώτη και το κf(α) στη δεύτερη .

{ κf(α) + λf(β) < κf(β) + λf(β) ==> λf(α) + κf(α) < λf(β) + κf(α)

{ κf(α) + λf(β) < (κ + λ)f(β) ==>

(κ + λ)f(α) < κf(α) + λ f(β)

=> (κ + λ)f(β) < κf(α) + λf(β) < (κ + λ)f(β) .

=> f(α) < κ f(α) + λf(β) < f(β) . κ + λ

Δηλαδή ο αριθμός ρ = κf(α) +�f(β) βρί-κ + σκεται μεταξύ των f(α) και f(β) , οπότε σύμφω-να με το Θεώρημα Ενδιάμεσων Τιμών, θα υπάρχει ξ Ε (α,β) τέτοιο ώστε :

f(ξ) = κ f(α) + λ f(β) = . κ + λ ρ

3ο; τρόπος Εφόσον η συνάρτηση f είναι συνεχής στο

κλειστό διάστημα [α,β ] , θα παρουσιάζει μέγιστη και ελάχιστη τιμή . Δηλαδή υπάρχουν m,M Ε � τέτοια ώστε :

m � f(x) � Μ για κάθε χ Ε [α, β ] . Η παραπάνω ανισότητα ισχύει για κάθε

χ Ε [α, β] , άρα και για χ = α και για χ = β . δηλαδή θα ισχύουν οι ανισότητες :

{m � f(α) � M m � f(β) � M

Πολλαπλασιάζουμε τις ανισότητες με τους θε­τικούς αριθμούς κ και λ αντίστοιχα.

{κm � f(α) � κ Μ λm � f(β) � λΜ

Προσθέτουμε τις ανισότητες κατά μέλη . (κ + λ)m � f(α) + f(β) � (κ + λ)Μ

Διαιρούμε με κ + λ > Ο και έχουμε :

m � κ f(α) +� f(β) � Μ . κ +

Δ λ δ , θ , κ f (α) + λ f (β) ,

η α η ο αρι μος ρ = λ , ανηκει κ + στο σύνολο τιμών της συνάρτησης f , οπότε θα υπάρχει ένα τουλάχιστον ξ Ε [α, β ] τέτοιο ώστε:

f(ξ) = = κf(α) + λ f(β) . ρ κ + λ Με την απαγωγή εις άτοπο θα αποδείξουμε ότι

ξ * α και ξ * β .

Έστω ότι ξ = α τότε:

f(α) = κf(α) + λ f(β) , κ + λ η

(κ + λ)f(α) = κf(α) + λ f(β) ή λf(α) = λ f(β) ή

f(α) = f(β) Άτοπο. Όμοια και για ξ = β .

Π αραη)ρηση Η παραπάνω άσκηση ισχύει και στη περίπτω­

ση που κ · λ > Ο . Επίσης μπορεί να γενικευτεί ή να εξειδικευθεί. Για παράδειγμα αναφέρουμε τις πα­ρακάτω ασκήσεις.

ΑΣ Κ Η l: Η 2 Α ν η συνάρτηση f είναι ορισμένη και συνε­

χής στο [α,β] με f(α) * f(β) τότε υπάρχει ένα τουλάχιστον ξ Ε (α,β) τέτοιο ώστε:

f(ξ) = 3f(α) + 4f(β) . 7

(Χρησιμοποιείστε οποιουσδήποτε θετικούς ακέ­ραιους στη θέση των κ , λ και φτιάξτε τη δική σας άσκηση)

Al: K H l: H 3 Έστω f συνάρτηση ορισμένη και συνεχής

στο [α, β] και γ Ε (α, β) . Αποδείξτε ότι υπάρχει ένα τουλάχιστον ξ Ε [α, β] τέτοιο ώστε:

f(ξ) = f(α) + 2f(β) + 3f(y). 6

Λύση 1 ος τρόπος Εφόσον η συνάρτηση f είναι συνεχής στο

κλειστό διάστημα [α, β] , θα παρουσιάζει μέγιστη και ελάχιστη τιμή . Δηλαδή υπάρχουν m,M Ε � τέτοια ώστε :

m � f(x) � Μ για κάθε χ ε [α, β] . Η παραπάνω ανισότητα ισχύει για κάθε

χ Ε [α, β] , άρα και για χ = α , για χ = β και για χ = γ δηλαδή θα ισχύουν οι ανισότητες : {m � f (α ) � M

2m � 2f (β ) � 2Μ

3m � 3f ( γ ) � 3Μ Προσθέτουμε τις ανισότητες κατά μέλη παίρ­

νουμε 6m � f(α) + 2f(β) + 3f(γ) � 6Μ , οπότε:

m � f(α) + 2f(β) + 3f(y) � Μ . 6

, , f(α) + 2f(β) + 3f(y) Δηλαδη ο αριθμος ρ =

6 ,

ανήκει στο σύνολο τιμών της συνάρτησης f , οπό-

ΕΥΚΛΕΙΔΗΣ Β ' 66 τ.2/64

Page 67: Ευκλειδης Β 66

Μαθη ματικά για την Γ Λυκείου

τε θα υπάρχει ένα τουλάχιστον ξ Ε [α, β] τέτοιο

ώστε: f(ξ) = ρ = f(α) + 2f(β) + 3f(γ) . 6

2"' τρόπος Θεωρούμε τη συνάρτηση : g(x) = 6f(x) - f(α) - 2f(β) - 3f(y) (ι)

Αρκεί να αποδείξουμε ότι υπάρχει ένα του­λάχιστον ξ Ε [α, β] τέτοιο ώστε g(ξ) = Ο .

Αντικαθιστούμε στη σχέση (ι) διαδοχικά ό­που χ το α , το β και το y , οπότε προκύπτουν οι ισότητες : {g(α) = 6f(α) - f(α) - 2f(β) - 3f(y)

g(β) = 6f(β) - f(α) - 2f(β) - 3f(y) g(y) = 6f(y) - f(α) - 2f(β) - 3f(y)

Πολλαπλασιάζουμε τη δεύτερη ισότητα με 2 και τη τρίτη με 3 . { g(α) = 6f(α) - f(α) - 2f(β) - 3f(y )

2g(β) = ι2f(β) - 2f(α) - 4f(β) - 6f(y) 3g(y) = ι8f(y) - 3f(α) - 6f(β) - 9f(y)

Προσθέτουμε τις ισότητες κατά μέλη και έ­χουμε :

g(α) + 2g(β) + 3g(y) = Ο (2) Αν τώρα g(α) = Ο ή g(β) = Ο ή g(y) = O , τό­

τε υπάρχει ξ Ε [α, β Ι ( ξ = α ή ξ = β ή ξ = y ) , τέ­τοιο ώστε g(ξ) = Ο .

Αν όμως g(α) · g(β) · g(y) :;t O , τότε από τη σχέση (2) συμπεραίνουμε ότι οι αριθμοί g(α) , g(β) και g(y) , αποκλείεται να είναι ομόσημοι (διότι είναι μη μηδενικοί και έχουν άθροισμα μη­δέν) . Άρα δύο τουλάχιστον από αυτούς θα είναι ετερόσημοι.

Ας υποθέσουμε, χωρίς να βλάπτεται η γενικό­τητα, ότι οι αριθμοί g(β) και g(y) είναι ετερόση-μοι οπότε g(β) · g(y) < O . Τότε θα ισχύει για την συνάρτηση g το Θεώρημα του Bolzano στο [β, y ] , οπότε θα υπάρχει ένα τουλάχιστον ξ Ε (β, y) τέτοιο ώστε g(ξ) = Ο .

Α Σ Κ Η l: Η 4 Έστω f συνάρτηση ορισμένη και συνεχής

στο [α, β] και γ Ε (α, β) . Αν κ:, λ, μ είναι ομόση-μοι πραγματικοί αριθμοί, αποδείξτε ότι υπάρχει ένα τουλάχιστον ξ Ε [α, β] τέτοιο ώστε:

f(ξ) = κf(α) + λf(β) + μf(γ) . κ + λ + μ

Al: K H l: H 5 Εστω f : [α, β ] � � μία συνεχής συνάρτηση,

χ ι , χυ . . · , χ 0 Ε [α, β] και k ι , k 2 , . . · , k 0 θετικοί ακέραιοι. Αποδείξτε ότι υπάρχει ένα τουλάχι­στον ξ Ε [α, β] , τέτοιο ώστε:

f(ξ) = k ι f(χ ι ) + k 2f(x 2 ) + . . . + k 0f(x 0 ) . k 1 + k 2 + . . · + k n

Ση μείωση Οι ασκήσεις 4 και 5 αποτε}.ούν γενίκευση της 3

και αντιμετωπίζονται ομοίως.

ΑΣ Κ Η l: Η 6 Εστω f : [0,2] � � μία συνεχής συνάρτηση

ώστε f(O) = f(2) . Αποδείξτε ότι υπάρχουν

X0 , y 0 E [0,2 ] με lχ0 - Υ0 Ι = ι ώστε

f(x0 ) = f(y 0 ) .

Λ1Jση Έχουμε: lxo - Yo l = ι �

� ( Χο - Υ ο = ι ή Χ ο - Υ ο = -ι )� � ( Υ ο = Χο - ι ή Υ ο = Χ ο + ι ) .

Αρκεί λοιπόν να αποδείξουμε ότι υπάρχει ένα του­λάχιστον Χ0 Ε [0,2] τέτοιο ώ-στε : f(x0 ) = f(x0 - ι) ή f(x0 ) = f(x0 + ι) . Θεωρούμε τις συναρτήσεις : g : [ι,2] � � με g(x) = f(x) - f(x - ι) και h : [Ο,ι] � � με h(x) = f(x) - f(x + ι) (*) . Αρκεί τώρα να αποδείξουμε ότι υπάρχει Χ0 Ε [0,2] τέτοιο ώστε: g(x0 ) = Ο ή h(x0 ) = Ο . g(l) = f(l) - f(O) g(2) = f(2) - f(ι) = f(O) - f(ι) Άρα g(ι) · g(2) = -(f(ι) - f(0))2 5; Ο �

� g(ι) . g(2) 5; ο . Η g είναι συνεχής στο [ ι ,2] . Αν g(ι) · g(2) < Ο , τότε σύμφωνα με το Θεώρημα του Bolzano, θα υπάρχει ένα τουλάχιστον ξ 1 Ε (ι,2) τέτοιο ώστε: g( ξ ι ) = Ο Αν g(l ) · g(2) = Ο τότε g(ι) = Ο ή g(2) = Ο , οπό­τε ξ 1 = ι ή ξ ι = 2 Τελικά θα υπά ει ένα τουλά ιστον ξι Ε [ι,2] τέτοιο ώστε : g ξ ι = Ο h(O) = f(O) - f(ι) h(t ) = f(ι) - f(2) = f(t) - f(O) Άρα h(O) · h(ι) = -(f(ι) - f(0))2 =:;; Ο � � h(O) · h(t) 5; Ο

ΕΥΚΛΕΙΔΗΣ Β ' 66 τ.2/65

Page 68: Ευκλειδης Β 66

Μαθη ματικά για την Γ Λυκείου

Η h είναι συνεχής στο [0,1 ] . Α ν h(O) · h(1) < Ο , τότε σύμφωνα με το Θεώρημα του Bolzaηo, θα υπάρχει ένα τουλάχιστον ξ 2 Ε (0,1) τέτοιο ώστε : h(ξ 2 ) = 0 Αν h(O) · h(1) = Ο τότε h(O) = Ο ή h(1) = Ο , οπό­τε ξ 2 = Ο ή ξ 2 = 1 .

Από τις σχέσεις ( 1 ) και (2) (θέτοντας Χ0 = ξ 1 ή χ0 = ξ2 ) συμπεραίνουμε ότι υπάρχει Χ0 Ε [0,2] τέτοιο ώστε : g(x0 ) = Ο ή h(x0 ) = Ο . (*) Εφόσον πεδίο ορισμού της συνάρτησης f , είναι το κλειστό διάστημα Δ = [0,2] και

g(x) = f(x) - f(x - 1) , πρέπει να ισχύουν οι ανισώσεις: { ο ::;; χ ::;; 2

δηλαδή {ο ::;; χ ::;; 2. ο :::; χ - 1 :::; 2 1 ::;; χ ::;; 3

Άρα το χ θα ανήκει στο κλειστό διάστημα [1,2 ] , που θα αποτελεί το πεδίο ορισμού της συνάρ-

τησης g . Όμοια και για τη συνάρτηση h , βρίσκουμε ότι

το πεδίο ορισμού της είναι το κλειστό διάστημα [0,1 ] .

Λ� Κ Η Σ Η 7 Εστω f : [α, β] � JR μία συνεχής συνάρτηση

ώστε f (α) = f (β) . Αποδείξτε ότι υπάρχουν

Χ0 , Υ 0 Ε [α, β ] με l xo - y0 1 =

β ; α ώστε

f(x0 ) = f(y0 ) .

Υπόδr.ιξη (Εργαζόμαστε όπως στην προηγούμενη άσκηση . )

I I β - α Χο - yo =

-2- ...

... (Υ ο = Χο - β � α ή Υ ο = Χ0 +

β � α) .

Θεωρούμε τις συναρτήσεις :

g : [α ; β , β]� JR με g(x) = f(x) - r( χ - β;α) και

h : [α, α ; β ] � JR με

h(x) = f(x) - r( χ + β ; α ) .

Εφαρμόζουμε το Θεώρημα του Bolzaηo, για τη

συνάρτηση g στο κλειστό διάστημα [α ; β , β]

και για τη συνάρτηση h στο κλειστό διάστημα

[α, α ; β ] .

ΑΣ Κ Η Σ Η 8 Εστω f : [0,1 ] � JR μία συνεχής συνάρτηση

ώστε f(O) = f(l) και η θετικός φυσικός αριθ­μός. Αποδείξτε ότι υπάρχει ένα τουλάχιστον

ξ Ε [0,1 ] ώστε f(ξ) = f(ξ + _!_) . η

. \ iJση Θεωρούμε τη συνάρτηση

g : [o, η :1]� JR με g(x) = f(x) - f(x +

�) .

Αρκεί να αποδείξουμε ότι υπάρχει ένα τουλάχι­στον ξ Ε [0,1 ] ώστε g(ξ) = Ο . Η συνάρτηση g είναι συνεχής στο [0,1 ] , οπότε θα παρουσιάζει μέγιστη και ελάχιστη τιμή . Δηλαδή υπάρχουν m,M Ε JR τέτοια ώστε :

m ::;; g(x) ::;; Μ για κάθε χ Ε [0,1 ] . Η παραπάνω ανισότητα ισχύει για κάθε χ Ε [0,

1] ,

' ο 1 2 αρα και για χ = , για χ = - , για χ = - , . . . . . . . για η η

η - 2 η - 1 χ = -- και για χ = -- . Δηλαδή θα ισχύουν η η

οι ανισότητες : m ::;; g(O) ::;; M

m ::;; g(� ) ::;; M

m ::;; g( � ) ::;; M

( η - 2 ) m ::;; g -η

- ::; Μ

( η - 1 ) m ::;; g -η- ::; Μ

( η - 1 ) m ::;; f -η- - f(1 ) ::;; M

ΕΥΚΛΕΙΔΗΣ Β ' 66 τ.2/66

Page 69: Ευκλειδης Β 66

Μαθη ματικά για την Γ Λυκείου

Προσθέτοντας τις παραπάνω ανισότητες κατά μέ­λη έχουμε : m ::;; Ο ::;; Μ . Δηλαδή το Ο ανήκει στο σύνολο τιμών της συνάρτησης g , οπότε σύμφωνα θα υπάρχει ένα τουλάχιστον ξ Ε (0,1 ] ώστε g(ξ) = Ο .

Στη συνέχεια παραθέτουμε μία σειρά από διά­φορες ασκήσεις που αναφέρονται στις συνεχείς συ­ναρτήσεις.

ΑΣ Κ Η Σ Η 9 Ορειβάτης ξεκινά απο τη βάση Β ενός βου­

νού στις 6 πμ και φτάνει στη κορυφή Κ στις 4 μμ (της ίδιας η μέρας). Την επόμενη ημέρα ξε­κινά από τη κορυφή Κ στις 6 πμ και ακολου­θόντας την ίδια διαδρομή κατεβαίνει στη βάση Β στις 4 μμ (της ίδιας η μέρας). Αποδείξτε ότι

υπάρχει ένα τουλάχιστον σημείο της διαδρομής στο οποίο βρ ίσκεται ο ορειβάτης την ίδια χρο­νική στιγμή και όταν ανεβαίνει και όταν κατε­βαίνει.

\ (,ση ( 1 ' τ μ ι) :τ ο � ) Έστω f η συνάρτηση που περιγράφει τη κί­

νηση του ορειβάτη κατά την άνοδο ( f(t) είναι το διάστημα που διανύει συναρτήσει του χρόνου t ) και g η συνάρτηση που περιγράφει τη κίνηση του ορειβάτη κατά την κάθοδο. Οι συναρτήσεις f, g είναι συνεχείς και επίπλέον ισχύουν οι ισότητες :

f(6) = g(16) (διότι τη χρονική στιγμή t = 6 τη πρώτη ημέρα και τη χρονική στιγμή t = 16 τη δεύ­τερη ημέρα ο ορειβάτης βρίσκεται στη βάση του βουνού) f(1 6) = g(6) (διότι τη χρονική στιγμή t = 1 6 τη πρώτη ημέρα και τη χρονική στιγμή t = 6 τη δεύτερη ημέρα ο ορειβάτης βρίσκεται στη κορυφή του βουνού)

Θεωρούμε τώρα τη συνάρτηση h(t) = f(t) - g(t) .

Η h είναι συνεχής στο [6,16] και h(6) . h(1 6) = (f(6) - g(6)) · (f(16) - g(1 6)) =

= (f(6) - g(6)) · (g(6) - f(6)) = -(f(6) - g(6)γ < ο Άρα σύμφωνα με το Θεώρημα του Bolzano,

θα υπάρχει ένα τουλάχιστον t 0 Ε (6,16) τέτοιο ώστε h(t 0 ) = 0 .

Δηλαδή τη χρονική στιγμή t 0 , ο ορειβάτης θα βρίσκεται στη θέση f(t 0 ) = g(t 0 ) και όταν ανε­βαίνει και όταν κατεβαίνει.

2"� τp6πος Ας υποθέσουμε ότι τη στιγμή που ο ορειβάτης ξεκινάει την κάθοδο από τη κορυφή (δεύ­τερη ημέρα), η "σκιά" του (ή ο "κλώνος" του αν θέ­λετε) ξεκινάει ταυτόχρονα από τη βάση του βουνού

και ανεβαίνει (ακριβώς όπως ο πραγματικός ορειβά­της τη προηγούμενη ημέρα) προς τη κορυφή.

Η ζητούμενη χρονική στιγμή είναι η στιγμή που θα "συναντηθεί" ο ορειβάτης με τη "σκιά" του .

ΑΣ Κ Η Σ Η 1 0 Αν μία συνάρτηση f : � � � είναι "1 - 1"

και συνεχής, aποδείξτε ότι είναι γνησίως μονό­τονη.

Λύση Έστω χ 1 , χ 2 , χ 3 Ε � με χ 1 < χ 2 < χ 3 , οπότε

αφού η f είναι 1 - 1 οι τιμές f (x 1 ) , f (x 2 ) , f (x3 ) θα είναι άνισες ανά δύο.

Έστω επίσης ότι η συνάρτηση f δεν είναι ού­τε γνησίως αύξουσα ούτε γνησίως φθίνουσα τότε δεν θα ισχύει καμία από τις σχέσεις f(x 1 ) < f(x 2 ) < f(x3 ) και f(x 1 ) > f(x 2 ) > f(x 3 ) . Δηλαδή το f(x 2 ) δεν θα βρίσκεται ανάμεσα στο f(x 1 ) και στο f(x3 ) .

Οπότε θα ισχύει μία από τις ανισότητες: f(x 1 ) < f(x3 ) < f(x 2 ) (1 ) f(x 2 ) < f(x 1 ) < f(x3 ) (2) f(x 1 ) > f(x 3 ) > f(x 2 ) (3) f(x 2 ) > f(x 1 ) > f(x 3 ) (4)

Ας υποθέσουμε ότι ισχύει η ( 1 ) .

εφόσον το f(x3 ) βρίσκεται μεταξύ του f(x 1 ) και στο f(x 2 ) , θα υπάρχει σύμφωνα με το θεώρη­μα ενδιάμεσων τιμών ξ Ε (χ 1 , χ 2 ) τέτοιο ώστε f(x 3 ) = f(ξ) . Δηλαδή για ξ < χ 3 έχουμε f(x3 ) = f(ξ) . Άτοπο διότι η f είναι "1 - 1" .

Όμοια καταλήγουμε σε άτοπο αν υποθέσουμε ότι ισχύουν οι ανισότητες ( 2) , ( 3) ή ( 4) .

ΑΣ Κ Η Σ Η 1 1 π

Α ν Ο < α < β < - και η συνάρτηση 2

f : (α, β ] � � είναι συνεχής στο [α, β ] με f (α) 'Φ f (β) , aποδείξτε ότι υπάρχει ένα τουλάχι­στον ξ Ε (α, β) τέτοιο ώστε:

f(ξ) = ημ 2ξ · f(α) + συν 2ξ · f(β) . ΛίJση Θεωρούμε τη συνάρτηση g(x) = f(x) - ημ 2 χ • f(α) - συν 2 χ • f(β)

Αρκεί να αποδείξουμε ότι υπάρχει ένα τουλάχι­στον ξ Ε (α, β) τέτοιο ώστε g(ξ) = Ο .

Η συνάρτηση g είναι συνεχής στο [α, β ] .

g(α) = f(α) - ημ 2α · f(α) - συν 2α · f(β) =

= f(α) · (ι - ημ 2α)- συν 2α · f(β) =

ΕΥΚΛΕΙΔΗΣ Β ' 66 τ.2/67

Page 70: Ευκλειδης Β 66

Μαθηματικά για την Γ Λυκείου

= f(α) · συν2α - συν

2α · f(β) =

= συν2α ·

(f(α) - f(β)

) (1)

g(β) = f(β) - ημ 2β · f(α) - συν

2β · f(β) =

= f(β) · (ι - συν2β )- η μ

2β · f(α) =

= f(β) · ημ 2β - ημ

2β · f(α) =

= ημ 2β ·

(f(β) - f(α)

) (2) .

Από τις σχέσεις (1) και (2) έχουμε :

g(α) · g(β) = -συν2α · ημ

2β ·

(f(β) - f(α)

γ < Ο .

Οπότε σύμφωνα με το Θεώρημα του Bolzaηo, θα υπάρχει ένα τουλάχιστον ξ Ε (α,β) τέτοιο ώστε g(ξ) = Ο .

λ Σ Κ Η Σ Η Ι 2 Α ν μία συνάρτηση f : [α, β ] � JR είναι συνε-

χής στο [α, β] και f(α) -=ι:. Ο , aποδείξτε ότι υπάρ­χει ένα τουλάχιστον ξ Ε (α, β) τέτοιο ώστε:

f(ξ) f(α) + f(β) -- = ------"-...0... ξ - α β - α

ΛίJση Αρκεί να δείξουμε ότι η εξίσωση :

f(x) = f(α) + f(β) χ - α β - α ( Ι ) έχει ρίζα στο (α, β)

Η εξίσωση ( 1 ) στο JR - {α} είναι ισοδύναμη με την εξίσωση

f(χ)(β - α) - (χ - α) ( f(α) + f(β) ) = Ο Θεωρούμε τη συνάρτηση g ( χ ) = f(χ)(β - α) - (χ - α) ( f(α) + f(β) )

Αρκεί λοιπόν να αποδείξουμε ότι υπάρχει ένα τουλάχιστον ξ Ε (α, β) τέτοιο ώστε g(ξ) = Ο . Η συνάρτηση g είναι συνεχής στο [α, β ] . g(α) = f(α)(β - α) - (α - α)

(f(α) + f(β)

)=

= f(α)(β - α) (1) g(β) = f(β)(β - α) - (β - α)

(f(α) + f(β)

)=

= (β - α)(f(β) - f(α) - f(β)

)=

= -f(α)(β - α) (2) Από τις σχέσεις (1) και (2) έχουμε:

g(α).g(β) = -f2 (α)(β - α)

2 < Ο

Οπότε σύμφωνα με το Θεώρημα του Bolzaηo, θα υπάρχει ένα τουλάχιστον ξ Ε (α,β) τέτοιο ώ-στε g(ξ) = Ο . Επειδή όμως ξ =t:- α , το συγκεκριμέ­νο ξ θα είναι ρίζα και της ( Ι ) .

/\Σ Κ Η Σ Η 1 3 Αν μία συνάρτηση f : [α, β] � JR είναι συνε-

χής στο [α, β] και f(β) -=ι:. Ο , aποδείξτε ότι υπάρ-

χει ένα τουλάχιστον ξ Ε (α, β) τέτοιο ώστε:

f(ξ) =

f(α) + f(β) ξ - β α - β

Υπόδει ξη : Θεωρούμε τη συνάρτηση g(x) = f(χ)(α - β) - (χ - β)

(f(α) + f(β)

)

Αρκεί να αποδείξουμε ότι υπάρχει ένα τουλάχι­στον ξ Ε (α, β) τέτοιο ώστε g(ξ) = Ο .

Al: K H l: H I 4 Α ν μία συνάρτηση f : [α, β ] � JR είναι συνε-

χής στο [α, β] και f(α) -=ι:. f(β) , και m, η θετικοί πραγματικοί αριθμοί, τότε να αποδείξετε ότι υπάρχουν δύο τουλάχιστον (διαφορετικά μετα­ξύ τους) ξ0 ξ2 Ε (α, β) τέτοια ώστε:

Ιr(ξ ι > - r(ξ 2 >

I =

Im - η Ι Ιr(β) - r(α>

Ι· m + η

ΛίJση Εφαρμόζοντας τώρα κάποιον από τους τρό­

πους της Ά σιίlι σης ! . αποδεικνύουμε ότι υπάρχει ένα τουλάχιστον ξ ι Ε (α, β) τέτοιο ώστε :

f(ξ ) - m f(α) + η f(β) ι - (1) . m + η Όμοια αποδεικνύουμε ότι υπάρχει ένα τουλάχι­στον ξ2 Ε (α, β) τέτοιο ώστε :

f(ξ ) - η f(α) + m f(β) 2 - (2) . m + η

Θα μπορούσαμε βέβαια το ξ ι της σχέσης (1 ) να το είχαμε ονομάσει ξ 2 και το ξ 2 της σχέσης (2) να το είχαμε ονομάσει ξ ι .

Σε οποιαδήποτε περίπτωση όμως, αφαιρώντας τις σχέσεις (1) και (2) παίρνομαι :

Ιr(ξ ι > - r(ξ 2 )

I =

Im - ηΙ Ιr(β) - r(α>

Ι· m + η

Τα ξ ι και ξ2 είναι διαφορετικά μεταξύ τους, διότι αν υποθέσουμε ξ 1 = ξ 2 , τότε f(ξ ι ) = f(ξ 2 ) , οπότε f(α) = f(β) άτοπο.

Α Σ Κ Η Σ Η Ι 5 Α ν μία συνάρτηση f : [α, β] � JR είναι συνε-

χής στο [α, β] με f(α) -=ι:. f(β) , και η Ε Ν * , τότε να αποδείξετε ότι υπάρχουν δύο τουλάχιστον (διαφορετικά μεταξύ τους) ξ ι , ξ 2 στο (α, β) τέτοια ώστε:

lf(ξ ι ) - f(ξ2 ) 1 =

lf(�

η-:�α>

Ι.

Υπόδειξη : Αρκεί να πάρουμε m = η + 1 στην προηγούμενη άσκηση .

ΕΥΚΛΕΙΔΗΣ Β ' 66 τ.2/68

Page 71: Ευκλειδης Β 66

Τα Μαθηματικά μας διασκεδάζουν Τα μαθηματικά αν και είναι επιστήμη που απαιτεί αυστηρή διατύπωση, έχουν τη μαγεία

να αποσπούν το ενδιαφέρον όλων των ανθρώπων. Επινοήσεις σε προβλήματα ή ασκήσεις με κατάλληλο τρόπο διατυπωμένα εξάπτουν το πνεύμα, διεγείρουν τη φαντασία και κεντρίζουν την περιέργεια. Πρώτοι οι Αρχαίοι Έλληνες όπως ο Διόφαντος, ο Ζήνωνας κ.ά. μας δίδαξαν αυτά τα μαθηματικά.

Στη στήλη αυτή θα παρουσιάζουμε θέματα τα οποία δεν απαιτούν ιδιαίτερες μαθηματικές γνώσεις αλλά μας διασκεδάζουν με την εκφώνησή τους ή τη λύση τους και είναι μια ευχάρι­στη και συναρπαστική ασχολία .

Επιμέλεια: Παναγιώτης Χριστόπουλος Λίγα λόγι α από τ η ν Ι στ ο ρ ί α Οι Αρχαίοι Έλληνες είναι εκείνοι που μας κληροδότησαν τα πρώτα μα­

θηματικά παιχνίδια, με πρώτο το Διόφαντο. Αργότερα κυκλοφόρησε η « Ελληνική Ανθολογία» συλλογή από επιγράμματα διαφόρων εποχών καθώς και του Chuquet «Ανακαλύψεις αριθμών» ( 1 484) στη Γαλλία. Το 1 6° αιώνα ο αριθμός αυτcον των προβλημάτων είναι σημαντικός. Έπειτα εγκαταλείφθηκε η μελέτη τέτοιων προ­βλη μάτων. Όμως σιγά- σιγά άρχισαν να κυκλοφορούν διάφορες συλλογές όπως του Claνius ( 1 608), του Claude-Gaspard Bachet ( 1 626), του Ozanam "Διασκεδαστικά των Μαθματικών και της Φυσικής» ( 1 692) κ.ά. Ακόμα μεγάλοι μαθη ματικοί όπως ο Λεονάρδο της Πίζας ή Fibonacci, ο Eu1er, ο Newton, έχουν προσθέσει την συμβολή τους σ· αυτό το σύνολο των μαθηματικών που μεταδίδεται από αιώνα σε αιώνα.

Τ υ σ ο r; ίσ μ υ τ υ είναι λανθασμένοι συλλογισμοί, που δίνουν την εντύπωση σωστών συλλογισμών. Ο Ζήνων ο Ελεάτης, 485 π.Χ. είναι αυτός που παρουσίασε τους συλλογισμούς του «Αχιλλέα με τη Χελώ­να» και «το Βέλος που πετά» οι οποίοι έδωσαν αφορμή για πολλές συζητήσεις. Ο Αχιλλέας αν και βαδί­ζει 1 Ο φορές πιο γρήγορα από την Χελώνα έλεγε ο Ζήνων δεν θα την φθάσει ποτέ αφού αυτή προηγείται κατά ένα στάδιο και όταν διανύσει την απόσταση αυτή ο Αχιλλέας, η Χελώνα θα έχει προχωρήσει κ.ο .κ. Ακό μα έλεγε ότι το Βέλος δεν μπορεί να φθάσει το στόχο του αφού κάθε φορά που διανύει το μισό της απόστασης υπάρχει η άλλη μισή κ.ο.κ. μια απειρία στιγμών.

Ο Αριστοτέλης αναφέρει το συλλογισμό ως εξής: Ουδέποτε ένας ταχυκίνητος θα προφτάνει ένα βρα­δυπορούντα, που ξεκίνησε μπροστά από αυτόν. Γιατί κάθε φορά, που ο ταχυκίνητος θα διανύει την μετα­ξύ τους απόσταση , ο βραδυκίνητος θα προχωρεί ένα νέο διάστημα και συνεπώς θα βρίσκεται πάντα πιο μπρος από τον ταχυκίνητο.

Ο Δη μόκριτος , που ασχολήθηκε με τα απείρως μικρά, δεν δίσταζε να λέει: «Αν παραδεχθούμε ότι δυο επίπεδες τομές ενός κώνου που προκύπτουν από δύο παράλληλα επίπεδα απείρως γειτονικά, είναι ί­σες, τότε όλοι οι κώνοι είναι κύλινδροι» . Ο Αριστοτέλης επίσης μελέτησε τα θέματα των απείρως μικρών και απέδειξε ότι η συνέχεια των μεταβολών είναι φαινόμενο στενά συνδεδεμένο με τη φύση του χρόνου, της κίνησης και του χώρου .

Τα παράδοξα είναι αποτελέσματα λανθασμένα που φαίνονται όμως να προκύπτουν από ισχυρές α­ποδείξεις κατά την διεξαγωγή των οποίων παραλείψαμε να εφαρμόσουμε ορισμένες αρχές ή εκτελέσαμε πράξεις που δεν έχουν έννοια π. χ. αν διαιρέσουμε με α-β όταν το α = β .

Τα μαγικά τετράγωνα είναι τετράγωνα με ν γραμμές και ν στήλες, στις θέσεις αυτές τοποθετούμε αριθμούς έτσι ώστε σε κάθε γραμμή , σε κάθε στήλη και σε κάθε διαγώνιο να έχουν το ίδιο άθροισμα. Το 1 500 ο Αλβέρτος Durer, χάραξε σε ξύλινη πινακίδα αυτό τον πίνακα που έχει τον τίτλο «Μελαγχολί­

ΕΥΚΛΕΙΔΗΣ Β ' 66 τ.2/69

α». Στο διπλανό σχήμα διακρίνου­με μαγικό τετράγωνο.

Τα i':ξυπνα παιχνίδ ια με σπίρ­τα, με κέρματα, με νομίσματα, με σχέδια, με κομμάτια χαρτιού, με διάφορες κατασκευές, με αριθ­μούς, με γεωμετρικά σχήματα, με σταυρόλεξα κ. ά.

Στα έξυπνα παιχνίδια είναι και αυτά με τα τραπουλόχαρτα. Τα τρα­πουλόχαρτα είναι ανατολικής προέ-

Page 72: Ευκλειδης Β 66

-------------- Τα Μαθη ματικά μας διασκεδάζουν -------------

λευσης τα οποία περιέχουν 4 χρώματα ( 4 εποχές), 1 3 είδη ( 1 3 κύκλοι σελήνης στο έτος), 52 σύνολο φύλλων (52 εβδο­μάδες του έτους). Τα τραπουλόχαρτα εμφανίστηκαν το 14° αιώνα στη Γαλλία και με την ανακάλυψη της ξυλογραφίας διαδόθηκαν ταχύτατα. Με τα τραπουλόχαρτα έ:γιναν έξυπνα παιχνίδια από ταχυδακτυλουργούς σJJ...iJ. και μαθηματικά παιχνίδια.

Πολλά βιβλία κυκλοφορούν σήμερα σχετικά με τέ­τοια προβλήματα όπως: «Μαθηματικές σπαζοκεφαλιές» του Brian Bolt, «ο Σατανάς ο Cantor και το άπειρο» του Raymond Smullyan,όπως και «την κυρία ή την τίγρη», «το τσίρκο των Μαθηματικών» του Martin Gardner, <Πο πανηγύρι των Μαθηματικών», «Διασκεδαστικά Μαθημα­τικά» του Υ akoν Perelman, «ο άνθρωπος που μετρούσε» του Malba Tahan, «Το σαλιγκαράκι» του Μ. Δαυιδόπου­λου - Κ.Τολιόπουλου, «Σκέψου έναν αριθμό» του Johnny Ball και πολλά άλλα. Επίσης πολλά περιοδικά και εφημε­ρίδες καθημερινά προβάλουν τέτοια θέματα.

Μ αντέψτε το χαρτί Πάρετε από μια τράπουλα 2 1 χαρτιά και τοποθετή­

στε τα σε 3 δέσμες των 7 χαρτιών. Ζητήστε από κάποιον φίλο σας να βάλει ένα από αυτά στο μυαλό του. Μπορεί­τε να το μαντέψετε;

τ ο 2 � · ' ' 2 + 2 1 ο = .::.ερουμε οτι ισχυει ημ χ συν χ =

'Ε χουμε συνχ = �1 - (ημχ)2 2 . Υψώνουμε την ισότη­

τα στον κύβο συν 3 χ = ( �1 - (ημχ)2 2 ) 3

Θέτουμε στην ισότητα χ = π βρίσκουμε - 1 = 1 ή 0=2 . Τ α ξύλα Κάποιος αγόρασε ένα δεμάτι ξύλα τα οποία

ήταν δεμένα με ένα σχοινί 2 μέτρα και έδωσε 8 ευρώ. Α ν αγόραζε το άλλο μεγάλο δεμάτι ξύλα που είχε δεθεί με διπλάσιο σε μήκος σχοινί και κόστιζε 20 ευρώ θα κέρδι­ζε ή θα έχανε;

Γ

χ

Α Ε C> V)

Β

Υ

600m

Ο Πέτρος και η Άννα Ο Πέτρος και η Άννα έφυ-

γαν από την αγορά Α για το σπί­τι τους στο Σ. Ο Πέτρος πήρε τη διαδρομή ΑΒ = 1 50 μέτρα και ΒΣ = 600 μέτρα. Η Άννα πήρε τη διαδρομή ΑΓ = χ μέτρα και ΓΣ = y μέτρα. Αν οι διαδρομές είναι ίσες πόσο είναι το χ και

Σ πόσο το y; Το δένδρο Μέσα σε ένα τετράγωνο οικόπεδο υπάρ­

χει ένα δένδρο που από την κορυφή Α του τετραγώνου απέχει 30 μέτρα, από τη Β απέχει 40 μέτρα και από τη Γ απέχει 50 μέτρα. Τι εμβαδά έχει το οικόπεδο ;

Β Γ

Το 64 = 65 Χωρίζουμε το τετράγωνο ABCD · που έχει 8χ8=64 τετραγωνάκια σε δυο ίσα ορθογώνια τρίγωνα και σε δύο ίσα τραπέζια. Τοποθετούμε τα τμήματα όπως δείχνει το σχήμα IKLM που είναι ορθογώνιο 1 3χ5=65 τετραγωνάκια. Άρα 64=65 ! ! ! Arτ-·-1-r-f-·�B

ι - ;- ι : + r -ι - 1 -f- :·-ι . , ·t1 __ , _) ' G ;"Ή.:·: '--,- -- i f i ι · : >·,;:ι,:.-1 1 ι-J1 t ϊ-, --,--,--1rι V__:._ ι - ! - 1 1 i 'ϊ: , ' ι Τ �ΤΓVΗj Γ-- - ε:__;_] -"

Απανη)σεις : Μ αντέψτε το χαρτί Ζητήστε να σας πει ο φίλος σας σε ποια

δέσμη είναι το χαρτί που διάλεξε. Ενώστε τις δέσμες βάζοντας τη δέσμη που έχει το χαρτί στη μέση . Ξαναβάλτε τα χαρτιά σε 3 δέσμες ρίχνοντάς τα με τη σειρά ένα σε κάθε δέσμη και επαναλαμβάνετε την ίδια διαδικασία. Συνεχίζουμε το ίδιο για 3η φορά. Τώρα το φύλλο του φίλου σας είναι ακριβός στη μέ­ση, 1 1 η θέση .

Το Ο = 2 : _Το σωστό είναι συν 3 χ = ( ± �1 - (ημχ):;, 2 ) 3 δηλαδή - 1 =- 1 .

Τα ΞίJλα : Το εμβαδά της τομής των ξύ­λων στην πρώτη περίπτωση είναι πρ 2 στη δεύτερη περίπτωση είναι 4πλάσιο, 4πρ 2 άρα κέρδισε 1 2 ευρώ.

Ο Πέτρος κ αι η λννα : x+y = 750 (χ+ 1 50) 2 + 600 2 = y 2

Αρα χ = 1 8 1 ,25 μέτρα και y = 568,75 μέτρα. Το δένδρο

Β α Γ α-χ

50

α α

α-y 30

Α Από το σχήμα έχουμε : χ 2 +(α-y) 2 = 30 2 ( 1 ) (α-χ) 2 +y 2 = 50 2 (2), x 2 +y 2 = 40 2 (3 ) Από ( 1 ) και (3) έχουμε : α 2 -2αy+700=0 (4) Από (2) και (3) έχουμε : α 2 -2αχ-900=0 (5) Αντικαθιστώντας από την (4) και την (5) τα χ και y στην (3) παίρνουμε : α 4 - 3400α 2 + 650000 = Ο από όπου προκύπτει α = 56,54 μ. περίπου .

Το 64 = 65 : Το λάθος είναι ότι τα τέσ­σερα τμήματα δεν καλύπτουν ακριβώς το ορθογώνιο . Αφήνουν ακάλυπτο ακριβώς ένα τετραγωνάκι.

ΕΥΚΛΕΙΔΗΣ Β' 66 τ.2!70

Page 73: Ευκλειδης Β 66

Ε υ ικλ.εi δ π ς rιι ρ οτ ε i vε ι

«Η καρδιά των μαθηματικών είναι τα προβλήματα και οι λύσεις και ο κύριος λόγος ύπαρξης του μαθηματικού είναι να λύνει προβλήματω>.

P. R. HALMOS

Επιμέλεια: Γ. Τριάντος, Ν. Αντωνόπουλος, Θ. Κυριακόπουλος

1 09 . Έστω y Ε Ζ*. Αν χ ι , χ2 , ... , Χ0 Ε Ζ*

- { ι} με

η Ε Ν και (χ ι · χ2 • . . . • Χ0 )2 · y � 22( n+ ι ) και

Χι . χ2 . . . . . xn . Υ = u + ι με u Ε Ν ' να αποδειχθεί ότι ένας τουλάχιστο\' από τους ακεραίους αριθ­μούς Χι , Χ2 , ... , Χ0 , U εί\'α\ πρώτος. ( Επροτάθη από τον φοιτητή του Ε .Μ.Π \1 ιχαήλ Θ. Ρασσιά )

Λ ίJση ( από τον ίδιο ) Θεωρούμε ότι χ 1 • χ 2 •

• • • · χ η = χ . Τότε ισχύει ότι xy = u + 1 .

Υποθέτουμε ότι οι ακέραιοι αριθμοί x 1 , x � , . . . , X 0 , U δεν είναι πρώτοι. Στην περίπτωση αυτή καθένας από τους ακεραίους Χ ι , χ 2 , . . . , Χ 0 , u γράφεται σαν γινόμενο τουλάχιστον δύο πρώτων παραγόντων, οπότε τελικά ισχύει xu = Ρϊ. , Ρ ϊ_ : Ρ ϊ. , · · ·Ρϊ. ,, με m :=:: 2(n + 1) , λ, m Ε Ν .

Επομένως, λαμβάνουμε χu :=:: pλ, Ρλ2Ρλ3 · · ·Ρλzι π•Ι > · Όμως, γενικά ισχύει ότι:

Ρι :=:: 2 , i = 1 , 2, . . . , 2(n + 1) . . ,

Ά > 22 ( n+ Ι J 'Ε - z Τ ' ρα, xu _ . στω xu - a , a E . οτε, ισχύουν οι σχέσεις xy = u + 1 και xu = a . Πολλα­πλασιάζοντας τις σχέσεις αυτές κατά μέλη προκύ­πτει ότι x2yu = a(u + 1) και επομένως

χ 2 y = a(l + _!_) > a και επειδή ισχύει ότι a :=:: 22( η+ Ι ) u

έχουμε x2y>i(n+l) ' δηλ. (Χ ι . χ2 . . . . . x n )2 • Υ > 22( n+l ) άτοπο, λόγω της υπόθεσης. Συνεπώς, ένας τουλά­χιστον από τους ακεραίους x 1 , x 2 , . . . , X 0 , U είναι πρώτος αριθμός. Π αρατήρηση : Αν υποθέσουμε ότι χ , = χ 2 = . . . = xn = b ' όπου b μή πρώτος, τότε υ­ποχρεωτικά ο u θα είναι πρώτος.

1 1 1 . Τα σημεία Αι , Β ι , Γι βρίσκονται πάνω στις πλευρές ΑΒ , ΒΓ, ΓΑ τριγώνου ΑΒΓ αντίστοι­χα, διάφορα των κορυφών του. Αν είναι ΑΑ1 ΒΒ ι ΓΓ ι , -- = λ , -- = μ , -- = ν και Ε0 το εμβαδον Α1Β Β 1Γ Γ1 Α

του τριγώνου που ορίζουν τα σημεία τομής των

ευθυγράμμων τμημάτων ΑΒ ι , ΒΓ1 , Γ Αι ανά

δύο, τότε να υπολογίσετε το εμβαδόν Ε0 συναρ­

τήσει των λ, μ, ν και του εμβαδού Ε του τριγώ­νου ΑΒΓ . (Επροτάθη από τον συνάδελφο Ν ι κ ό­λα ο Βαδ ιβούλη - Άρτα ) Λύση (από τον συνάδελφο Γιώργο Αποστολό­πουλο - Μεσολόγγι )

Α

Φέρουμε τις ΑΛ,ΒΝ, ΓΜ . Είναι: (ΑΒΓ1 ) = ΑΓΊ =� (1) και (ΑΛΓ1 ) = ΑΓ1 = _!_ (2) . (ΒΓΓ1 ) ΓΓ1 ν (ΛΓΓ1 ) ΓΓ1 ν

Από τις (1) , (2) έχουμε :

(ΑΒΓ1 ) (ΑΛΓ1 ) (ΑΒΓ1 ) (ΒΓΓ1 ) --'-----'-'-- = � = � (ΒΓΓ1 ) (ΛΓΓ1 ) (ΑΛΓ1 ) (ΛΓΓ1 )

(ΑΒΓ1 ) - (ΑΛΓ1 ) (ΒΓΓ1 ) - (ΛΓΓ1 ) � = � (ΑΛΓ1 ) (ΛΓΓ1 )

� (ΑΒΛ) = (ΒΓΛ) � (ΑΒΛ) = (ΑΛΓ1 ) <;_!_ (3) (ΑΛΓ1 ) (ΛΓΓ1 ) (ΒΓΛ) (ΛΓΓ1 ) ν

Επίσης, είναι: (ΑΑιΓ) = ΑΑι = λ ( 4) και (ΒΑ1Γ) ΒΑ1

(ΑΑιΛ) = ΑΑι = λ (5) (ΒΑ1Λ) ΒΑ1

Από τις (4), (5) έχουμε (ΑΑ1Γ) (ΑΑ1Λ) (ΑΑ1Γ) (ΒΑ1Γ) --'------'---'-- = � = � (ΒΑ1Γ) (ΒΑjΛ) (ΑΑ1Λ) (ΒΑ1Λ)

(ΑΑ1Γ) - (ΑΑ1Λ) (ΒΑ1Γ) - (ΒΑ1Λ) � = � (ΑΑ1Λ) (ΒΑ1Λ)

(ΑΛΓ) = (ΒΓΛ) � (ΑΛΓ) = (ΑΑ1Λ) �λ (6) (ΑΑ1Λ) (ΒΑ1Λ) (ΒΓΛ) (ΒΑ1Λ)

Με πρόσθεση κατά μέλη των ισοτήτων (3 ) , ( 6) παίρνουμε

ΕΥΚΛΕΙΔΗΣ Β ' 66 τ.2/7 1

Page 74: Ευκλειδης Β 66

-------------- Ο Ευκλείδης προτείνει . . . --------------

(ΑΒΛ) + (ΑΛΓ) = _!_ + λ => Ε - (ΒΓΛ) = 1 + λν => (ΒΓΛ) · ν (ΒΓΛ) ν

=> Ε = 1 + ν + λν => (ΒΓΛ) = ν Ε (Ί) (ΒΓΛ) ν 1 + ν + λν

Όμοια, έχουμε (ΑΓΝ) = λ Ε (8) , 1 + λ + λμ

(ΑΒΜ) = μ Ε (9) 1 + μ + νμ

Τελικά: Ε0 = Ε - (ΒΓΛ) - (ΑΓΝ) - (ΑΒΜ) =

= Ε - ν Ε - λ Ε - μ Ε 1 + ν + λν 1 + λ + λμ 1 + μ + νμ

= E (l - ν λ _ μ ) = . . . = 1 + ν + λν 1 + λ + λμ 1 + μ + νμ

(1 - λμν)2 = Ε (1 + ν + λν)(l + λ + λμ)(l + μ + νμ)

Λύσεις επίσης έσtειλαν οι συνάδελφοι: Αντιί)\ης Ι ωαννί­δης - Λάρισα, Ροδ6λφος :\1πίφης - Δάφνη Γιά\'\ης 2:τα­ματογιά\'\11ς - Δροσιά, Γεώργιος Τσαπακίδης - Αγρίνιο και ο Πολιτικός Μηχανικός Ι ωά\'\11ς Α νδρής - Αθήνα. 1 1 2 . Να βρείτε τους μη αρνητικούς ακέραιους x,y, w, ώστε να ισχύει

[2χ + 3Υ + �(2χ + 3Υ )2 + 1 ] (w2 - �w4 + 1 ) = -1

(Προτείνεται από τον συνάδελφο Χρήστο Δεμιρ­τζόγλου - Λράμα )

Λ ίJση ( από τον ίδιο ) Θέτουμε γ + 3Υ = a , - w2 = b οπότε η δο­

θείσα σχέση γράφεται (a + �)(b + �b2 + 1 ) = 1 (1)

Όμως, (a + �)(a - �) = - 1 (2) . Από τις ισότητες ( 1 ), (2) παίρνουμε:

b + �b2 + 1 = -a + �a2 + 1 (3)

Ομοίως, επειδή ισχύει (b+Jb2 +1)(b-Jb2 +1)=-1 ,

από την (1) έχουμε b -Jb2 + 1 = -a -Ja2 + 1 (4) Με πρόσθεση κατά μέλη των ισοτήτων (3) , (4) έ­χουμε : 2b = -2a => a + b = Ο . Άρα 2χ + 3Υ - w2 = 0 ή το αυτό 2 χ + 3Υ = w2 (5) . Διακρίνουμε περιπτώσεις : 1 . Αν y = Ο η ισότητα (5) παίρνει τη μορφή : 2χ - 1 = w2 οπότε 2x =W -1<::::>2x =(w+1Xw-1) (6) . Από την τελευταία ισότητα προκύπτει ότι οι ακέραιοι αριθμοί w + 1, w - 1 είναι δυνάμεις του 2 . Οι μόνες δυνάμεις του 2 που έχουν διαφορά 2 είναι οι ακέ­ραιοι 2 , 4 . Συνεπώς, w + 1 = 4 , w - 1 = 2 και τελικά w = 3 . Από την (6) έχουμε 2χ = 8 <::::> χ = 3 . Άρα η τριάδα (x , y, w) = (3, 0, 3) είναι μία λύση του προ-

βλήματος. χ χ

2. Αν y > O η (5) δίνει: Y =W -2x =(w+22Xw-22 ) και για x=2p,p εN είναι 3Υ = (w + 2P )(w - 2P ) (6)

Από την ( 6) προκύπτει ότι καθένας από τους όρους του γινομένου πρέπει να είναι δύναμη του

3 . Θέτουμε: μ, ν ε Ν . (7) {w + 2P = 3μ W - 2p = 3v

Από τις σχέσεις (7) με αφαίρεση παίρνουμε : 3μ - 3ν = 2p+I (8) ενώ η ( 6) παίρνει την μορφή : 3Υ = 3μ+ν (9) . Το δεύτερο μέλος της ισότητας (8) διαιρείται με το 3 μόνον όταν είναι ν = Ο , οπότε έχουμε 3μ - 1 = 2p+I (1 Ο) .

Η (1 Ο) για p = Ο ( και άρα χ = Ο ), δίνει 3μ = 3 => μ = 1 .Από την (7) παίρνουμε w = 2 ενώ από την (9) παίρνουμε y = 1 . Συνεπώς, η τριάδα (χ, y, w) = (0, 1 , 2) είναι λύση του προβλήματος.

Για p ε Ν* , οπότε χ = 2p , η (1 Ο) δίνει: μ μ

2p+l = (32 + 1)(32 - 1) Από την τελευταία ισότητα προκύπτει ότι κα­

θένας από τους παράγοντες του δευτέρου μέλους της πρέπει να είναι δύναμη του 2 με διαφορά 2 .

� � Άρα πρέπει 32 + 1 = 4, 32 - 1 = 2 , απ όπου προκύπτει μ = 2 . Τότε 2p+1 = 8 = 23 =>p = 2 , οπότε χ = 4 , ενώ από την (9) έχουμε 3Υ = 32 => y = 2 . Τέλος, από τις σχέσεις (7) προκύπτει ότι w = 32 - 22 = 5 και η τριάδα (x , y, w) = (4, 2, 5) είναι μία λύση του προ­βλήματος. Άρα, το σύνολο λύσεων Λ της εξίσω­σης είναι Λ = { (3, 0, 3), (0, 1 , 2), (4, 2, 5)} . 1 1 3 . Δίνεται κανονικό επτάγωνο ΑΒΓΔΕΖΗ . Αν ονομάσουμε β το κοινό μήκος των μικροτέρων και γ το κοινό μήκος των μεγαλυτέρων διαγω-

νίων του, να δείξετε ότι ο λόγος 1!_ είναι λύση γ

της εξίσωσης χ3 + 2χ2 - χ - 1 = Ο . (Επροτάθη από τον συνάδελφο Γ. i\ ικητάκη - Σητεία Λασιθίου)

Λύση J 'l ( από τον ίδιο ) Έστω ΑΓ = β , ΑΔ = γ και α η πλευρά του

κανονικού επταγώνου . Εφαρμόζουμε το πρώτο Οεώρη μα του Πτολεμαίου στα τετράπλευρα ΑΒΓ Δ και ΗΒΓ Δ και παίρνουμε αντίστοιχα: ΑΓ · ΒΔ = ΑΒ · ΓΔ + ΒΓ · ΑΔ => β2 = α2 + αγ (1) ΗΓ · ΒΔ = ΗΒ · ΓΔ + ΒΓ · ΗΔ => γβ = αβ + αγ (2)

Με διαίρεση κατά μέλη των ( 1 ) , (2) παίρνου-

ΕΥΚΛΕΙΔΗΣ Β ' 66 τ.2/72

Page 75: Ευκλειδης Β 66

--------------- Ο Ευκλείδης προτείνει . . . --------------

με : Q.

= α + γ � α = β2 + βγ - γ2

(3) . γ β + γ γ

Από την (2) έχουμε: α = � (4) . Από τις (3), (4) β + γ Β

γεγονός, που αποδεικνύει το ζητούμενο. Λύση 2'� ( από τον συνάδελφο Γ ιώργο Τσαπακί­δη - Αγρίνιο )

Η

Β

Ζ Ε

Είναι ΑΔr = 2π = θ , Μ Δ = 4π = 2θ . Από 7 7

τον νόμο των ημιτόνων στο τρίγωνο ΑΓ Δ έχουμε : β γ β ημθ β 1 - = -- � - = -- � - = (1)

ημθ ημ2θ γ ημ2θ γ 2συνθ

Για να είναι ο Q.

ρίζα της εξίσωσης γ

χ3 + 2χ2 - χ - 1 = Ο , αρκεί να Ισχύει:

(-1-)3 + 2(-1-)2 - -1- - 1 = Ο ή το αυτό : 2συνθ 2συνθ 2συνθ

(2) Θεωρούμε τον μιγαδικό z = συνθ + ίημθ .

Αλλά z7 = συν7θ + ίημ7θ = συν2π + ίημ2π = Ι ,

άρα z7 - I = O<::::> (z- l)(z6 +z5 + z4 + z3 +r +z+ I) = O και επειδή είναι z * Ι , έχουμε : z6 + z5 + z4 + z3 + z2 + z + Ι = Ο <=>

Ι Ι 1 <::::> z3 + z2 + z + 1 + - + -2 + 3 = Ο z z z

Ι 1 Ι <::::> (z3 + -τ) + (z2 + -7 ) + (z + -) + Ι = Ο <=>

z z- z

Ι 1 1 1 <::::> (z + -)3 - 3(z + -) + (z + -)2 - 2 + (z + -) + Ι = Ο z z z z Ι 3 Ι 2 Ι

<=> (z + -) + (z + -) - 2(z + -) - 1 = 0 (3) z z z

Όμως, z +.!. = συνθ + ίημθ + συνθ- ίημθ = 2συνθ , z

οπότε η εξίσωση (3) είναι η ζητούμενη εξίσωση (2) . Λύσεις επίσης έστειλαν οι συνάδελφοι Αντιίη·ιΕ Ι ωαν\· ί­δης - Λάρισα, Γιtiψγος Αποστολόπουλος - Μεσολόγγι και ο πολιτικός Μηχανικός κ. Α νδρής I ωάννης - Αθήνα.

Π ρ οτεινόμενες Ασκήσεις 1 29 . Ονομάζουμε ρ την ακτίνα του εγγεγραμμένου κύ­κλου σ' ένα τρίγωνο ΑΒΓ. Οι εφαπτόμενες του κύκλου αυτού που είναι παράλληλες στις πλευρές του τριγώνου ΑΒΓ σχηματίζουν με τις πλευρές του τριγώνου αυτού τα τρίγωνα ΑΑ ιΑ2, ΒΒ ιΒ2 και ΓΓ ι Γ2 . Ονομάζουμε ρ ι , ρ2 , και ρ3 τις ακτίνες των εγγεγραμμένων κύκλων στα τρίγωνα αυτά, αντιστοίχως. Να δείξετε ότι: ρ=ρ ι+ρ2+ρ3 . (Προτείνεται από τον συνάδελφο Λνη!Jνη Κυριακ<ι­πουλο - Α θήνα)

1 3 0 . Να αποδείξετε ότι για οποιοδήποτε μιγαδικό αριθ-

μό z, z;tO, ισχύει: l z + il + �� + i l > lz + �� (Π οτείνεται απότον συνάδελ ο :\ ίκο Λντωνδπουλο_: Ι λιον)

Διόρθωση Στην προτεινόμενη άσκηση 1 1 8 του τεύ­

χους 64, αντί (Κ ι , Κ2 ) να γραφεί (Κ ιΚ2 )

Το β ι βλίο περιδιαβαίνει το χώρο των

Μαθηματικών Κατεύθυνσης της Γ Λυκείου και

φιλοδοξεί να αποτελέσε ι ουσιαστι κό βοήθημα γ ια

τους μαθητές-υποψήφιους και τους καθηγητές.

Παράλληλα , επιδιώκει να συστήσει τ η βάση

πάνω στην οποία θα αναπτυχθεί ένας γόνιμος

προβλη ματισμός γύρω από τα θέματα και το

πνεύμα των Π ανελλαδι κών Εξετάσεων.

ΕΚΔΟΣΕΙΣ ΜΑΥΡΙΔΗ Τηλ. : 231 0 228009 • Fax: 231 0 287097

Κεντρική Διάθεση : Σ. Κοκοτσάκης Τηλ: 2 1 0 3804347 - 23 1 0 2303 1 0

ΕΥΚΛΕΙΔΗΣ Β ' 6 6 τ.2/73

Page 76: Ευκλειδης Β 66

Νιοστές Ρίζες., το σύμβολο \Γα και τα σχολικά βιβλία

1 . Ρ ΙΖΕΣ ΣΤΟ ΣΥΝΟΛΟ R+o= I O ,+oo) Θεώρημα κα ι ορ ισμός. Δοθέντος, ενός

πραγματικού αριθμού �Ο και ενός φυσικού αριθμού ν>Ο, υπάρχει ένας μοναδικός πραγματικός αριθμός β;:::Ο με βν=α. Ο αριθμός αυτός β ονομά­ζεται νιοστή ρίζα του α στο σύνολο IR+0 και συμβολίζεται με ifi; .

Έτσι με α,β ε!R+0 και νεlΝ* , έχουμε : βν = α � β = 4α .

Π . χ. έχουμε : .J4 = 2 (το 4 δεν έχει άλλη τε­τραγωνική ρίζα στο IR+0) . � =2 (το 8 δεν έχει άλλη κυβική ρίζα στο IR+0) . Στη συνέχεια αποδεικνύονται όλες οι γνωστές

ιδιότητες των ριζών.

2 . Ρ Ι ΖΕΣ ΣΤΟ R Ορ ισμός. Δοθέντος, ενός αριθμού αε!R και

ενός φυσικού αριθμού ν>Ο, κάθε α­ριθμός x e!R με χν=α ονομάζεται μία (πραγματική) νιοστή ρίζα του α (στον ορισμό αυτό δεν ι:ννοοίψε κ α ­

νένα σίψβολο ). Έτσι με α, χ ε!R και ν εlΝ*, έχουμε :

(χ νιοστή ρίζα του α) � χν = α. Αποδεικνύεται εύκολα ότι:

ί) Το Ο έχει μια μόνο νιοστή ρίζα, το 0: fo=O . ίί) Έστω ότι α>Ο. Τότε :

• Α ν ο ν είναι άρτιος, ο α έχει δυο νιοστές ρίζες, τις χ = \Γα και χ = -\Γα .

• Αν ο ν είναι περιττός, ο α έχει μία μόνο νι­οστή ρίζα, την χ = \Γα .

ί ί ί) Έστω ότι α<Ο. Τότε • Α ν ο ν είναι άρτιος, ο α δεν έχει νιοστές

ρίζες. • Αν ο ν είναι περιττός, ο α έχει μία μόνο νι-

οστή ρίζα, την χ = -� = -νfαj . Π αραδείγματα

α) Το 4 έχει δυο τετραγωνικές ρίζες τις 2 και -2, γιατί: χ2=4� (χ=2 ή χ=-2) . Έχουμε : 2 = .J4 και -2 = -.J4 (και όχι .J4 = ±2 ) .

β) Το 27 έχει μια μόνο κυβική ρίζα το 3 , γιατί: χ3=27�χ=3 . Έχουμε 3 = ifii .

του Αντώνη Κυριακόπουλου

γ) Το -4 δεν έχει τετραγωνικές ρίζες, γιατί η εξί-2 4 ' δ ' σωση : χ =- ειναι α υνατη .

δ) Το -8 έχει μια μόνο κυβική ρίζα το -2 , γιατί: χ3 = 8 � χ = -2 . Έχουμε:

-2 = -{/Fsί = -� ο

Ση μείωση . Ο παραπάνω ορισμός επεκτείνε­ται και στο σύνολο των μιγαδικών αριθμών C. Δη­λαδή , με α, z ε C και νεlΝ*, ορίζουμε :

(z νιοστή ρίζα του α) � z ν=α. Μάλιστα δε στο C δεν ορίζουμε σύμβολο για

καμιά νιοστή ρίζα . Για παράδειγμα, η έκφραση : -./3 + 2ί δεν έχει νόημα.

3 . Τ Ι ΓΡΑΦΟΥl\ ΤΑ Σ Χ Ο. \ Ι ΚΑ Β Ι Β Λ Ι Λ Το βιβλίο των μαθηματικών της Β ' Γυμνασίου

δίνει τον ορισμό της τετραγωνικής ρίζας μόνο στο σύνολο IR+o και μάλιστα χωρίς να αναφέρει τίποτα για την ύπαρξη (σελίδα 4 1 ) . Το ίδιο κάνει και το βι­βλίο της Γ Γυμνασίου (σελίδα 20) . Το ίδιο κάνει και το βιβλίο της Α ' Λυκείου για τη νιοστή ρίζα (σελίδα 44) . Μάλιστα, το βιβλίο αυτό στη σελίδα 45 παρατηρεί ότι ο αριθμός \Γα ( α;:::Ο) είναι η μη αρ­νητική λύση της εξίσωσης χν=α. Έτσι αν η εξίσωση αυτή έχει και μια αρνητική λύση, τότε αυτή δεν εί­ναι μια νιοστή ρίζα του α ! ! ! Με άλλα λόγια, σύμ­φωνα με τα σχολικά βιβλία, π.χ. το -2 δεν είναι μια τετραγωνική ρίζα του 4, αν και (-2)2 = 4. Ε ίναι φα­νφό ότι έχουν μπr.ρδέψει την έννο ια των ν ιοστών

ρ ιζών ενός αριΟμοίJ α ε R με το σύμβολο Va .

Αξίζει να σημειωθεί ότι το βιβλίο των μαθημα­τικών της ΓΆυκείου (θετικής και Τεχνολογικής Κατεύθυνσης, έκδοση 2007) στη σελίδα 1 1 5 ανα­φέρει ότι οι λύσεις της εξίσωσης z ν= 1 , όπου zεC, λέγονται νιοστές ρίζες της μονάδας. Λοιπόν; Άλλα λένε στα παιδιά στην Α ' .\υκείου και άλλα στη Γ Λυκείου ;

Θέλω να σημειώσω ακόμα ότι τα νέα βιβλία των μαθηματικών και των τριών τάξεων του Γυ­μνασίου, εκτός από το λάθος που αναφέρω παραπά­νω, έχουν και άλλα πολλά λάθη, με αποτέλεσμα σε πολλά σημεία να μπερδεύουν τους μαθητές, να τους στέλνουν λανθασμένα μηνύματα και να τα μαθαί­νουν να δουλεύουν μηχανικά. Τ α λάθη αυτά τα έχω επισημαίνει στους συγγραφείς, καθώς και στο Παι­δαγωγικό Ινστιτούτο. Ελπίζω να διορθωθούν το συ­ντομότερο δυνατόν.

ΕΥΚΛΕΙΔΗΣ Β' 66 τ.2174

Page 77: Ευκλειδης Β 66

π I

Η στήλη αυτή έχει ως στόχο την ανάπτυξη μαθηματικού διαλόγου. Φιλοδοξούμε να συμμετάσχουν όλοι όσοι έχουν ένα γενικότερο ενδιαφέρον για τα Μαθηματικά.

Επιμέλεια: Γιάννης Στρατής - Βαγγέλης Ευσταθίου

Μια γεωμετρική ερμηνεία του θέματος 4β των Πανελλαδικών εξετάσεων του 2007

Δόρτσιος Κων/νος, Βρέκας Χάρης, Καραφέρης Τριαντάφυλλος

Κατά τις πρόσφατες Πανελλαδικές εξετάσεις Μαίου- Ιουνίου 2007, το 4° Θέμα παρουσίαζε ένα ενδιαφέρον και μια «δυσκολία» κυρίως στο δεύτερο ερώτημα, το οποίο απαιτούσε την απόδειξη της ισχύος μιας ανισότητας που περιείχε τη γνωστή συνάρτηση ,

χ

F(x) = fJ(t)dt α

όπου f συνεχής συνάρτηση ορισμένη σε διάστημα Ι και α Ε Ι . Παραθέτουμε στο άρθρο αυτό μια προσπάθεια γεωμετρικής ερμηνείας της ανισότητας που

πραγματεύεται το θέμα αυτό, και ειδικότερα το δεύτερο ερώτημα, χρησιμοποιώντας κατάλληλο λογισμικό τριών διαστάσεων και συγκεκριμένα το Cabri3D. Επειδή δεν είναι δυνατό να παρουσιαστεί σε δυναμική μορφή η κινητικότητα του σχήματος ώστε να το βλέπει κανείς από διάφορες οπτικές γωνίες, παραθέτουμε διάφορα «στιγμιότυπα» της εξέλιξης του σχήματος, ώστε στο τέλος να γίνει αντιληπτή η σχέση των στερεών που προκύπτουν. Πιστεύουμε ότι η χρήση τέτοιων λογισμικών διευκολύνει την καθημερινή δουλειά του μαθηματικού στην τάξη, την κάνει πιο αποτελεσματική , κι ακόμα προάγει τη δημιουργική φαντασία και αναπτύσσει τη διαισθητική ικανότητα των μαθητών μας.

Θ Ε !\'Ι Α 4° Έστω f μια συνεχής και γνησίως αύξουσα συνάρτηση στο διάστημα [ο, ι] για την οποία ισχύει

/(0))0. Δίνεται επίσης συνάρτηση g συνεχής στο διάστημα [ο, ι] για την οποία ισχύει g(x))O για

κάθε χ Ε [ο, ι] . χ

Ορίζουμε τις συναρτήσεις: F (x) = jJ( t )g( t)dt, χ Ε [Ο, ι] , ( 1 ) ο

χ G(x) = Jg(t)dt, χ Ε [Ο, ι ] . (2)

α. Ν α δειχθεί ότι F (χ)) Ο για κάθε χ στο διάστημα (Ο, ι] . β. Να αποδειχθεί ότι:

για κάθε χ στο διάστημα (Ο, ι] γ. Να αποδειχθεί ότι ισχύει:

για κάθε χ στο διάστημα (Ο, ι] .

J (x) · G (x))F (x) (3)

F (x) F ( ι) G(x) � G(ι) (4)

ΕΥΚΛΕΙΔΗΣ Β' 66 τ.2/75

Page 78: Ευκλειδης Β 66

Το Βήμα του Ευκλείδη

ΛίJσΈ1 : Θα ασχοληθούμε μόνο με το ερώτημα 4β αναφέροντας αρχικά τη λύση που κυκλοφόρησε αμέσως

μετά τις εξετάσεις. I oc η16πος Η ζητούμενη σχέση (3 ), λόγω των ( 1 ) και (2), γράφεται:

αυτή ισοδυναμεί με την :

κι ακόμα με την :

Δηλαδή :

.τ :r

f (x) · fg ( t )dt) fJ (t ) g ( t )dt, Vx E (0, 1 ] ο ο

.\'" χ

fJ (x) · g ( t )dt) fJ (t ) g ( t )dt , \ix Ε ( 0, 1 ] ο ο χ

f[J (x) · g ( t ) - f ( t ) g ( t )}it)O, \ix Ε ( 0, 1 ] ο χ

f[J (x) - f (t )} g ( t ) dt)O, Vx E (0, 1 ] (5) ο

Όμως επειδή η ι είναι γνησίως αύξουσα, θα είναι: f (χ) - f ( t ) � Ο, \ix Ε ( Ο, 1 ] , Vt : ο ::;; t ::;; χ \εξάλλου : g (χ ))0, \ix Ε [ Ο, 1 ]

Άρα η σχέση (5) αληθής και συνεπώς η (3) αληθής.

'�� Η λύση αυτή προτάθηκε στα Βαθμολογικά από την Επιτροπή των εξετάσεων του Υπουργείου Παιδείας, αλλά και από διάφορους άλλους οι οποίοι ασχολήθηκαν με το θέμα και δημοσίευσαν λύσεις.

2"c τρόπος_ Έστω ότι η σχέση (3) δεν είναι αληθής. Τότε θα υπάρχει ένα χ0 Ε (Ο, 1 ] , τέτοιο ώστε να ισχύει:

f(x0 ) • G(x0 ) ::;; F(x0 ), (6) Θεωρούμε τη συνάρτηση : h(x) = f(x0 ) • G(x) - F(x), χ Ε [Ο, χ0 ]

η οποία ικανοποιεί τις προϋποθέσεις του Θ.Μ.Τ. του Διαφορικού Λογισμού στο διάστημα [Ο, χ0 ] καθόσον οι συναρτήσεις F και G είναι παραγωγίσιμες στο διάστημα αυτό και η ποσότητα f(x0 ) αποτελεί μια σταθερή ποσότητα.

Άρα θα υπάρχει ένα τουλάχιστον ξ Ε ( Ο, χ0 ) τέτοιο ώστε να ισχύει:

h'(ξ) = h(x0 ) - h(O)

= h(x0 )

= f(x0 ) • G(x0 ) - F(x0 ) , (?)

Χο Χο Χο

Επειδή χ0 )0 η (7) λόγω της (6) δίνει ότι: h'(ξ) ::;; Ο, (8)

Όμως

Δηλαδή :

h'(x) = f(x0 ) • G'(x) - F'(x) = f(x0 ) • g(x) - f(x) · g(x) = [f(x0 ) - f(x)] · g(x) h'(x) = [f(x0 ) - f(x)] · g(x) (9)

Η σχέση (8) λόγω της (9) γίνεται: [f(x0 ) - /(ξ)] · g(ξ) ::;; Ο

Η τελευταία δεν ισχύει γιατί εφόσον Ο(ξ(χ0 ( 1 και η f είναι γνησίως αύξουσα άρα θα ισχύει:

f(x0 ) - /(ξ))Ο Εξάλλου και g(x))O, Vx E [0, 1 ] . Άρα η υπόθεσή μας (6) είναι ψευδής και συνεπώς αληθής είναι η (3) .

ΕΥΚΛΕΙΔΗΣ Β ' 66 τ.2176

Page 79: Ευκλειδης Β 66

Το Βήμα του Ευκλείδη

Γεωμετρική ερμηνεία Θεωρούμε ένα τρισορθογώνιο σύστημα αναφοράς το ΟΧΨΖ. Στο επίπεδο ΧΟΨ θεωρούμε τη

z

Σχήμα 1

γραφική παράσταση C1 της f η οποία είναι μια συνεχής και γνησίως αύξουσα συνάρτηση στο [0, 1] με

/(0))0 . (Σχήμα 1 ) Επίσης στο επίπεδο ΧΟΖ θεωρούμε το γράφημα Cg της g , η οποία είναι μια συνεχής συνάρτηση

στο [0, 1] με g(x))O, \fχ ε [Ο, 1] (Σχήμα 2).

z

g(o)

Σχήμα 2

ΕΥΚΛΕΙΔΗΣ Β ' 66 τ.2/77

Page 80: Ευκλειδης Β 66

Το Βήμα του Ευκλείδη

Στο σχήμα 3 το εμβαδόν του χωρίου (Ω) εκφράζεται από το ορισμένο ολοκλήρωμα της συνάρτησης

z

(Ω)

• Μ . '

\ jB \�

g(o)

χ ο

g κατά τον άξονα των ΟΧ από το Ο έως το χ0 • Δηλαδή : Ε(Ω) = J g(u)du ο

Σχήμα 3

Τέλος στο σχήμα 4 φαίνεται το στερεό (ΑΒΓΟΕΖΗΔ), το οποίο έχει όγκο � που δίνεται από το

1 . .

. • •

\. .. •

• •

z

γινόμενο: (εμβαδόν βάσης)Χ(ύψος)=(ΑΟΕΖ)Χ(ΟΓ), δηλαδή : � �

� = Ε(Ω) · f(x0 ) = ( Jg(u)du) · f(x0 ) = f(x0 ) · ( Jg(u)du) ο ο

ή ακόμα:

ΕΥΚΛΕΙΔΗΣ Β' 66 τ.2/78

Σχήμα 4

Page 81: Ευκλειδης Β 66

Το Βήμα του Ευκλείδη

Στο σχήμα 5 θεωρούμε μια τιμή u της μεταβλητής χ πάνω στον άξονα των ΟΧ και τις τιμές f(u), g(u) των συναρτήσεων f και g οι οποίες εκφράζουν τα μήκη των ευθυγράμμων τμημάτων (ΜΝ) και (ΜΣ) αντίστοιχα. Τα μήκη αυτά ξεκινούν από το σημείο Μ με τετμημένη u και καταλήγουν στα σημεία Ν και Σ που ανήκουν στα γραφήματα C1 κα Cg αντίστοιχα.

z

Σχήμα 5

Τ ο γινόμενο (ΜΝ)· (ΜΣ) εκφράζει το εμβαδόν του ορθογωνίου (ΜΝΡΣ) και αν αυτό πολλαπλασιασθεί με τη στοιχειώδη αύξηση du της μεταβλητής u , που εκφράζει την τετμημένη σημείου Μ, τότε προκύπτει ο όγκος του στοιχειώδους «πρισματικού» στερεού (ΜΝΡΣ, du ), που έχει βάση το ορθογώνιο (ΜΝΡΣ) και ύψος το στοιχειώδες τμήμα μήκους du . Άρα είναι: V(MNΣP, du) = f(u) · g(u)du

z

g(o)

Cf . . . . . .

ι

Σχήμα 6

Α��ω�---------;�--------� 1

Α ν στη συνέχεια θεωρήσουμε όλα αυτά τα «πρισματικά πακέτα» που δημιουργούνται με τον τρόπο που περιγράψαμε ανωτέρω, τότε η «άθροιση» αυτή , δηλαδή η «ολοκλήρωση» της Φ(u) = f(u) · g(u) από το Ο έως το χ0 θα δώσει ως αποτέλεσμα ότι: v; = }f(u)g(u)du = F(x0 ) ( 1 1 )

ο

όπου V2 ο όγκος του πρισματοειδούς (ΟΑΒΓ1ΕΖΗΘ) ο οποίος είναι προφανές ότι είναι μικρότερος του li'ι που είναι ο όγκος του πρισματοειδούς (ΟΑΒΓΕΖΗΘ) γιατί το ένα πρισματοειδές περιέχεται μέσα στο άλλο. Άρα: li'ι ) V2 δηλαδή : f (x0 ) · G (x0 ))F (x0 ) , χ0 ε (Ο, Ι ]

ΕΥΚΛΕΙΔΗΣ Β ' 66 τ.2/79

Page 82: Ευκλειδης Β 66

ΕΛΛΗΝΙΚΉ ΜΑΘΗΜΑτΙΚΉ ETAIPEIA

Πανεπιστημίου (Ελευθερίου Βενιζέλου) 34 1 06 79 ΑΘΗΝΑ • ο

GRE E K MATH E MAτiCAL SOC IETY

34, Panepistimiou (Eieftheriou Venizelou) Street GR. 1 06 79 - Athens - HELLAS

Τηλ. 3 6 1 6532 - 3 6 1 7784 - Fax : 364 1 025 e-mai l : [email protected]

www.hms.gr

Tel . 36 1 6532 - 3 6 1 7784 - Fax: 364 1 025 e-mai l : [email protected]

www.hms .gr

Σ Ε Μ Ι Ν ΑΡ Ι Ο Ε Π ΙΜ Ο Ρ Φ Ω Σ Η Σ

Μ Α Κ ΡΑΣ Δ Ι Α ΡΚΕ Ι Α Σ

Γ Ι Α Π ΤΥ Χ Ι Ο ΥΧ Ο '\'"Σ Μ Α Θ Η ΜΑ Τ Ι Κ Ο Υ Σ

Η Ελληνική Μαθηματική Εταιρεία (Ε.Μ.Ε.) είναι επιστημονικό σωματείο που ιδρύθηκε και λειτουργεί από το 1 9 1 8 . Μέλη της είναι Μαθηματικοί από όλους τους χώρους του Δημόσιου και Ι­διωτικού τομέα κι όλων των βαθμίδων της Εκπαίδευσης. Διαθέτει 34 παραρτήματα σε όλη την Ελ­λάδα. Κύριο μέλημα της είναι η αναβάθμιση της Μαθηματικής Παιδείας και γενικότερα της παι­δείας στη χώρα μας. Στους κύριους σκοπούς της Εταιρείας συγκαταλέγονται : • η πρόοδος και διάδοση των Μαθηματικών. • η διοργάνωση επιμορφωτικών σεμιναρίων στα μαθηματικά. • η διοργάνωση επιμορφωτικών σεμιναρίων στη πληροφορική . • η προαγωγή της ελεύθερης ανταλλαγής πληροφοριών μεταξύ μαθηματικών. • η ουσιαστική συνεισφορά στη συνεχή βελτίωση της μαθηματικής εκπαίδευσης και την πρόοδο

της γενικής παιδείας. • η ενίσχυση της μαθηματικής έρευνας.

Η Ε.Μ.Ε. ανταποκρινόμενη στην απαίτηση των συναδέλφων οργανώνει σεμινάριο επιμόρφω­σης για πτυχιούχους μαθηματικούς που επιθυμούν να απασχοληθούν στην εκπαίδευση , στις παρα­κάτω ενότητες.

μέσον) .

άτων: ΠΑΡΑΣΚΕΥΉ ( 1 7 :00-2 1 :00) , ΣΆΒΒΑΤΟ ( 1 0 :00- 14 : 00) και ΚΥΡΙΑΚΗ ( 1 0 :00- 1 4 :00

Έναρξη Μαθημάτων Παρασκευή 1 6/5/2008, ώρα 1 8.30 Γ Ε Ν Ι Κ Ε Σ Π Λ Η Ρ Ο Φ Ο Ρ Ι Ε Σ

ΠΛΗΡΟΦΟΡΙ ΕΣ :

ΩΡΕΣ 20 50 40 30 1 0

Καθημερινά στα γραφεία της Ε.Μ.Ε. Πανεπιστημίου 34 1 06 79 ΑΘΗΝΑ ή στα τηλέφωνα 2 1 0 3616532 -361 7784 fax 2 1 0 3641025 από τις 08 :00 έως 1 5 :00 και από 1 7 :00 έως 22:00. ΑΠΗΣΕ ΙΣ :

α) . Αίτηση με φωτογραφία β) . Αντίγραφο πτυχίου. Οι αιτήσεις θα γίνονται δεκτές όλες τις μέρες της εβδομάδας από τις 08 :00- 1 5 :00 και 1 7 :00-2 1 :00

Page 83: Ευκλειδης Β 66

Μόλις κυκλοφόρησαν τα βιβλία με τα θέματα των Διαγωνισμών 1 997 - 2007 της Ε.Μ.Ε.

Ο ΘΑΛΗΣ, Ο ΕΥΚΛΕΙΔΗΣ, Ο ΑΡΧΙΜ ΗΔΗΣ, Ο ΜΕΣΟΓΕΙΑΚΟΣ ΜΑΘΗΜΑτΙΚΟΣ ΔΙΑΓΩΝ ΙΣΜΟΣ

Με τα πρ β ατα.

203 σελίδες 20€

./ Ο θλΝfΣ ../ Ο Ε'ι'ΚλΕtΙΙΗΣ ./ OAPXJMHAHI

1 997 . 2007

./ Ο ΜΠΟΠΙλrοΣ ΜΑΘΗΜιt1ΙΚΟΣ ΔWΩΝt!ΜΟΣ

• Το wροβλήμστο • Οι λιίοειςτοuς • Ιστοριιcό C1Ι')μtlώ.,στα

--

360 σελίδες 30€

Κεντρική διάθεση : Ελληνική Μαθη ματική Εταιρεία Πανεπιστη μίου 34 - 1 06 79 Αθήνα Τηλ. : 2 1 0 361 6532 , 2 1 0 36 1 7784 fax: 2 1 0 364 1 025 [email protected]

ΜΑΘΗΜΑΤΙΚΕΣ ΠΡΟΚΛΗΣΕΙΣ

Πρψήματα yισ την σνάιπuξη της μσtημσιικής cnιέφης ΚΥΚΛοtΟΡΕΙ ΠΑ ΑΠΛΙΚΑ ΚΑΙ ΠΑ ΓΑΛΛΙΚΑ

ΙιιWιιb Yll IIIJιm DU ΙUιαειέΧΟUV ιτιικ ιιονrιmοuιίΚ • ΕDιινιιίΚ ΙιΒιιιιιmιίκ

το gf&'lιo nεJ>ιεχει :

Μf.ΡΟΣ 14 Α. nEPt ΑΡΙθ11ΩΝ �Ι ΤΩΝ ι-rr'8/!ΡΙΩΝ ΤΟ\'Σ

Μf.ΡΟΣ 1• Ε. tεrΟΡΙι<Α ΣrOIXEI4 Ano ΤΑ EMf/NI� θΕΜΕΛΙΑ

ΤΩΝ MAθf/MAΠWJ

ΜΕΡΟΣ 2• Α. nPog{lf/MA1A ΜεΣΧf/ΜΑΤΑ, ΝΟΜΙΣΜΑ1Α, ΣnΙΡfΑ � ΜΕΡΟΣ 2• Ε. ΓΕΩΜΕrΡΙΚΟ/ �/φΟ/

Μf.ΡΟΣ 2· r. �ιφσι rιοnιαιε

Μf.ΡΟΣ 24 D. ΑΡΙθΜΟθΕΩΡf/111(01 �ΙΦΟ/

ΜΕΡΟΣ 24 Ε.. �ΙΦΟΙ WDYAΣrtιaιΣ

Μf.ΡΟΣ 2• Σf. �ΙΦDΙ εΙΡΑ1f/ΠιαιΣ

Μf.ΡΟΣ 3ο Α. ΑΝθΟιΊΟΠΟ θΕΜΑ1ΩΝ MAθiiMA11WJ ΔΙΑΓΩΝΙΣΜΩΝ

Τf/Σ ΕΜf/ΝtιαιΣ ΜΑθf/ΜΑ1ΙιαιΣ εrΑΙΡΕΙ4Σ

Α. rvMNAΣIO E. �Yt<E/0

ΜΕΡΟΣ 3ο Ε. ΑΡΧθΟ θΕΜΑ1ΩΝ MAθiiMA11WJ

ΔΙΑΓQΝΙΣΜΩΝ ΑΜΩΝ ΚΡΑ1ΩΝ

Μf.ΡΟΣ ι,. AnAN1f/ΣEIΣ

Page 84: Ευκλειδης Β 66

< Ν

...ι < cι:: Ζ c:.:ι < -

ι­W cι::

ι::::! :Ε w w :Ε ι­w < ...Ι :Ε = σ α: α..

από r ιs ε κδόσε ι s <<Δ ΙΟΦΑΝΤΟΣ>>

Γιώργος Μ. Μιχαηλίδης

ΜΑθΗΜΑΤΙΚΑ Γ ΛΥΚΕΙΟΥ

ΤΟΜΟΣ Β' θετική - Τεχνολογική Κατεύθυνση

ΟΛΟΚΛΗΡΩΜΑΤΑ

8 θt:ωQία 8 Μεθοδολοy{α

• ΙlαQα'fηρήσει;: - Σχιiλια 8 Λuμένα ΠαQαδείγματα

• ΕQιΙΙtήσεις Κuταν6ηnης • Λυκήσεις- θέματα

)ΔΙΟΦΑΝτΟΣ ΗΔΟΣF.!Σ-

Γιώργος Μ. Μιχαηλίδης

I�J�t=Hiι'J�'il�, Γ ΛΥΚΕΙΟΥ

θετική - Τεχνολογική Κατεύθυνση

eξeΊ;άσe ι ς 2008 - 2009

Πιθανά θέμα-&α

Γιώργος Μ. Μιχαηλίδης Octavίan Ν. Stanasίla

(Prof.unι..Po-. BuehaιesQ

Γιώργος Μ. Μιχαηλίδης

< (.) ϊ= cι:: a: ::ε c::ι w ...ι .... .... cι:: < :ε !:: cι:: :::! (.) CCI - < 1- α:ι � c::ι ι- α: < α.. � ==

ι α: _ c::ι - w Ι- :ι:

� �-::ι u..

Octavian Ν. Stanasίla (Prof. Unlv. Poιηehnlcι βucharest)

I ι'1 �Η]: I ι'1 �' i I :t, Γ'ΛΥΚΕΙΟΥ

Γενικής Ποιδείος

:> σuνιφrήσειs :> σrιπισrική

:> πιaav6rηrεs

� �

ΔωΦΑΝΤΟΣ ΕΚΔΟΣΕΙΣ